505 Test 2 Review

Réussis tes devoirs et examens dès maintenant avec Quizwiz!

The nurse is administering an intravenous analgesic to a laboring woman. The woman inquires as to why the nurse is waiting for a contraction to begin before she infuses the medication into the intravenous line. Which is the nurse's most appropriate response?

"Because the uterine blood vessels constrict during a contraction, the fetus will be less affected by the medication."

A client in active labor has severe second-degree burns on her buttocks. When questioned about the burns, the client replies, "I was trying to use that hot water thing to help my hemorrhoids." What should the nurse say in response to this client?

"Does your doctor know about the burns?"

Which statement, if made by the laboring client, most likely indicates that the client is in the second stage of labor?

"I feel like I need to push."

A nurse is providing instructions to a mother who is bottle-feeding a baby and who is complaining of breast engorgement. Which statement by the client indicates a need for further instructions?

"I should avoid wearing a bra at this time." (Wearing a bra or applying a breast binder applies pressure, which reduces congestion and discomfort. Ice packs reduce circulation and thus congestion and also provide an anesthetic effect. Analgesics help relieve the pain.)

A nurse is providing discharge teaching to a client who is 3 days postoperative following a cesarean birth. Which of the following client statement should indicate to the nurse the teaching is effective?

"I will resume taking my prenatal vitamins" "I will call my provider if I have discharge from my incision" "I should not have unrelieved pain in my abdomen"

A nurse is caring for a client who is postpartum. The nurse should recognize which of the following statement by the client as an indication of inhibition of parental attachment?

"I wish he had more hair. I will keep a hat on his head until he grows some."

A nurse is teaching a client who is breastfeeding about dietary recommendations. Which of the following statement by the client indicates understanding of the teaching?

"I'll eat more protein at each meal"

A postpartum client who is bottle-feeding her neonate asks the nurse when she can expect her menstrual period to return. How should the nurse respond?

"In 7 to 9 weeks"

A client who gave birth 24 hours ago continues to experience urine retention after several catheterizations. The physician prescribes bethanechol (Urecholine), 10 mg by mouth three times per day. The client asks, "How does bethanechol act on the bladder?" How should the nurse respond?

"It stimulates the smooth muscle of the bladder."

A nurse is assigned to care for a client in the immediate postpartum period who received methylergonovine maleate (Methergine). The nurse determines the medication is effective when the client says:

"My afterpains are really strong." (Methylergonovine maleate is an ergot alkaloid that stimulates smooth muscles. Because the smooth muscle of the uterus is especially sensitive to the medication, it is used postpartally to stimulate the uterus to contract and control excessive blood loss. The client statements in options 1, 2, and 4 are not related to this medication.)

A nurse is providing teaching about newborn care to a client who is 2 hr postpartum. Which of the following statements by the client indicates a need for further teaching?

"My baby's temperature will be checked rectally every hour"

The nurse is reviewing true and false labor signs with a multiparous client. The nurse determines that the client understands the signs of true labor if she makes which statement?

"My contractions will increase in duration and intensity."

The new breast-feeding mother has been seen in the clinic for the treatment of mastitis. The nurse knows that the mother needs further teaching when the mother states:

"My left breast is sore, so I will offer only my right breast frequently for breast-feeding." (Failure to nurse equally on both sides will decrease the flow of milk through the breast, causing engorgement of the breast that has been offered less frequently)

The nurse encourages a postpartum client to discuss the childbirth experience. Which client outcome is most appropriate for this client?

"The client demonstrates the ability to integrate the childbirth experience and progress to the task of maternal role attainment."

A new mother is seen in the health care clinic 2 weeks after the birth of a healthy newborn. The mother says that she feels as though she has the flu and complains of fatigue and aching muscles. On further data collection the nurse notes a localized area of redness on the left breast, and the mother is diagnosed with mastitis. The mother asks the nurse how the condition occurs. The appropriate nursing response is which of the following?

"The infection can occur at anytime during breast-feeding." (Mastitis is an infection of the lactating breasts and occurs most often during the second and third weeks after birth, although it may develop at any time during breast-feeding. It is more common in mothers nursing for the first time and usually affects at least one breast. Constriction of the breasts from a bra that is too tight may interfere with emptying of all the ducts and may lead to infection.)

After surgical evacuation and repair of a vaginal hematoma, a 3-day postpartum mother is discharged. The nurse determines that the mother needs further discharge instructions if the new mother states:

"The only medications that I will take are prenatal vitamins and stool softeners." (After surgical evacuation and repair of a vaginal hematoma, the client will need an antibiotic because she is at increased risk for infection because of the break in skin integrity and collection of blood at the hematoma site. The client statements in options 1, 2, and 3 indicate that the client understands the necessary home care measures.)

A nurse is caring for a client who is beginning to breastfeed her newborn after delivery. The new mother states, "I don't want to take anything for pain because I am breastfeeding." Which of the following statements should the nurse make?

"We can time your pain medication so that you have an hour or two before the next feeding"

Accompanied by her husband, a client seeks admission to the labor and delivery area. She states that she's in labor and says she attended the facility clinic for prenatal care. Which question should the nurse ask her first?

"What is your expected due date?"

A stillborn was delivered in the birthing suite a few hours ago. After the birth, the family has remained together, holding and touching the baby. Which statement by the nurse should further assist the family in their initial period of grief?

"Would you like to hold your baby?" (Nurses should explore measures that assist the family to create memories of an infant so that the existence of the child is confirmed and the parents can complete the grieving process.)

The mother of a neonate expresses concern about how she'll continue to breast-feed when she returns to work in 6 weeks. What is the best response by the nurse?

"You can continue breast-feeding after you go back to work. You can pump your breasts and put the milk in a bottle."

A nurse is assisting a client who is postpartum with her first breastfeeding experience. When the client asks how much of the nipple she should put into the newborn's mouth, which of the following responses should the nurse make?

"You should place your nipple and some of the areola into her mouth"

a. b. c. d.

() (a-) (b-) (c-) (d-)

(SELECT ALL THAT APPLY) Which conditions are contraindications to epidural blocks?

(1) Infection at the injection site, (2) Allergy to the anesthetic drug, (4) Anticoagulant therapy, (5) Bleeding disorder

(SELECT ALL THAT APPLY) The nurse is collecting data on client who is 1 day postpartum. The nurse expects which normal findings?

(1) Lochia Rubra, (4) Heart rate of 50 to 70 beats/minute

(SELECT ALL THAT APPLY) A primigravida experiences spontaneous rupture of the membranes. What should the nurse do?

(1) Perform a nitrazine test to confirm that the membranes are ruptured, (2) Monitor fetal heart rate and pattern., (3) Assess maternal temperature.

Latent phase of labor:

0-3 cm mild to moderate contractions irregular q 5-30 min lasts 30-40 sec

The physician prescribes phytonadione (AquaMEPHYTON), 0.5 mg I.M., for a neonate born 30 minutes ago. The nurse has a solution containing 2 mg/ml. How many milliliters of solution should the nurse administer to achieve this dose?

0.25

nurse in prenatal clinic caring for client, pregnant, asks nurse of her EDB, LMP began july 27, what is EDB?

05 04 30 days in april + 7 days = may 4 SUBTRACT 3 MONTHS FROM LMP month ADD 7 DAYS

how much is one gram of weight when measuring a peripad for lochia discharge?

1 ml of blood

how many days after birth will the fundus no longer be palpable?

10 days

A client has just begun taking an oral contraceptive that contains estrogen and progestin. The nurse should explain that full contraceptive benefits won't occur until the client has taken the drug for at least:

10 days.

CVS can be done at

10-12 weeks Chorionic villus sampling (CVS) is a prenatal test that diagnoses chromosomal abnormalities such as Down syndrome, as well as a host of other genetic disorders. The doctor takes cells from tiny fingerlike projections on your placenta called the chorionic villi and sends them to a lab for genetic analysis.

The nurse determines that a postpartum client's perineal pad weighs 100 g. The nurse should document this client's blood loss as:

100 ml

Rise in temp is normal after birth. What is the norm?

100.4

How much blood is lost during a c-section?

1000 mL

Labor typically begins within ___ hours after ROM.

12

When do WBC return to normal after birth?

12 days

What is the normal range of WBC postpartum?

12,000-20,0000

How long should the newborn nurse?

15-20 minutes (but don't educate the mother on the timing)

Pregnant mothers should consume how much water each day?

2 to 3 Liters of water from food and beverage sources.

PP the mom should consume ______ to_______ ml of water each day.

2000-3000

The nurse is reviewing the record of a client in the labor room and notes that the health care provider has documented that the fetal presenting part is at the -1 station. This documented finding indicates that the fetal presenting part is located at which area? Click on the image to indicate your answer.

3.

normal newborn RR

30-60

GBS can be tested when?

35-37 week GA

nurse assessing NB 1 hr after birth, RR w/in expected range?

48/ min 30-60bpm

When should the uterus return to the prepregnant size?

5-6 weeks after birth

Encourage women who are lactating to add an additional ___ calories/day to their prepregnancy diet.

500

It is recommended that clients who are lactating consume ____ mcg of folic acid.

500

A nurse is providing nutritional counseling to a new mother who is breast-feeding her newborn. The nurse instructs the mother that her calorie intake needs to increase by approximately:

500 calories per day (If the mother is breast-feeding, her calorie needs increase by approximately 500 calories per day. The mother should also be instructed regarding the need for increased fluids and the need for prenatal vitamins and iron supplements.)

Newborn temp should be

97.7-98.9

biophysical profile (BPP) normal score is

A biophysical profile (BPP) test measures the health of your baby (fetus) during pregnancy. 8-10

An assisted birth using forceps or a vacuum extractor may be performed for ineffective pushing, for large infants, to shorten the second stage of labor, or for a malpresentation. The nurse caring for the mother following an assisted birth should keep which of the following in mind?

A vacuum extractor is safer than forceps because it causes less trauma to the baby and the mother's perineum.

A client is recovering in the labor and delivery area after delivering a 6-lb, 3-oz boy. On assessment, the nurse finds that the client's fundus is firm and located two fingerbreadths below the umbilicus. Although she didn't have an episiotomy, her perineal pad reveals a steady trickle of blood. What is the probable cause of these assessment findings?

A vaginal laceration

A nurse is teaching about fetal development to a group of clients in the antenatal clinic. Which of the following statements should the nurse include in the teaching? A. "The baby's heart beat is audible by a Doppler stethoscope at 12 weeks of pregnancy." B. "The sex of the baby is determined by week 8 of pregnancy." C. "Very fine hairs, called lanugo, cover your baby's entire body by week 36 of pregnancy." D. "You will first feel your baby move in week 24 of pregnancy."

A. "The baby's heart beat is audible by a Doppler stethoscope at 12 weeks of pregnancy." i: The fetal heartbeat is audible by Doppler stethoscope between 8 to 17 weeks of gestation.

A nurse in a prenatal clinic is caring for a client who believes that she might be pregnant because she feels the baby moving. Which of the following statements should the nurse make? A. "This is a presumptive sign of pregnancy." B. "This is a probable sign of pregnancy." C. "This is a possible sign of pregnancy." D. "This is a positive sign of pregnancy."

A. "This is a presumptive sign of pregnancy." i: Presumptive signs of pregnancy include physical changes that are apparent to the client, such as quickening.

A nurse midwife is examining a client who is a primigravida at 42 weeks of gestation and states that she believes she in in labor. Which of the following findings confirm to the nurse that the client is in labor? A. Cervical dilation B. Reports of pain above the umbilicus C. Brownish vaginal discharge D. Amniotic fluid in the vaginal vault

A. Cervical dilation i: Cervical dilation and effacement are indications of true labor.

A nurse in a prenatal clinic is caring for a client who asks what her estimated date of delivery will be if her last menstrual period was May 4, 2015. Which of the following is the appropriate response by the nurse? A. February 11, 2016 B. February 27, 2016 C. April 27, 2016 D. April 11, 2016

A. February 11, 2016 i: Subtracting 3 calendar months and adding 7 days plus one year will result in this estimated date of delivery.

Thrombophlebitis: position arm:

Above the lever of the heart

After a client enters the second stage of labor, the nurse notes that her amniotic fluid is port-wine colored. What does this finding suggest?

Abruptio placentae

At her follow-up examination, a client who's 6 weeks postpartum tells the nurse that she's exhausted and sore from breast-feeding and wants to formula-feed her baby. She also mentions that she feels like a failure and finds it increasingly difficult "just to get out of bed in the morning." Which intervention should the nurse attempt before notifying the physician?

Acknowledging the client's feelings, asking about other life stressors, and identifying the client's support system

A postpartum client tells the nurse she isn't having regular bowel movements. The nurse should recommend that the client do what to combat constipation?

Add high-fiber foods to her diet.

Fetal distress is occurring with a woman in labor. As the nurse prepares her for a cesarean birth, what other intervention should the nurse implement?

Administer oxygen at 8 to 10 L/min via face mask.

Which action should the nurse perform if the client's blood pressure falls during the first or second stage of labor?

Administer oxygen through a face mask at 6 to 10 L/minute.

A breast-feeding client is diagnosed with mastitis. Which nursing intervention would be most helpful to her?

Advising her to massage the affected area gently while breast-feeding

An amniocentesis may be performed when?

After 14 weeks gestation

When should you start measuring a woman's fundal height?

After > 12 weeks GA

Which of the following physiologic changes during labor makes it necessary for the nurse to check blood pressure frequently?

Alterations in cardiovascular function affect the fetus.

The nurse is assigned to care for the client after a cesarean section. To prevent thrombophlebitis, the nurse encourages the woman to take which priority action?

Ambulate frequently. (Stasis is believed to be a major predisposing factor for the development of thrombophlebitis. Because cesarean delivery poses a risk factor, the client should ambulate early and frequently to promote circulation and prevent stasis.)

Which of the following behaviors would cause the nurse to suspect that a client's labor is moving quickly and that the physician should be notified?

An increased sense of rectal pressure

A nurse is caring for a client who is considering several methods of contraception. Which of the following methods of contraception should the nurse identify as being most reliable?

An intrauterine device (IUD)

A nurse is reviewing the health history of a client who has a new prescription for a combined oral contraceptive (COC). The nurse recognizes that which of the following client medications can interfere with the effectiveness of the COC?

Anticonvulsants

When caring for a client who's a primigravida, the nurse would expect the second stage of labor to last how long?

Approximately 2 hours

A client who's in active labor is yelling, "Get out of here!" As the nurse enters the client's room, she notices the client's estranged husband in the room. How should the nurse intervene?

Ask the husband to leave or phone security.

A client in labor develops complications and is given general anesthesia. The physician informs the husband that without emergency cesarean delivery his wife and the baby may die. The husband asks the nurse what he should do. How should the nurse respond?

Ask the husband whether he and his wife ever discussed treatment options in the event of an emergency.

A client in labor for the past 10 hours shows no change in cervical dilation and has stayed at 5 to 6 cm for the past 2 hours. Her contractions remain regular at 2-minute intervals, lasting 40 to 45 seconds. Which of the following would be the nurse's initial action?

Assess for presence of a full bladder.

The nurse is admitting a pregnant client to the labor room and attaches an external electronic fetal monitor to the client's abdomen. After attachment of the electronic fetal monitor, what is the next nursing action?

Assess the baseline fetal heart rate.

The nurse assists the health care provider to perform an amniotomy on a client in labor. Which is the priority nursing action after this procedure?

Assess the fetal heart rate.

After the spontaneous rupture of a laboring woman's membranes, the fetal heart rate drops to 85 beats/minute. Which should be the nurse's priority action?

Assess the vagina and cervix with a gloved hand.

The nurse is caring for a client who just delivered triplets. Which intervention by the nurse is most important?

Assessing fundal tone and lochia flow

Which care intervention is appropriate for the fourth stage of labor?

Assessing lochia and the location and consistency of the fundus

The nurse understands that measures are necessary to contain health care costs. Which intervention demonstrates effective resource management?

Assigning the nurse's aide to deliver meal trays and to stock rooms; assigning the licensed practical nurse to collect assessment data

A client in the early stages of active labor wants to get out of bed and walk around the room. Which action by the nurse is best?

Assisting the client to ambulate in the room

A nurse is caring for a client who is experienced a vaginal delivery 12 hr ago. When palpating the client's abdomen, at which of the following positions should the nurse expect to find the uterine fundus?

At the level of the umbilicus

After delivering a neonate, a client delivers the placenta. At this time, where does the nurse expect to palpate the uterine fundus?

At the midline, 0.4" to 0.8" (1 to 2 cm) below the umbilicus

A nurse is caring for a client who is receiving opioid epidural analgesia during labor. Which of the following findings is the nurse's priority? A. The client reports weakness of the lower extremities. B. Blood pressure 80/56 mmHg C. Temperature 38.2C / 100.8F D. The client reports perfuse itching.

B. Blood pressure 80/56 mmHg i: When using the airway, breathing, circulation approach to client care, the nurse's priority finding is a blood pressure of 80/56, which indicates hypotension. The client's blood pressure is not adequate to sustain uteroplacental perfusion and oxygen to the fetus, which can lead to respiratory distress and possibly death.

A nurse is caring for a client who is primigravida, at term, and having contractions but is statins that she is "not really sure if she is in labor or not." Which of the following should the nurse recognize as a sign of true labor? A. Rupture of the membranes B. Changes in the cervix C. Station of the presenting part D. Pattern of contractions

B. Changes in the cervix i: Assessment of progressive changes in the effacement and dilation of the cervix is the most accurate indication of true labor.

A nurse is preparing to measure the fundal height of a client who is at 22 weeks of gestation. At which location should the nurse expect to palpate the fundus? A. 3cm above the umbilicus B. Slightly above the umbilicus C. Slightly below the umbilicus D. 3cm below the umbilicus

B. Slightly above the umbilicus i: At 22 weeks of gestation, the fundal height should be just above the level of the umbilicus. The distance in centimeters from the symphysis pubis to the top of the fundus is a gross estimate of the weeks of gestation.

A client is progressing through the first stage of labor. Which finding signals the beginning of the second stage of labor?

Bearing-down reflex

A client who is breast-feeding her newborn infant is experiencing nipple soreness. To relieve the soreness, the nurse suggests that the client:

Begin feeding on the less sore nipple. (The nurse would instruct the mother to begin feeding on the less sore nipple. The infant sucks with greater force at the beginning of feeding. Rotating breast-feeding positions, breaking suction with the little finger, nursing frequently, not allowing the newborn to chew on the nipple or to sleep holding the nipple in the mouth, and applying tea bags soaked in warm water to the nipple are also measures to alleviate nipple soreness. Options 1, 2, and 3 are incorrect measures.)

During labor, a client asks the nurse why her blood pressure must be measured so often. Which explanation should the nurse provide?

Blood pressure reflects changes in cardiovascular function, which may affect the fetus.

The nurse is caring for a client who is experiencing a precipitous labor and is waiting for the health care provider to arrive. When the infant's head crowns, what instruction should the nurse give the client?

Breathe rapidly

A nurse in a provider's office is caring for a client who is at 36 weeks of gestation and scheduled for an amniocentesis. The client asks why she is having an ultrasound prior to the procedure. Which of the following is an appropriate response by the nurse? A. "This will determine is there is more than one fetus." B. "It is useful for estimating fetal age." C. "It assists in identifying the location of the placenta and fetus." D. "This is a screening tool for spina bifida."

C. "It assists in identifying the location of the placenta and fetus." i: Identifying the positions of the fetus, placenta, and amniotic fluid pockets immediately prior to the amniocentesis increases the safety of this test by assisting with correct placement of the needle.

A nurse is teaching a client who is at 23 weeks of gestation about immunizations. Which of the following statements should the nurse include in the teaching? A. "You should not receive the rubella vaccine while breastfeeding." B. "You should receive a varicella vaccine before you deliver." C. "You can receive an influenza vaccination during pregnancy." D. "You cannot receive the Tdap vaccine until after you deliver."

C. "You can receive an influenza vaccination during pregnancy." i: It is recommended that pregnant women receive annual influenza vaccinations.

If the ovum is to be fertilized, where is the most common site of fertilization? A. Uterine wall B. Fimbria C. Ampulla of the fallopian tube D. Isthmus of the fallopian tube

C. Ampulla of the fallopian tube i: This is correct- this is the outer 1/3 of the fallopian tube

A nurse is caring for a client who is in active labor with 7cm of cervical dilation and 100% effacement. The fetus is at 1+ station, and the client's amniotic membranes are intact. The client suddenly states that she needs to push. Which of the following actions should the nurse take? A. Assist the client into a comfortable position. B. Observe the perineum for signs of crowning. C. Have the client pant during the next contractions. D. Help the client to the bathroom to void.

C. Have the client pant during the next contractions. i: Panting is rapid, continuous, shallow breathing. It helps a client in labor refrain from pushing before her cervix reaches full dilation. Observe for hyperventilation and have the client exhale slowly through pursed lips.

A nurse is caring for a client who is gravida 3, para 2, and is in active labor. The fetal head is at 3+ station after a vaginal examination. Which of the following actions should the nurse take? A. Apply fundal pressure. B. Observe for the presence of a nuchal cord. C. Observe for crowning. D. Prepare to administer oxytocin.

C. Observe for crowning. i: In the descent phase of the second stage of labor, crowning occurs when the fetal head is at +2 to +4 station. Because this is the client's third childbirth experience, it is reasonable to assume that delivery is imminent.

A nurse is caring for a client who is having a nonstress test performed. The fetal heart rate is 130 to 150/min, but there has been no fetal movement for 15min. Which of the following actions should the nurse perform? A. Immediately report the situation to the client's provider and prepare the client for induction of labor. B. Encourage the client to walk around without the monitoring unit for 10min, then resume monitoring. C. Offer the client a snack of orange juice and crackers. D. Turn the client onto her left side.

C. Offer the client a snack of orange juice and crackers. i: A nonstress test depends upon fetal movement, and this fetus is most likely asleep. Most fetuses are more active after meals due to the increase in the mother's blood sugar. Giving the mother a snack will promote fetal movement.

A nurse is caring for a client who is at 18 weeks gestation. The client tells the nurse that she felt fluttering movements in her abdomen 3 days ago. The nurse should interpret this finding as which of the following? A. Ballottement B. Lightening C. Quickening D. Chloasma

C. Quickening i: Clients describe quickening as a fluttering sensation, which can be felt as early as the 14th week of gestation. It reflects fetal movement.

A client with intrauterine growth retardation is admitted to the labor and delivery unit and started on an I.V. infusion of oxytocin (Pitocin). Which action should be included in the plan of care?

Carefully titrating the oxytocin based on the client's pattern of labor

A client with active genital herpes is admitted to the labor and delivery area during the first stage of labor. Which type of birth should the nurse anticipate for this client?

Cesarean

A nurse is caring for a client who is a primigravida, at term, and having contractions but is stating that she is "not really sure if she is in labor or not." Which of the following should the nurse recognize as a sign of true labor?

Changes in the cervix

A client in the second stage of labor experiences membrane rupture. Which intervention by the nurse is appropriate?

Check for a prolapsed cord.

The nurse prepares a plan of care for the client with preeclampsia and documents that if the client progresses from preeclampsia to eclampsia, the nurse should take which first action?

Clear and maintain an open airway.

Certain drugs used during the postpartum period may affect blood pressure. Which drug would decrease a postpartum client's blood pressure?

Codeine phosphate

A nurse is caring for a woman who is being treated with antibiotics for mastitis. The nurse reinforces instructions and tells the woman to:

Complete the entire antibiotic regimen. (If antibiotics are prescribed, the client must complete the regimen even though symptoms will be reduced in 24 to 48 hours. Options 1, 3, and 4 are inappropriate treatment measures for mastitis. The client should breast-feed, wear a supportive bra, and take analgesics as prescribed.)

Early decels mean:

Compression of the fetal head resulting from uterine contraction Vaginal exam Fundal pressure

A client who tested positive for the human immunodeficiency virus (HIV) is in active labor. During delivery, blood splashes and contaminates the care area. Which action should the nurse take?

Contact housekeeping and ask them to clean the area because it has been contaminated by blood-borne pathogens.

A client who's in labor and who attended natural childbirth classes is asking for something for pain. What should the nurse do?

Contact the physician and support the client until something can be ordered for pain.

The nurse is preparing to care for a client in labor. The health care provider (HCP) has prescribed an intravenous (IV) infusion of oxytocin. The nurse should ensure that which is implemented before the beginning of the infusion?

Continuous electronic fetal monitoring

There is a high risk of _________ surrounding external cephalic version.

Cord prolapse

A nurse is providing teaching about expected gestational changes to a client who is at 12 weeks of gestation. Which of the following statements by the client indicates a need for further teaching? A. "I will reduce my stress level." B. "I will tell my doctor before using home remedies for nausea." C. "I will monitor my weight gain during the remaining months." D. "I will use only nonprescription medications while pregnant."

D. "I will use only nonprescription medications while pregnant." i: Both nonprescription and prescription medications can be harmful to the fetus. The client needs to understand the importance of disclosing all medications, supplements, and vitamins to the provider prior to use during pregnancy.

A nurse in a prenatal clinic is caring for a client who is at 7 weeks of gestation. The client reports urinary frequency and asks if this will continue until delivery. Which of the following responses should the nurse make? A. "It's a minor inconvenience, which you should ignore." B. "In most cases it only lasts until the 12th week, but it will continue if you have poor bladder tone." C. "There is no way to predict how long it will last in each individual client." D. "It occurs during the first trimester and near the end of the pregnancy."

D. "It occurs during the first trimester and near the end of the pregnancy." i: Urinary frequency is due to increased bladder sensitivity during the first trimester and recurs near the end of the pregnancy as the enlarging uterus places pressure on the bladder.

A nurse is providing teaching about nutrition to a client at her first prenatal visit. Which of the following statements by the nurse should be included in the teaching? A. "You will need to increase your calcium intake during breastfeeding." B. "Prenatal vitamins will meet your need for increased vitamin D during pregnancy." C. "Vitamin E requirements decline during pregnancy due to the increase in body fat." D. "You will need to double your intake of iron during pregnancy."

D. "You will need to double your intake of iron during pregnancy." i: During pregnancy, the need for iron increases to allow transfer of the appropriate amounts to the fetus and to support expansion of the client's red blood cell volume.

A patient is trying to prevent pregnancy by using the rhythm method. On what cycle day is she the most fertile. A. 21-28 B. 1-6 C. 6-9 D. 9-16

D. 9-16 i: This time period allows for early ovulation and for the egg that lives for 2 days.

A nurse is caring for a client who is at 40 weeks gestation and is in active labor. The client has 6cm of cervical dilation and 100% cervical effacement. The nurse obtains the client's blood pressure reading as 82/52mmHg. Which of the following nursing interventions should the nurse perform? A. Prepare for a cesarean birth. B. Assist the client to an upright position. C. Prepare for an immediate vaginal delivery. D. Assist the client to turn onto her side.

D. Assist the client to turn onto her side. i: Maternal hypotension results from the pressure of the enlarged uterus on the inferior vena cava. Turning the client to her right side relieves this pressure and restores blood pressure to the expected reference range.

A nurse is caring for a client following an amniotomy who is now in the active phase of the first stage of labor. Which of the following actions should the nurse implement with this client? A. Maintain the client in the lithotomy position. B. Perform vaginal examinations frequently. C. Remind the client to bear down with each contraction. D. Encourage the client to empty her bladder every 2hr.

D. Encourage the client to empty her bladder every 2hr. i: A client in labor should be encouraged to empty her bladder every 2 hr. Bladder distention can impede the descent of the fetus and slow the progression of labor. It can also contribute to uterine atony after delivery, increasing the client's risk of postpartum hemorrhage.

A nurse is caring for a client who has a positive pregnancy test. The nurse is teaching the client about common discomforts in the first trimester of pregnancy as well as warning signs of potential danger. The nurse should instruct the client to call the clinic if she experiences which of the following manifestations? A. Leukorrhea B. Urinary frequency C. Nausea and vomiting D. Facial edema

D. Facial edema i: Facial edema is a warning sign of a hypertensive condition or preeclampsia and should be reported immediately to the provider.

A nurse in a prenatal clinic is caring for a client who is at 12 weeks gestation. The client asks about the cause of her heartburn. Which of the following responses should the nurse make? A. Retained bile in the liver results in delayed digestion. B. Increased estrogen production causes increased secretion of hydrochloric acid. C. Pressure from the growing uterus displaces the stomach. D. Increased progesterone production causes decreased motility of smooth muscle.

D. Increased progesterone production causes decreased motility of smooth muscle. i: Increased progesterone production causes a relaxation of the cardiac sphincter of the stomach and delayed gastric emptying, which can result in heartburn.

A nurse is caring for a client who is in labor and has an epidural anesthesia block. The client's blood pressure is 80/40 mmHg and the fetal heart rate is 140/min. Which of the followign is the priority nursing action? A. Elevate the client's legs. B. Monitor vital signs every 5 min. C. Notify the provider. D. Place the client in a lateral position.

D. Place the client in a lateral position. i: Based on Maslow's hierarchy of needs, the client should be moved to a lateral position or a pillow placed under one of the client's hips to relieve pressure on the inferior vena cava and improve the blood pressure.

A nurse in a prenatal clinic is caring for a client. Using Leopold maneuvers, the nurse palpates a round, firm, moveable part in the fundus of the uterus and a long, smooth surface on the client's right side. In which abdominal quadrant should the nurse expect to auscultate fetal heart tones? A. Left lower B. Right lower C. Left upper D. Right upper

D. Right upper i: Fetal heart tones are best auscultated directly over the location of the fetal back, which, in this breech presentation, would be in the right upper quadrant.

nurse is caring for client w/ preeclampsia and is being tx with MgSO4 IV, clients RR is 10/min. DTR absent. priority action?

D/C medication infusion

nurse caring for adolescent who is G1P0, admitted to hospital at 38wks with dx of preeclampsia, findings that are inconsistent with preeclampsia

DTR - +1 should be increased. OTHER - expected findings 1+ pitting edema +3 proteinuria in urine BP 148/98

A nurse is assessing a client who is 4 hr postpartum following a vaginal delivery. Which of the following findings should the nurse identify as the priority?

Deep tendon reflexes 4+

A nurse is performing a physical examination of a client who is 1 day postpartum. Which of the following findings requires immediate interventions?

Displaced fundus from the midline

The nurse in the labor room is caring for a client who is in the first stage of labor. On assessing the fetal patterns, the nurse notes an early deceleration of the fetal heart rate (FHR) on the monitor strip. Based on this finding, which is the appropriate nursing action?

Document the findings and continue to monitor fetal patterns.

A nurse is caring for a client who is 1 hr postpartum and observes a large amount of lochia rubra and several small clots on the client's perineal pad. The fundus is midline and firm at the umbilicus. Which of the following actions should the nurse take?

Document the findings and continue to monitor the client

The nurse is caring for a client in active labor. Which nursing intervention would be the best method to prevent fetal heart rate (FHR) decelerations?

Encourage an upright or side-lying maternal position.

A postpartum nurse obtains the vital signs on a mother who delivered a healthy newborn infant 2 hours ago. The mother's temperature is 100° F (38° C). The initial nursing action would be to:

Encourage oral fluid intake. (During the first 24 hours following delivery, the mother's temperature may rise to 100° F (38° C) as a result of the dehydrating effects of labor. Therefore the initial nursing action is to encourage fluid intake. The nurse would document the temperature, but this is not the initial action. Options 2 and 3 are not necessary at this time)

Reactive NST

FHR is a normal baseline rate with moderate variablity Two accelerations to 15 bpm for at least 15 secs in 20 min period

The nurse is caring for a client in the transition phase of the first stage of labor. The client is experiencing uterine contractions every 2 minutes and she cries out in pain with each contraction. What is the nurse's best interpretation of this client's behavior?

Fear of losing control

The nurse notices that a client in the first stage of labor seems agitated. When the nurse asks why she's upset, she begins to cry and says, "I guess I'm a little worried. The last time I gave birth, I was in labor for 32 hours." Based on this information, the nurse should expect which nursing diagnosis in the client's plan of care?

Fear related to a potentially difficult childbirth

Which assessment following an amniotomy should be conducted first?

Fetal heart rate pattern

A client in labor is attached to an electronic fetal monitor (EFM). Which of the following data provided by an EFM most reliably indicates adequate uteroplacental and fetal perfusion?

Fetal heart rate variability within an acceptable range

Regarding kick counts, what are signs that a woman needs further evaluation?

Fetal movements of less than 3 in one hour No fetal movement for 12 hours

A 28-year-old woman gave birth 1 hour ago to a full-term baby boy. Which finding should the nurse expect when palpating the client's fundus?

Firm, at the level of the umbilicus

The nurse in a labor room is assisting with the vaginal delivery of a newborn infant. The nurse should monitor the client closely for the risk of uterine rupture if which occurred?

Forceps delivery

Fetal descent and cervical dilation are caused by

Frequency, duration, and strength of contractions

A nurse is caring for a client who is postpartum and received methylergonovine. Which of the following findings indicates that the medication was effective?

Fundus firm to palpation

A nurse is caring for a client who is 12 hr postpartum following a vaginal delivery. Which of the following findings should the nurse expect?

Fundus firm, at the level of the umbilicus

A nurse is caring for a client who is 2 hr postpartum following a vaginal birth. Which of the following findings indicates the client's bladder is distended?

Fundus palpable to right of midline

Betamethoasone

Glucocorticoid given to enhance fetal lung maturity and surfactant production.

a nurse in a providers office is caring for a client who is 34 weeks and at risk for placental abruption. nurse should recognize which common risk factor for abruption?

HTN

Accelerations mean:

Healthy fetal/placental exchange

A nurse is planning for a nursery room admission of a large-for-gestational-age (LGA) infant. In getting ready to care for this infant, the nurse prepares equipment for which diagnostic test?

Heelstick blood glucose (After birth, the most common problem in the LGA infant is hypoglycemia, especially if the mother has diabetes mellitus. At delivery when the umbilical cord is clamped and cut, the maternal blood glucose supply is lost. The newborn continues to produce large amounts of insulin, which depletes the infant's blood glucose within the first hours after birth. If immediate identification and treatment of hypoglycemia are not performed, the newborn may suffer central nervous system damage because of inadequate circulation of glucose to the brain. Indirect and direct bilirubin levels are usually prescribed after the first 24 hours because jaundice is usually seen at 48 to 72 hours after birth. There is no rationale for prescribing an Rh and ABO blood type unless the maternal blood type is O or Rh negative. Serum insulin levels are not helpful because there is no intervention to decrease these levels in order to prevent hypoglycemia.)

A client who comes to the emergency department in the early stages of labor is admitted to the labor and delivery unit. The nurse observes that the client hasn't recently bathed or changed her clothes. What should the nurse do to help this client?

Help the client to undress and suggest a quick bath to freshen up.

(SELECT ALL THAT APPLY) A client is at risk for which postpartum complication during the fourth stage of labor?

Hemorrhage

Thrombosis tx drugs:

Heparin Warfarin (coumadin)

A client is receiving oxytocin (Pitocin) to treat postpartum hemorrhage. When planning the client's care, the nurse anticipates monitoring for which common adverse reactions?

Hypertension and tachycardia

A client in labor receives epidural anesthesia. The nurse should assess carefully for which adverse reaction to the anesthetic agent?

Hypotensive crisis

a nurse is completing HX for client at 6wks gest. Client informs the nurse that she smokes one pack of cig a day. the nurse should advise the client that smoking places the clients NB at risk for...?

IUGR - intrauterine growth restriction placental abruption placenta previa preterm delivery fetal death

A client with Rh isoimmunization delivers a neonate with an enlarged heart and severe, generalized edema. Which nursing diagnosis is most appropriate for this client?

Impaired parenting related to the neonate's transfer to the intensive care unit

A client arrives at a birthing center in active labor. Following examination, it is determined that her membranes are still intact and she is at a -2 station. The health care provider prepares to perform an amniotomy. What will the nurse relay to the client as the most likely outcomes of the amniotomy? Select all that apply.

Increased efficiency of contractions The need for frequent fetal heart rate monitoring to detect the presence of a prolapsed cord

The nurse-manager overhears a nurse tell a client, "If I were you, I'd ask the doctor for something for pain; you shouldn't have to suffer during labor." How should the nurse-manager respond to the nurse's comment?

Inform the nurse that she'd like to speak with her, then discuss the inappropriateness of her comment in a private location.

A client in the early stage of labor states that she has a thick, yellow discharge from both of her breasts. Which action by the nurse is most appropriate?

Informing the client that the discharge is colostrum, which is a normal finding

Following a precipitous delivery, examination of the client's vagina reveals a fourth-degree laceration. Which of the following steps would be contraindicated when caring for this client?

Instructing the client to use two or more peripads to cushion the area

A nurse is teaching a client who is postpartum and has a new Rx of an injection of Rh0 (D) immunoglobulin. Which of the following should be included in the teaching?

It prevents the formation of Rh antibodies in mothers who are Rh negative

If a BPP comes back as 6.....

It should be retested

During an annual checkup, a client tells the nurse that she and her husband have decided to start a family. Ideally, when should the nurse plan for childbirth education to begin and end?

It should begin before conception and end 3 months after delivery.

Foods high in folic acid are:

Leafy vegetables Dried peas Dried beans Seeds Orange juice

A client has had a midline episiotomy. In relation to clients with other types of episiotomies, the nurse anticipates that the client will generally experience:

Less pain (Midline episiotomies are effective, easily repaired, and generally result in less pain. The blood loss is greater and the repair is more difficult and painful with the mediolateral than the midline episiotomy.)

On her 3rd postpartum day, a client complains of chills and aches. Her chart shows that she has had a temperature of 100.6° F (38.1° C) for the past 2 days. The nurse assesses foul-smelling, yellow lochia. What do these findings suggest?

Localized infection

Smoking tobacco during pregnancy is associated with :

Low Birth Weight

Which intervention should be included in the safety plan for the maternal-infant unit?

Making sure that the spouse or significant other wears an identification band

A nurse is caring for a client who is 2 hr postpartum. The nurse notes that the client soaked a perineal pad in 10 min, the client's skin color is ashen, and she states she feels weak and light headed. After applying oxygen via nonrebreather face mask at 10 L/min which of the following actions should the nurse take next?

Massage the client's fundus to promote contractions

During the fourth stage of labor, the nurse notes that the client's fundus is boggy and located above the umbilicus. How should the nurse intervene?

Massage the client's fundus.

Fetal tachycardia means:

Maternal infection Fetal anemia Fetal heart failure Fetal cardiac dysrythmias Maternal use of cocaine or meth Maternal dehydration

A client with a 38-week twin gestation is admitted to a birthing center in early labor. One of the fetuses is a breech presentation. Which intervention is least appropriate in planning the nursing care of this client?

Measure fundal height.

Decrease or loss of variability means:

Meds that depress the CNS Fetal hypoxemia w/resulting acidosis Fetal sleep cycle congenital abnormalities

The goal for a woman with partial premature separation of the placenta is: "The woman will not exhibit signs of fetal distress." Which outcome, documented by the nurse, indicates that this goal has been achieved?

Moderate variability present

The nurse is creating a plan of care for a client experiencing dystocia and includes several nursing interventions in the plan. The nurse prioritizes the plan and selects which nursing intervention as the highest priority?

Monitoring fetal status

When caring for a client who has had a cesarean section, which of the following actions is appropriate?

Monitoring pain status and providing necessary relief

The nurse has created a plan of care for a client experiencing dystocia and includes several nursing actions in the plan of care. What is the priority nursing action?

Monitoring the fetal heart rate

The labor room nurse assists with the administration of a lumbar epidural block. How should the nurse check for the major side effect associated with this type of regional anesthesia?

Monitoring the mother's blood pressure

Epidural analgesia is administered to a woman for pain relief after a cesarean birth. The nurse assigned to care for the woman ensures that which medication is readily available if respiratory depression occurs?

Naloxone (Narcan) (Opioids are used for epidural analgesia. An adverse effect of epidural analgesia is a delayed respiratory depression. Naloxone (Narcan) is an opioid antagonist, which reverses the effects of opioids and is given for respiratory depression. Morphine sulfate and meperidine hydrochloride are opioid analgesics. Betamethasone is a corticosteroid that is administered to enhance fetal lung maturity.)

Absent or undetectable variability is considered:

Non-reasurring

During labor, a client demonstrates signs of fatigue and possible fetal distress. How should the nurse intervene?

Notify the charge nurse and prepare to assist her with necessary interventions.

The nurse is monitoring a client in active labor and notes that the client is having contractions every 3 minutes that last 45 seconds. The nurse notes that the fetal heart rate between contractions is 100 beats/minute. Which nursing action is most appropriate?

Notify the health care provider (HCP).

When assessing a postpartum client, the nurse notes a continuous flow of bright red blood from the vagina. The uterus is firm and no clots can be expressed. Which action should the nurse take?

Notify the physician.

For a client who's moving into the active phase of labor, the nurse should include which of the following as the priority of care?

Offer support by reviewing the short-pant form of breathing.

Meds given to tx PP hemorrhage

Oxytocin Methergine Cytotec Hemabate

nurse is preparing to admi methylergonovine IM to client who experienced vaginal delivery . nurse should explain to client, purpose of this medication to prevent, condition?

PPH

A prenatal client with vaginal bleeding is being admitted to the labor unit. The labor room nurse is performing the admission assessment and should suspect a diagnosis of placenta previa if which finding is noted?

Painless vaginal bleeding

The nurse is caring for a client in labor and prepares to auscultate the fetal heart rate (FHR) by using a Doppler ultrasound device. Which action should the nurse take to determine fetal heart sounds accurately?

Palpating the maternal radial pulse while listening to the FHR

Risk factors for subinvolution of uterus:

Pelvic infection and endometritis Retained placental fragments not completely expelled from the uterus

The nurse is reviewing the health care provider's (HCP's) prescriptions for a client admitted for premature rupture of the membranes. Gestational age of the fetus is determined to be 37 weeks. Which prescription should the nurse question?

Perform a vaginal examination every shift

A client is experiencing an early postpartum hemorrhage. Which action by the nurse is most appropriate?

Performing fundal massage

Positive CST is indicated when:

Persistent and consistent late decels on more than half of the contractions.

A nurse in the labor and delivery unit is caring for a client who is undergoing external fetal monitoring. The nurse observes that the fetal heart rate begins to slow after the start of a contraction and the lowest rate occurs after the peak of the contraction. Which of the following actions should the nurse take first?

Place the client in the lateral position.

Several minutes after a vaginal delivery, the nurse observes blood gushing from the client's vagina, umbilical cord lengthening, and a globular-shaped uterus. The nurse should suspect which condition?

Placental separation

The nurse assists in the vaginal delivery of a newborn infant. After the delivery, the nurse observes the umbilical cord lengthen and a spurt of blood from the vagina. The nurse documents these observations as signs of which condition?

Placental separation

The nurse assists in the vaginal delivery of a newborn. Following the delivery, the nurse observes the umbilical cord lengthen and a spurt of blood from the vagina. The nurse should document these observations as signs of which condition?

Placental separation

A mother is breast-feeding her newborn infant. The mother complains to the nurse that she is experiencing severe nipple soreness. The nurse should provide which of the following suggestions to the client?

Position the newborn infant with the ear, shoulder, and hip in straight alignment and with the baby's stomach against the mother's. (Severe nipple soreness most often occurs as a result of poor positioning, incorrect latch-on, improper suck, or monilial infection. Comfort measures for nipple soreness include positioning the newborn with the ear, shoulder, and hip in straight alignment and with the baby's stomach against the mother's.)

A nurse is preparing to administer methylergonovine IM to a client who experienced a vaginal delivery. The nurse should explain to the client that the purpose of this medication is to prevent which of the following conditions?

Postpartum hemorrhage

When checking a client 1 hour after vaginal delivery, the nurse notes blood gushing from the vagina, pallor, and a rapid, thready pulse. What do these data collection findings suggest?

Postpartum hemorrhage

A 23-year-old primigravida client is in the active stage of labor. She and her husband have been using breathing techniques. The husband asks whether he can do anything more to help his wife during labor. What should the nurse suggest?

Provide helpful distractions.

The nurse palpates the fundus and checks the character of the lochia of a postpartum client who is in the fourth stage of labor. The nurse expects the lochia to be:

Red (The color of the lochia during the fourth stage of labor is bright red, and this may last from 1 to 3 days. The color of the lochia then changes to a pinkish brown, and occurs from day 4 to 10 postpartum. Finally, the lochia changes to a creamy white color that occurs from day 10 to 14 postpartum.)

A nurse is caring a client who is 3 days postpartum and is attempting to breastfeed. Which of the following findings indicate mastitis?

Red and painful area in one breast

What does REEDA stand for?

Redness edema ecchymosis drainage approximation

An amnioinfusion can be used to:

Reduce severity of variable decelerations (cord compression) Dilute meconium-stained amniotic fluid

To obtain a good monitor tracing on a client in labor, the mother lies on her back. Suddenly, she complains of feeling light-headed and becomes diaphoretic. Which of the following should be the nurse's first action?

Reposition the client to her left side.

A client with a first-degree tear and swollen perineum is 28 hours postpartum when she requests assistance with her first sitz bath. Which intervention by the nurse is necessary at this time?

Requesting that the client call for assistance to walk back to bed when she's finished with the sitz bath

A 45-year-old woman delivered her first baby by cesarean section 5 days ago. The postpartum recovery has been complicated by thrombophlebitis in her left leg. She cries frequently and requests to have her newborn infant stay in the nursery. The nurse recognizes that the mother may have intensified "postpartum blues" because she is:

Required to stay on bedrest (Clients with thrombophlebitis are placed on bedrest with elevation of the affected extremity. Bedrest restricts normal newborn care, feeding, and parenting and will require interventions that promote attachment. )

Presence of PG on a fetal lung test is associated with:

Respiratory distress

Lochia normally progresses in which of the following patterns?

Rubra, serosa, alba

The nurse is assessing a woman in labor. Her cervix is dilated 8 cm. Her contractions are occurring every 2 minutes. She's irritable and in considerable pain. What type of breathing should the nurse instruct the woman to use during the peak of a contraction?

Shallow chest breathing

As a part of discharge teaching, a new mother has been provided with instructions about how to perform postpartum exercises. The nurse determines that the client understands the instructions when she states that:

She should alternately contract and relax the muscles of the perineal area. (Kegel exercises are extremely important to strengthen the muscle tone of the perineal area. Postpartum exercises can begin soon after birth. The initial exercises should be simple, with progression to increasingly strenuous exercises. Women who maintain the perineal muscle tone may benefit in later life by the development of less stress urinary incontinence.)

What is the sensation the patient will feel when completion of dilation and fetal descent?

She will feel like she needs to take a big ole poooo!

A nurse observes the client following delivery for normal maternal physiological changes that are anticipated. The nurse would document which expected changes?

Slowed pulse rate and elevated blood pressure (Following delivery of the placenta, the maternal cardiac system begins to make several normal changes, leading to slowing of the pulse rate and an elevation in blood pressure. Respirations usually remain unchanged and the client should be alert and oriented.)

The nurse is caring for a client who is receiving oxytocin for induction of labor and notes a nonreassuring fetal heart rate (FHR) pattern on the fetal monitor. On the basis of this finding, the nurse should take which action first?

Stop the oxytocin infusion.

The nurse is caring for a client in labor and notes that minimal variability is present on a fetal heart rate (FHR) monitor strip. Which conditions are most likely associated with minimal variability? Select all that apply.

Tachycardia Fetal hypoxia Metabolic acidemia Congenital anomalies

PP hemorrhage VS findings:

Tachycardia Hypotension

A client delivered a healthy full-term baby girl 2 hours ago by cesarean section. When assessing this client, which finding requires immediate nursing action?

Tachycardia and hypotension

Which of the following is the most serious adverse effect associated with oxytocin (Pitocin) administration during labor?

Tetanic contractions

Which of the following correctly defines puerperium?

The 6 weeks following birth

Which assessment finding indicates that the infant isn't latching on properly during breast-feeding.

The baby's lips smack.

A non reactive NST would indicate what about the fetus?

The fetal heart rate does not accelerate adequately with fetal movement. -A BPP or CST will need to be done now

When assessing the fetal heart rate tracing, the nurse assesses the fetal heart rate at 170 beats/minute. This rate is considered fetal tachycardia if which of the following occurs?

The fetal heart rate remains at greater than 160 beats/minute for 10 minutes.

The nurse is monitoring a client with dysfunctional labor for signs of fetal or maternal compromise. Which finding should alert the nurse to a compromise?

The passage of meconium

A nurse on the labor and delivery unit is caring for a client following a vaginal examination by the provider which is documented as: -1. Which of the following interpretations of this finding should the nurse make?

The presenting part is 1 cm above the ischial spines.

Subinvolution is when:

The uterus remains enlarged with continued lochial discharge and may result in PP hemorrhage

The nurse is collecting data on a neonate. Which finding indicates that the neonate's fontanels are normal?

They're soft to touch.

As part of the postpartum follow-up, the nurse calls a new mother at home a few days after discharge. The client answers the telephone, begins to cry, and tells the nurse that she has feelings of inadequacy and isn't coping with the demands of motherhood. Based on this information, which of the following assessments would the nurse make?

This is expected behavior for a client 3 to 7 days postpartum.

Which of the following interventions would be appropriate to include in the plan of care for a client during the fourth stage of labor?

Time with the baby to initiate breast-feeding

The nurse is caring for a client who's in the first stage of labor. What is the shortest but most difficult part of this stage?

Transitional phase

The nurse is caring for a client during the second stage of labor. On assessment, the nurse notes a slowing of the fetal heart rate and a loss of variability. What is the initial nursing action?

Turn the client on her side and administer oxygen by face mask at 8 to 10 L/min.

The nurse is caring for a client during the second stage of labor. On assessment, the nurse notes a slowing of the fetal heart rate and a loss of variability. Which is the initial nursing action?

Turn the client onto her side and give oxygen by face mask at 8 to 10 L/min.

Shortly after receiving epidural anesthesia, a laboring woman's blood pressure drops to 95/43 mm Hg. Which immediate actions should the nurse take? Select all that apply.

Turn the woman to a lateral position. Increase the rate of the intravenous infusion. Administer oxygen by face mask at 10 L/minute.

A nurse in labor and delivery is caring for a client. Following delivery of the placenta, the nurse examines the umbilical cord. Which of the following vessels should the nurse expect to observe in the umbilical cord?

Two arteries and one vein

A primigravid client is admitted to the labor and delivery area, where the nurse evaluates her. Which data collection finding may indicate the need for cesarean delivery?

Umbilical cord prolapse

Gonorrhea S&S

Urethral discharge Painful urination Frequency Yellowish-green vag discharge Reddended vulava & vag walls

Terbutaline (Brethine)

Uses to relax uterine smooth muscle to inhibit uterine activity

Oxytocin (Pitocin) is administered to a client following the delivery of the placenta. The nurse assisting in caring for the client monitors for an effective response from the medication by monitoring for:

Uterine contractions (Oxytocin stimulates uterine contractions and is administered to reduce the incidence of hemorrhage after expulsion of the placenta. It does not directly affect urinary output or milk production. The subsequent contraction of the uterus may cause an increase in the afterbirth pains. )

A pregnant client with a history of cardiac dysfunction has been taking propranolol (Inderal), a beta-adrenergic blocker, to treat hypertension. During labor, the nurse should stay alert for which adverse effect of this drug?

Uterine hypertonus

The nurse is monitoring a client in labor. The nurse suspects umbilical cord compression if which is noted on the external monitor tracing during a contraction?

Variable decelerations

Breast engorgement occurs on the second or third postpartum day. Which of the following processes causes engorgement?

Vasodilation, which causes the breast to feel full

What vitamin aids in the absorption of Iron?

Vitamin C

A client has received dinoprostone (Prostin E2) to help ripen her cervix. The nurse should monitor the client for which adverse effect?

Vomiting

Chlamydia s&s

Vulvar itching Vag spotting White, watery vaginal discharge

The nurse is preparing to perform a postpartum assessment on a client who gave birth 5 hours ago. Which precaution should the nurse plan to take for this procedure?

Washing the hands and wearing gloves

Which of the following should be the nurse's initial action immediately following the birth of the neonate?

Which of the following should be the nurse's initial action immediately following the birth of the neonate?

A client who has been in the latent phase of the first stage of labor enters the transition to the active phase. During the transition, the nurse expects to see which client behavior?

Withdrawal, irritability, and resistance to touch

A pregnant client admitted to the labor room arrived with a fetal heart rate (FHR) of 94 beats/minute and the umbilical cord protruding from the vagina. The client tells the nurse that her "water broke" before coming to the hospital. What is the appropriate nursing action?

Wrap the cord loosely in a sterile towel soaked with warm, sterile normal saline.

A nurse receives report about assigned clients at the start of the shift. Which of the following clients should the nurse plan to see first?

a client who experienced a cesarean birth 4 hr ago and reports pain

nurse is caring for several clients. the nurse should recognize that it is safe to administer tocolytic therapy to which client?

a client who is experiencing preterm labor at 26 wks

The nurse visits a client at home on the tenth postpartum day. When assessing the client's uterus, the nurse expects to find:

a nonpalpable fundus in the abdomen.

Where is the fundus initially felt after the placenta has been expelled?

at or below the level of the umbilicus

a nurse is caring for several clients. the nurse should recognize that it is safe to administer tocolytic therapy to which of the following clients? a. a client who is experiencing fetal death at 32 weeks of gestation b. a client who is experiencing preterm labor at 26 weeks of gestation c. a client who is experiencing braxton-hicks contractions at 36 weeks of gestation d. a client who has a post-term pregnancy at 42 weeks of gestation

b (Tocolytic medications, such as terbutaline, indomethacin, and nifedipine are used to relax the uterus in preterm labor. A client who is in preterm labor at 26 weeks of gestation is a candidate for tocolytic therapy.)

Mechanical methods to ripen cervix:

balloon caths hydroscopic dilators and sponges laminara tents synthetic dilators and sponges

A _______ layerof the lining remains to generate new endometrium to prepare for future pregnancies.

basal

Term birth

between 38 weeks and 42 weeks

What are the 2 most common disorders associated with postpartum psychosis?

bi polar disease major depression

The absence of lochia is not normal and may be associated with _____ _______ or infection.

blood clots

What happens to blood pressure after birth?

blood pressure falls when sits or stands. orthostatic hypotension

mongolian spots

bluish purple spots of pigmentation

a nurse is admitting a client who is at 30 weeks of gestation and is in preterm labor. the client has a new prescription for betamethasone and asks the nurse about the purpose of this medication. the nurse should provide which of the following explanations? a. "It is used to stop preterm labor contractions." b. "It halts cervical dilation." c. "It promotes fetal lung maturity." d. "It increases the fetal heart rate."

c

The REEDA acronym is also useful for assessing?

c-section incision

What is the focus for nursing care of the perinium?

comfort and hygeinic measures

a nurse is admitting a client who is at 33 weeks of gestation and has a diagnosis of placenta previa. which of the following is the priority nursing action? a. monitor vaginal bleeding b. administer glucocorticoids c. insert an IV catheter d. apply an external fetal monitor

d (Based on Maslow's hierarchy of needs, the nurse should immediately apply the fetal monitor to determine if the fetus is in distress.) (a-The quantity of vaginal bleeding and any associated pain should be monitored, but this is not the priority action by the nurse.) (b-Glucocorticoids should be administered to the client who is at less than 34 weeks of gestation to promote fetal lung maturity, but this is not the priority action by the nurse.) (c-IV access should be established, but this is not the priority action by the nurse.)

When pregnant, the uterine lining is called?

decidua

After birth, the body must rid itself of extra fluids. It does this in 2 ways?

diuresis diaphoresis

nurse is caring for client, 1hr PP, observes large amount of lochia rubra, several small clots on pad. fundus is midline and firm at the umbilicus. nurse action to take?

document findings and continue to monitor the client. expected findings.

When not pregnant, the uterine lining is called?

endometrium

crackles and wheezing are signs of

fluid or infection in the lungs

A nurse is caring for a client who is postpartum and received methylergonovine. Which of the following findings indicates that the medication was effective?

fundus firm to palpation

increased susceptibility to develop blood clots

hypercoagulability

nurse is caring for client, 1st stage of labor, observes umbilical cord protruding from the vagina. first action to perform?

insert gloved hand into vagina to relieve pressure on the cord

Low birth weight

less than 2500 grams

less than 10cm stain of lochia is?

light

Is it normal to have discharge from the perineal suture line?

no

APGAR of 7-10 indicates

no distres

When does kidney function return to normal after birth?

one month

What hormone is released when an infant nurses that helps contract the uterus?

oxytocin

How is the amount of lochia determined?

peripads are counted or weighed.

Where is oxytocin released from?

posterior pituitary

nurse is caring for a client who is at 18wk, client tells nurse she felt fluttering movements in her abdomen 3 days ago, interpret findings as?

quickening around 14th week

The nurse is caring for a client on her second postpartum day. The nurse should expect the client's lochia to be:

red and moderate.

What are the signs of thrombosis in the legs?

reddened, tender area edema pain pallor homans sign

nurse is assessing NB immediately following c-section delivery, following assessment priority??

respiratory distress

nurse in prenatal clinic, caring for client, using leopolds manuvers, palpates a round, firm, moveable part in the fundus of the tuerus and a long smoth surface on the clients right side. which quadrant should the nurse expect to auscultate FHT

right upper, breech presentation

nurse assisting w/ BF, explains which reflex will promote NB latch?

rooting

superficial venous thrombosis involves what vein of the lower leg, characterized by painful, hard, red, warm vein that is easily seen?

saphenous vein (SVT)

less that 5cm stain of lochia is?

scant

APGAR of 0-3 indicates

severe distress

nurse is caring for pt during 1st trimester, after reviewing pt blood work, nurse notices she does not have immunity to rubella, which times should the nurse understand is recommended for rubella immunization?

shortly after birth

When does the gastrointestinal system get back to normal after birth/

shortly after when progesterone decreases

Grunting and nasal flaring are:

signs of respiratory distress

What increases circulation to the perineum and promotes healing?

sitz bath

First degree laceration extends through:

skin of perineum

Second degree laceration extends through:

skin of perineum muscles of perineum

Third degree laceration extends through:

skin of perineum muscles of perineum anal sphincter

Fourth degree laceration extends through:

skin of perineum muscles of perineum anal sphincter anterior rectal wall

teaching fetal development to group of clients at antenatal clinic, nurse include in the teaching?

the baby's heart beat is audible by a doppler stethoscope at 12 weeks of preg sex is determined at conception lanugo covers fetal body at 20wks and disappears by week 36 quickening the feeling of fetal movement begins between weeks 14 to 16 in multiparous and week 18 or later in nulliparous

Postpartum the mother should not lift anything that is heavier than:

the infant

nurse is assessing a client in active labor, and notes the presenting part is 0 station. correct interpretation of clinical finding?

the lowermost portion of the fetus is at the level of the ischial spines

nurse is caring for client in labor, EFmonitor. observe late decels, on the strip and interprets this as?

uteroplacental insuffficiency

A blood clot in a vein

venous thrombosis

A postpartum client decides to bottle-feed her neonate. To prevent breast engorgement, the nurse should recommend that she:

wear a supportive, well-fitting brassiere.

nurse in family planning clinic, caring for 17y.o, request OC, client states she is nervous, never had a pelvic exam, nurse response?

what part of the exam makes you most nervous?

The parents of a male neonate who is not circumcised request information on how to clean the newborn's penis. Which of the following is the correct response for the nurse to make to the parents?

"Avoid retracting the foreskin to cleanse the glans because this may cause adhesions." (In newborn males, the prepuce is continuous with the epidermis of the glans and is nonretractable. Forced retraction may cause adhesions to develop. Separation should be allowed to occur naturally, which will take place between 3 years and 5 years of age. Most foreskins are retractable by 3 years of age and should be pushed back gently for cleaning once a week.)

A pregnant client tests positive for the hepatitis B virus (HBV), and the client asks the nurse whether she will be able to breast-feed the baby as planned after delivery. The nurse makes which response to the client?

"Breast-feeding is allowed once the baby has been vaccinated." (Although HBV is transmitted in breast milk, once the first dose of hepatitis B vaccine and the serum immune globulin have been administered to the newborn, the woman may breast-feed without risk to the newborn. Options 2, 3, and 4 are incorrect responses.)

The nurse is providing teaching to a client who's being discharged after delivering a hydatidiform mole. Which expected outcome takes highest priority for this client?

"Client will use a reliable contraceptive method until her follow-up care is complete in 1 year and her hCG level is negative."

After a vaginal delivery, a postpartum client complains of perineal discomfort when sitting. To promote comfort, the nurse should provide which instruction?

"Contract your buttocks before sitting or rising."

A 24-year-old multigravida client who had an uncomplicated, spontaneous vaginal delivery 7 hours ago is uninterested in her baby and wants to sleep. The student nurse assigned to care for the client is concerned and tells the licensed practical nurse (LPN) who's also assigned to her care. Which response by the LPN is most effective in educating the student nurse?

"Extreme fatigue from the delivery is common, and new mothers initially focus on recovery and taking in the birth experience."

On the second postpartum day, a woman complains of burning on urination, urgency, and frequency of urination. A urine sample is collected for urinalysis, and the results indicate the presence of a urinary tract infection. The nurse instructs the new mother regarding measures to take for the treatment of the infection. Which statement by the mother indicates the need for further instructions?

"Foods and fluids that will increase urine alkalinity should be consumed." (The woman with a urinary tract infection must be encouraged to take the medication for the entire time it is prescribed. The woman also should be instructed to drink at least 3000 mL of fluid each day to flush the infection from the bladder and to urinate frequently throughout the day. Foods and fluids that acidify the urine need to be encouraged.)

A nurse is reinforcing instructions to a postpartum cesarean delivery client who is preparing for discharge. Which statement by the client indicates a need for more information?

"I can start doing abdominal exercises as soon as I get home." (Abdominal exercises should not start following abdominal surgery until 3 to 4 weeks postoperatively to allow for healing of the incision. )

A postpartum client is getting ready for discharge. The nurse suspects that the client is in need of further teaching related to breast-feeding when she states:

"I don't need birth control since I will be breast-feeding." (Amenorrhea may occur during breast-feeding, but the client can still ovulate without menstruating. The use of soap on the breasts is avoided because it tends to remove natural oils, which can lead to cracked nipples. The caloric intake should be increased by 200 to 500 cal/day (per health care provider's prescription), and the diet should include additional fluids and prenatal vitamins, as prescribed.)

A nurse is caring for a client who delivered a healthy newborn via vaginal delivery. An episiotomy was performed, and the woman has developed a wound infection at the episiotomy site. The nurse provides instructions to the mother regarding care related to the infection. Which statement by the mother indicates the need for further instructions?

"I need to isolate my infant for 48 hours after starting the antibiotics." (Broad-spectrum antibiotics will be prescribed for the mother, and she should be instructed to take them as prescribed. Analgesics often are necessary, and warm compresses or sitz baths may be used to provide comfort. The infant is not routinely isolated from the mother with a wound infection, but the mother must be taught how to protect the infant from contact with contaminated articles.)

A nurse is caring for a client who delivered a healthy newborn via vaginal delivery. An episiotomy was performed, and the woman has developed a wound infection at the episiotomy site. The nurse provides instructions to the mother regarding care related to the infection. Which statement by the mother indicates a need for further instruction?

"I need to isolate my infant for 48 hours after the starting the antibiotics." (Broad-spectrum antibiotics will be prescribed for the mother, and the mother should be instructed to take the antibiotics as prescribed. Analgesics are often necessary, and warm compresses or sitz baths may be used to provide comfort in the area. The infant is not routinely isolated from the mother with a wound infection, but the mother must be taught how to protect the infant from contact with contaminated articles.)

A nurse is providing teaching about comfort measures for breast engorgement to a client who is postpartum and is breastfeeding.. Which of the following statements by the client indicates a need for further teaching?

"I should apply hot packs to my breasts during feeding."

A nurse is providing instructions to a client who had an episiotomy during the birthing process. Which statement by the client would indicate a need for further instructions?

"I should take sitz baths three or four times a day and test the water temperature to be sure that it is at 115° F." (Following episiotomy, the client should be instructed in measures to decrease discomfort and perineal swelling. Gluteal muscle tightening reduces direct pressure on the perineum, so discomfort is minimized. Ice decreases circulation, promotes vasoconstriction, reduces edema, and promotes a local anesthetic effect. Local anesthetic sprays reduce discomfort. Heat from sitz baths increases circulation to the perineum, thereby promoting oxygenation and healing, which reduces discomfort. However, the water temperature should not be any greater than 100° F to 105° F.)

A nurse is providing instructions to the mother following delivery regarding care of the episiotomy site to prevent infection. Which statement by the mother indicates a need for further instructions?

"I will change the perineum pads three times a day." (Warm sitz baths and cleansing with warm water are helpful for relieving pain, and these measures will promote cleanliness in the perineum area to prevent infection. The mother also should be instructed to wipe the perineum from front to back after voiding and defecation to decrease the risk for contamination with microorganisms from the anus to the vagina. Warm water should be used to rinse the perineum after elimination. The mother also should be instructed that the perineal pad should be changed after each elimination and may be changed in between.)

A postpartum client who delivered at 32 weeks of gestation would like to breast-feed her preterm infant. At this time, the infant is receiving tube feedings only. What is the nurse's best response to the mother?

"You can begin pumping as soon as possible after delivery with an electric breast pump." (Prematurity usually causes a delay before the baby can be fed at the breast. Mothers must initiate and maintain their milk supply with an electric breast pump. Milk expression by electric pump needs to begin as soon as possible after delivery and be done eight or more times each 24 hours. Hand expression is not as effective as using an electric pump.)

A nurse is caring for a client who is postpartum and is breastfeeding. The client states that she is concerned about dietary precautions since she has a family history of food allergies. The nurse offers which of the following responses?

"You might want to avoid eating peanuts"

A postpartum client with gestational diabetes is scheduled for discharge. During the discharge, the client asks the nurse, "Do I have to worry about this diabetes anymore?" The nurse makes which response to the client?

"You will be at risk for developing gestational diabetes with your next pregnancy and developing overt diabetes mellitus." (The client is at risk for developing gestational diabetes with each pregnancy. She also has an increased risk of developing overt diabetes and needs to comply with follow-up appointments. She needs to be taught techniques to lower her risk for developing diabetes, such as weight control. The diagnosis of gestational diabetes indicates that this client has an increased risk for developing overt diabetes; however, with proper care it may not develop.)

A nurse is caring for a client who is at 6 weeks of gestation with her first pregnancy and asks the nurse when she can expect to experience quickening. Which of the following responses should the nurse make? A. "This will occur during the last trimester of pregnancy." B. "This will happen by the end of the first trimester of pregnancy." C. "This will occur between the fourth and fifth months of pregnancy." D. "This will happen once the uterus begins to rise out of the pelvis."

C. "This will occur between the fourth and fifth months of pregnancy." i: Quickening is defined as the first time the client is able to feel her fetus move. In a primigravida client, this usually occurs at 18 weeks of gestation or later. In a multigravida client, this can occur as early as 14 to 16 weeks.

(SELECT ALL THAT APPLY) The nurse is caring for a client who has been diagnosed with abruptio placenta. What signs and symptoms of abruptio placenta should the nurse expect to find when she is collecting data on this client?

(1) Vaginal bleeding, (3) Uterine tenderness on palpation, (6) Abnormal fetal heart tones

(SELECT ALL THAT APPLY) On examining a client who gave birth 3 hours ago, the nurse finds that the client has completely saturated a perineal pad within 15 minutes. Which of the following actions should the nurse take?

(2) Assess the client's vital signs., (3) Palpate the client's fundus.

(SELECT ALL THAT APPLY) The nurse is instructing the client on breast-feeding. Which instructions should she include to help the mother prevent mastitis?

(2) Change the breast pads frequently., (3) Expose your nipples to air part of each day., (4) Wash your hands before handling your breast and breast-feeding., (6) Release the baby's grasp on the nipple before removing him from the breast.

(SELECT ALL THAT APPLY) A client in labor is given meperidine (Demerol), 25 mg I.V., for labor pain. The nurse should monitor the client for which adverse effects of the drug?

(2) Nausea & Vomiting, (3) Respiratory Depression, (5) Tachycardia

(SELECT ALL THAT APPLY) The nurse is assisting in developing a care plan for a client with an episiotomy. Which interventions would be included for the nursing diagnosis Acute pain related to perineal sutures?

(3) Administer sitz baths three to four times per day., (4) Encourage the client to do Kegel exercises.

(SELECT ALL THAT APPLY) The nurse is assisting in caring for a client who has just given birth to a neonate through vaginal delivery. The nurse is monitoring for signs of placental separation. Which of the following signs indicate that the placenta has separated?

(3) Sudden gush of vaginal blood, (4) Change in shape of the uterus, (5) Lengthening of the umbilical cord

A nurse in a prenatal clinic is caring for a client who is pregnant and asks the nurse for her estimated date of birth (EDB). The client's last menstrual period began on July 27. What is the client's EDB?

0504 - May 4th i: Using Nägele's rule, the nurse subtracts three months from the date of the last menstrual period, then adds 7 days. July minus 3 months equals April. There are 30 days in April, so 27 + 7 = May 4. The client's EDB is May 4, which would be written as 0504 in the MMDD format.

APGAR Color scoring

0= Blue, pale 1= pink body, cyanotic hands and feet (acrocyanosis) 2= Completely pink

APGAR Muscle tone scoring

0= Flaccid 1= some flexion 2= well-flexed

APGAR Heart rate scoring

0= absent 1= <100 2= >100

APGAR RR scoring

0= absent 1= slow, weak cry 2= good cry

APGAR Reflex irritability

0= none 2= grimace 3= cry

how much does the fundus descend each day after birth?

1 cm

When do the breasts of a non nursing mother return to their normal size?

1 to 2 weeks

First stage of labor:

1-1.5 cm (onset of labor)

The nurse is assessing the deep tendon reflexes of a client with severe preeclampsia who is receiving intravenous magnesium sulfate. The nurse should perform which procedure to assess the brachioradialis reflex? Click on the image to indicate your answer.

1.

Which client care assignment is the most appropriate assignment for a newly graduated licensed practical nurse (LPN)?

A 24-year-old primigravida who delivered a 6-lb, 4-oz (2,835-g) baby vaginally 4 hours ago and is unable to void

A client in labor is receiving oxytocin by intravenous infusion to stimulate uterine contractions. Which finding indicates that the rate of infusion needs to be decreased?

A fetal heart rate of 180 beats/min

The nurse is caring for a client in labor who is receiving oxytocin by intravenous infusion to stimulate uterine contractions. Which assessment finding should indicate to the nurse that the infusion needs to be discontinued?

A fetal heart rate of 90 beats/minute

The nurse in a maternity unit is reviewing the clients' records. Which clients should the nurse identify as being at the most risk for developing disseminated intravascular coagulation (DIC)? Select all that apply.

A gravida II who has just been diagnosed with dead fetus syndrome A primigravida at 29 weeks of gestation who was recently diagnosed with severe preeclampsia

A nurse is monitoring a new mother in the fourth stage of labor for signs of hemorrhage. Which of the following signs, if noted in the mother, would indicate an early sign of excessive blood loss and shock?

A increase in the pulse rate from 88 to 102 beats per minute (During the fourth stage of labor, the maternal blood pressure, pulse, and respiration should be checked every 15 minutes during the first hour. A rising pulse is an early sign of excessive blood loss because the heart pumps faster to compensate for reduced blood volume. The blood pressure will fall as the blood volume diminishes, but a decreased blood pressure would not be the earliest sign of hemorrhage. An elevation in temperature is not a sign of excessive blood loss. Although the respiratory rate may increase, this would not be an early sign. In addition, an increase in the respiratory rate from 18 to 22 breaths per minute is not significant.)

A nurse is preparing to teach a new mother to breast-feed. Which factor is important to promote an effective and positive learning experience?

A positive nurse-client relationship (Because hospital stays are short, all contacts with the mother become teachable moments. A positive nurse-client relationship is a growth-fostering experience that will enhance the teaching and learning experience. Separation of the infant and mother decreases the chance of correct latch and suck in the immediate postpartum period. The infant should be placed at the breast immediately after delivery. The mother, not the health care provider, makes the decision regarding the method of feeding. Although previous breast-feeding experience is helpful, the most significant factor is the nurse-client relationship.)

A nurse is assisting in developing a plan of care for a client preparing to breast-feed. In planning care, which factor is significant in teaching a client to breast-feed?

A positive nurse-client relationship (The nurse-client relationship is most significant. Option 1 is the opposite of what needs to happen. Brief separation decreases the chance of correct latch and suck in the immediate postpartum period. Infants should be placed at the breast immediately after delivery. Previous breast-feeding experience and a health care provider who encourages clients to breast-feed are not the significant factors.)

Regarding BISHOP scoring, when is a woman ready for labor?

A score of 9 for nulliparas 5 or more for multiparas

When caring for a client who has recently delivered, the nurse assesses the client for urinary retention with overflow. Which of the following descriptions provides an accurate picture of retention with overflow?

A varying urge to urinate with an average output of 100 ml

A nurse in a clinic is reviewing the medical records of a group of clients who are pregnant. The nurse should anticipate the provider will order a maternal serum alpha-fetoprotein (MSAFP) screening for which of the following clients? A. A client who has mitral valve prolapse B. A client who has been exposed to AIDS C. All of the clients D. A client who has a history of preterm labor.

C. All of the clients i: MSAFP is a screening tool to detect open spinal and abdominal wall defects in the fetus. This maternal blood test is recommended for all pregnant woman.

A nurse admits a woman who is at 38 weeks of gestation and in early labor with ruptured membranes. The nurse determines that the client's oral temperature is 38.9C/102F. Besides notifying the provider, which of the following is the appropriate nursing action? A. Recheck the client's temperature in 4 hr. B. Administer glucocorticoids intramuscularly. C. Assess the odor of the amniotic fluid. D. Prepare the client for emergency cesarean section.

C. Assess the odor of the amniotic fluid. i: Chorioamnionitis is an infection of the amniotic cavity that presents with maternal fever, tachycardia, increased uterine tenderness, and foul-smelling amniotic fluid.

A nurse is performing Leopold maneuvers on a client who is in labor and determines the fetus is in an RSA position. Which of the following fetal presentations should the nurse document in the client's medical record? A. Vertex B. Shoulder C. Breech D. Mentum

C. Breech i: An RSA position indicates that the body part of the fetus that is closest to the cervix is the sacrum. Therefore, the buttocks or feet are the presenting part, which is classified as a breech presentation.

A nurse is caring for a client who is to undergo a biophysical profile. The client asks the nurse what is being evaluated during this test. Which of the following should the nurse include? (SATA) A. Fetal breathing B. Fetal motion C. Fetal neck translucency D. Amniotic fluid volume E. Fetal gender

A, B, D i: A biophysical profile is an assessment of fetal well-being and includes ultrasound evaluation of fetal breathing movements, gross fetal movements, and amniotic fluid volume. A biophysical profile is an assessment of fetal well-being and includes ultrasound evaluation of fetal breathing movements, gross fetal movements, and amniotic fluid volume. A biophysical profile is an assessment of fetal well-being and includes ultrasound evaluation of fetal breathing movements, gross fetal movements, and amniotic fluid volume.

A nurse in a prenatal clinic is reviewing the health record of a client who is at 28 weeks of gestation. The history includes one pregnancy, terminated by elective abortion at 9 weeks; the birth of twins at 36 weeks; and a spontaneous abortion at 15 weeks. According to the GTPAL system, which of the following describes the client's current status? A. 4-0-1-2-2 B. 3-0-2-0-2 C. 2-0-0-2-0 D. 4-2-0-2-2

A. 4-0-1-2-2 i: This response correctly describes the client's current status: pregnant currently and had 3 prior pregnancies (G); no term births (T); one pregnancy resulted in the preterm birth (P) of twins; two pregnancies ended in abortion (A); and she has two living children (L).

A nurse is assigned to care for a client in the immediate postpartum period who received epidural anesthesia for delivery, and the nurse monitors the client for complications. Which of the following would most likely indicate a hematoma?

Changes in vital signs (Changes in vital signs indicate hypovolemia in the anesthetized postpartum woman with vulvar hematoma. Options 3 and 4 are inaccurate for a client who is anesthetized. Heavy bruising may be noted, but vital sign changes are most likely to indicate the presence of a hematoma.)

A nurse is caring for a client during the first trimester of pregnancy. After reviewing the client's blood work, the nurse notices she does not have immunity to rubella. Which of the following times should the nurse understand is recommended for rubella immunization? A. Shortly after giving birth B. In the third trimester C. Immediately D. During her next attempt to get pregnant

A. Shortly after giving birth i: The rubella immunization should be offered to the client following birth, preferably prior to discharge from the hospital. This prevents the client from contracting rubella during the current or subsequent pregnancies, which would put her fetus at risk for rubella syndrome.

A nurse is caring for a 3-hour-old infant and notes that the infant has not eaten since birth, is jittery, and has a weak cry. The mother states that she can't get the baby to eat. What action should the nurse take first?

Check the blood glucose level. (This infant has classic symptoms of hypoglycemia. The nurse should plan to check the infant's blood glucose to determine the extent of hypoglycemia, if any, and then to take action by calling the health care provider and feeding the infant as per agency policy. Allowing the infant to sleep may cause the hypoglycemia to remain untreated and result in neurological damage)

A nurse is caring for a client during the immediate recovery phase or fourth stage of labor. The nurse's important action at this time is to:

Check the uterine fundus and lochia. (A potential complication following delivery is hemorrhage. The most significant source of bleeding is the site where the placenta is implanted. It is critical that the uterus remain contracted, and vaginal blood flow is monitored every 15 minutes for the first 1 to 2 hours. )

A nurse is collecting data on a postpartum client and performs which best intervention when checking for thrombophlebitis in the legs?

Checks the calf areas for redness or swelling (Redness, swelling, and pain in the calf area are signs of thrombophlebitis, a potential complication in the postpartum period. Options 1 and 4 do not determine the presence of thrombophlebitis. Although the client with thrombophlebitis may experience pain when ambulating, option 3 is not the best intervention from those provided in the options.)

Pulmonary Embolus S&S

Chills Apprehension Pleruitic Chest pain Dyspnea tachypnea hemoptysis heart murmurs peripheral edema distended neck veins elevated temp hypotension hypoxia

The nurse is assisting in the care of a client in labor who is having an amniotomy performed. The nurse should report which abnormal findings to the health care provider (HCP)? Select all that apply.

Clear, dark amber amniotic fluid Light green amniotic fluid with no odor Thick white amniotic fluid with no odor

A clinical pathway is guiding care for a postpartum client who had an uncomplicated vaginal delivery of an 8-lb, 2-oz (3,686-g) baby 24 hours ago. The client has no episiotomy and is bottle-feeding her baby. Which outcome should be achieved within the next 8 hours?

Client will demonstrate ability to bottle-feed the neonate.

A nurse on the labor and delivery unit is caring for a client following a vaginal examination by the provider which is documented as: -1. Which of the following interpretations of this finding should the nurse make? A. The presenting part is 1cm above the ischial spines. B. The presenting art is 1cm below the ischial spines. C. The cervix is 1cm dilated. D. The cervix is effaced 1cm.

A. The presenting part is 1cm above the ischial spines. i: Station is the relation of the presenting part to the ischial spines of the maternal pelvis and is measured in centimeters above, below, or at the level of the spines. If the station is minus 1, then the presenting part is 1cm above the ischial spines.

The nurse in the labor room is caring for a client in the active stage of the first phase of labor. The nurse is assessing the fetal patterns and notes a late deceleration on the monitor strip. What is the most appropriate nursing action?

Administer oxygen via face mask.

One day after having a cesarean birth, a client complains of incisional pain that she rates as a 3 on a 1-to-10 scale, with 10 representing the most severe pain. The physician prescribed ibuprofen (Motrin), 400 mg by mouth every 4 to 6 hours, as needed. Which intervention should the nurse take when administering this drug?

Administer the drug with meals or milk.

The nurse is caring for a client in the active stage of labor. The nurse notes that the fetal pattern shows a late deceleration on the monitor strip. Based on this finding, the nurse should prepare for which appropriate nursing action?

Administering oxygen via face mask

The client has just given birth to her first child, a healthy, full-term baby girl. The client is Rho(D)-negative and her baby is Rh-positive. What intervention will be performed to reduce the risk of Rh incompatibility?

Administration of Rho(D) immune globulin I.M. to the mother within 72 hours

A nurse in the postpartum unit notes that the result of a rubella titer drawn on a postpartum client during the antepartum period is 1:8. Which of the following would the nurse anticipate to be prescribed by the health care provider?

Administration of a subcutaneous rubella virus vaccine (A blood sample for rubella titer is done on all women in the antepartum or postpartum period. A postpartum woman with a titer of 1:8 or less should receive a subcutaneous rubella virus vaccine (Meruvax II). This stimulates active immunity against the rubella virus. The woman should be counseled to avoid pregnancy for 3 months after receiving the vaccine.)

The nurse is monitoring a client who is in the active stage of labor. The nurse documents that the client is experiencing labor dystocia. The nurse determines that which risk factors in the client's history placed her at risk for this complication? Select all that apply.

Age 54 Body mass index of 28 Previous difficulty with fertility

A client had a cesarean delivery with a low transverse uterine incision. The nurse explains the benefits of this type of incision to the client, knowing that this type of incision:

Allows a vaginal birth after cesarean (VBAC) to be possible in a subsequent pregnancy (A low transverse uterine incision is unlikely to rupture during a subsequent labor and is the only type of uterine incision considered safe for a subsequent VBAC delivery. It cannot be extended laterally because of the location of the major uterine blood vessels in the lower uterine segment. In the presence of a placenta previa, a classic incision into the body of the uterus would be needed to prevent incising into the placental area. A suprapubic skin incision can be made with a lower uterine transverse incision.)

A nurse in a postpartum unit identifies which client as being at risk for developing endometritis following delivery?

An adolescent experiencing an emergency cesarean delivery for fetal distress (Endometritis is an acute infection of the mucous lining of the uterus that occurs immediately after delivery. Cesarean delivery is the primary risk factor for uterine infection, especially after emergency procedures. Other risk factors include prolonged rupture of membranes, multiple vaginal examinations, and an excessive length of labor. Options 1, 2, and 3 do not describe the client "at risk" to develop endometritis following delivery.)

Late in the first stage of labor, a client receives a spinal block to relieve discomfort. A short time later, her husband tells the nurse that his wife feels dizzy and is complaining of numbness around her lips. What do the client's symptoms suggest?

Anesthesia overdose

The nurse brings a new mother her baby for the first time approximately 1 hour after the baby's birth. After checking the identification, the nurse hands the baby to the mother. Within a few minutes, the mother begins to undress her baby. Which of the following should the nurse do?

Anticipate and support the behavior as a normal part of bonding.

A client who delivered by cesarean birth 3 days ago is bottle-feeding her neonate. While the nurse collects data, the client complains that her breasts are painful, hard, and warm to the touch. How should the nurse intervene?

Apply an elastic bandage to bind the breasts.

The nurse is caring for a client during the first postpartum day. The client asks the nurse how to relieve pain from her episiotomy. What should the nurse instruct the woman to do?

Apply an ice pack to her perineum.

A nurse is caring for a client who reports unrelieved episiotomy pain 8 hr following a vaginal birth. Which of the following actions should the nurse take?

Apply an ice pack to the affected area

A nurse is caring for a client who is 2 days postpartum, is breastfeeding, and reports nipple soreness. Which of the following measures should the nurse suggest to reduce discomfort during breastfeeding? (Select all that apply.)

Apply breast milk to the nipples before each feeding Start breastfeeding with the nipple that is less sore Change the infant's position on the nipples

A nurse is caring for a client who is 2 days postpartum, is breastfeeding, and reports nipple soreness. Which of the following measures should the nurse suggest to reduce discomfort during breastfeeding?

Apply breast milk to the nipples before each feeding Start breastfeeding with the nipple that is less sore Change the infant's position on the nipples

A woman in labor shouts to the nurse, "My baby is coming right now! I feel like I have to push!" An immediate nursing assessment reveals that the head of the fetus is crowning. After asking another staff member to notify the physician and setting up for delivery, which nursing intervention is most appropriate?

Applying gentle pressure to the baby's head as it's delivered

The nurse is planning to discharge a 24-year-old gravida 1, para 1, non-English-speaking Hispanic client. The client's English-speaking cousin is acting as a translator for the nurse and client. Which nursing intervention takes priority?

Arranging for a home care nurse to assess the client in her home environment

A nurse is assisting in caring for a newborn whose mother is Rh negative. In planning the newborn's care, it would be important for the nurse to:

Ask about the newborn's blood type and direct Coombs'. (To further assess and plan for the newborn's care, the newborn's blood type and direct Coombs' must be known. If the newborn's blood type is Rh negative, or if the newborn's blood type is Rh positive with a negative direct Coombs' test, then there is no concern for Rh incompatibility. If the newborn's blood type is Rh positive and the direct Coombs' is positive, then Rh incompatibility exists. Options 1 and 2 are inappropriate at this time because additional data are needed. Option 4 is incorrect because vitamin K is given to prevent hemorrhagic disease in the newborn.)

A nurse in a clinic is caring for a client who is 3 weeks postpartum following the birth of a healthy newborn. The client reports feeling "down" and sad, having no energy, and wanting to cry. Which of the following is a priority action by the nurse?

Ask the client if she has considered harming the newborn

A nurse is preparing to assist in performing a fundal assessment on a postpartum client. The nurse understands that the initial nursing action when performing this assessment is which of the following?

Ask the client to urinate and empty her bladder. (Before fundal assessment is started, the nurse should ask the mother to empty her bladder so that an accurate assessment can be done. The nurse can then assess the bladder for complete emptying and accurately assess uterine involution. When fundal assessment is performed, the woman is asked to lie flat on her back with the knees flexed. Massaging the fundus is not appropriate unless the fundus is boggy or soft, in which case it should be massaged gently until firm.)

A nurse is performing a postpartum fundal assessment on a client 6 hours after delivery. The nurse finds the fundus above the umbilicus and displaced to the right. Which intervention should the nurse do first?

Assist the client to the bathroom to void and then reassess the fundus. (A full bladder causes the uterus to be displaced above the umbilicus and well to one side of the abdominal midline. After voiding, if the fundus is boggy, it can be massaged, but this is not the first action. The woman should be assisted to the bathroom to void and then the fundus reassessed. Turning the client to her left side will not bring the fundus to midline. This is not a normal finding; the fundus should be firm at the umbilicus or 1 fingerbreadth below the umbilicus 6 hours after delivery.)

A client experiencing hard, steady labor is crying and asking for something for pain. Her medical record shows that she doesn't tolerate typically prescribed pain medications. Which nonpharmacologic intervention might be helpful to this client?

Assisting the client into another position and providing a gentle backrub

When caring for a client during the second stage of labor, which action would be most appropriate?

Assisting the mother with pushing

A delivery room nurse performs an assessment on a mother who just delivered a healthy newborn infant. The nurse checks the uterine fundus, expecting to note that it is positioned:

At the level of the umbilicus (Immediately after delivery, the uterine fundus should be at the level of the umbilicus or one to three fingerbreadths below it and in the midline of the abdomen. If the fundus is more than 1 cc above the umbilicus, this may indicate that there are blood clots in the uterus that need to be expelled by fundal massage. A fundus that is not located in the midline may indicate a full bladder.)

After delivery, the nurse checks the height of the uterine fundus. The nurse expects that the position of the fundus would most likely be noted:

At the level of the umbilicus (After delivery, the uterine fundus should be at the level of the umbilicus or one to three fingerbreadths below it and in the midline of the abdomen. If the fundus is 4 cm above the umbilicus, this may indicate that there are blood clots in the uterus that need to be expelled by fundal massage. If the fundus is noted to the right of the abdomen, it may indicate a full bladder. By about 10 days postpartum, the uterus will be in the symphysis pubis area.)

A nurse in a prenatal clinic overhears a newly licensed nurse discussing conception with a client. Which of the following statements by the newly licensed nurse requires intervention by the nurse? A. "Fertilization takes place in the outer third of the fallopian tube." B. "Implantation occurs between 2 to 3 weeks after conception." C. "Sperm remain viable in the woman's reproductive tract for 2 to 3 days." D. "Bleeding or spotting can accompany implantation."

B. "Implantation occurs between 2 to 3 weeks after conception." i: This statement requires clarification because implantation occurs between 6 to 10 days following conception.

A nurse is teaching a group of clients who are in their first trimester about exercising during pregnancy. Which of the following statements should the nurse include in the teaching? A. "Refrain from exercises that include stretching." B. "Moderate exercise improves circulation." C. "It is recommended to increase your weight-bearing exercises." D. "It is recommended to rest for 30 minutes before each new exercise."

B. "Moderate exercise improves circulation." i: Improving circulation is just one of the many benefits of moderate exercise during pregnancy. It enhances well-being, promotes rest and relaxation, and improves muscle tone.

A nurse is caring for a client at the first prenatal visit who has a BMI of 26.5. The client asks how much weight she should gain during pregnancy. Which of the following responses should the nurse make? A. "It would be best if you gained about 11 to 20 pounds." B. "The recommendation for you is about 15 to 25 pounds." C. "A gain of about 25 to 35 pounds is recommended for you." D. "A gain of about 1 pound per week is the best pattern for you."

B. "The recommendation for you is about 15 to 25 pounds." i: Clients who are overweight, having a BMI of 25 to 29.9, should be advised that the recommended weight gain is 7 to 11.5 kg (15 to 25 lb). The pattern of weight gain is also important, with minimal gain in the first trimester.

A nurse is providing teaching about Kegel exercises to a group of clients who are in the third trimester of pregnancy. Which of the following statements by a client indicates understanding of the teaching? A. "These exercises help prevent constipation." B. "These exercises help pelvic muscles to stretch during birth." C. "They can help reduce back aches." D. "They can prevent further stretch marks."

B. "These exercises help pelvic muscles to stretch during birth." i: Kegel exercises improve the strength of perineal muscles, facilitating stretching and contracting during childbirth.

A nurse is creating the plan of care for a client who is at 39 weeks of gestation and in active labor. Which of the following actions should the nurse include in the plan of care? A. Keep four side rails up while the client is in bed. B. Check the cervix prior to analgesic administration. C. Monitor the fetal heart rate (FHR) every hour. D. Insert an indwelling urinary catheter.

B. Check the cervix prior to analgesic administration. i: Prior to administering an analgesic during active labor, the nurse must know how many centimeters the cervix has dilated. Administration too close to the time of delivery could cause respiratory depression in the newborn.

A nurse is admitting a client who is at 38 weeks of gestation and is in the first stage of labor. Which of the following assessment findings should the nurse report to the provider first? A. Expulsion of a blood-tinged mucous plug B. Continuous contraction lasting 2 min C. Pressure on the perineum causing the client to bear down D. Expulsion of clear fluid from the vagina

B. Continuous contraction lasting 2 min i: A uterus contracting for more than 90 seconds is a sign of tetany and could lead to uterine rupture, which is the greatest risk to the client at this time. The nurse should report this finding immediately.

A postpartum client asks the nurse when she may resume sexual activity. The nurse tells the client that sexual activity may:

Be resumed in about 3 weeks when the episiotomy has healed and the lochia has stopped (It is recommended that the woman refrain from sexual intercourse until the episiotomy has healed and the lochia has stopped. This process usually takes about 3 weeks)

A client in labor is receiving oxytocin (Pitocin). During oxytocin therapy, why must the nurse monitor the client's fluid intake and output closely?

Because oxytocin causes water intoxication

A new mother attempting breast-feeding for the first time has developed mastitis. She states, "My breasts look terrible and I think that I will stop breast-feeding." The nurse plans care, knowing that the client's statement relates to:

Body image (Inflammation and engorgement are symptoms of mastitis that may alter the new breast-feeding mother's body image. The client's statement does not relate to a problem with newborn nutrition, or inadequacy. Although mastitis is the result of a break in the skin at the nipple resulting in infection and leading to the need for antibiotics, the question is psychosocial and not physiological.)

A client is resting comfortably 4 hours after delivering her first child. When measuring her heart rate, the nurse expects which normal finding?

Bradycardia

Assessment of a client in active labor reveals meconium-stained amniotic fluid and fetal heart sounds in the upper right quadrant. Which of the following is the most likely cause of this situation?

Breech position

The nurse is performing an assessment on a client diagnosed with placenta previa. Which assessment findings should the nurse expect to note? Select all that apply.

Bright red vaginal bleeding Soft, relaxed, nontender uterus Fundal height may be greater than expected for gestational age

A client has progressed through the first stage of labor. Which data collection finding suggests she's in the transition to the second stage?

Bulging of the vaginal introitus

The nurse is preparing to administer an analgesic to a client in labor. Which analgesic is contraindicated for a client who has a history of opioid dependency?

Butorphanol tartrate

A client is admitted to the labor and delivery area. How can the nurse most effectively determine the duration of the client's contractions?

By timing the period from the onset of uterine tightening to uterine relaxation

A nurse in a prenatal clinic is completing a skin assessment of a client who is in the second trimester. Which of the following findings should the nurse expect? (SATA) A. Eczema B. Psoriasis C. Linea nigra D. Chloasma E. Striae gravidarum

C, D, E i: Linea nigra manifests as a line of pigmentation extending from the symphysis pubis to the top of the fundus and is an expected finding during pregnancy. Chloasma, or the mask of pregnancy, manifests as blotchy, brownish hyperpigmentation of the skin over the forehead, nose, and cheeks and is an expected finding during pregnancy. Striae gravidarum, or stretch marks, occur because of the separation of underlying connective tissue on the breasts, thighs, and abdomen. They are an expected finding during pregnancy.

A nurse in a prenatal clinic is instructing a client about an amniocentesis, which is scheduled at 15weeks of gestation. Which of the following should be included in the teaching? A. "The test will be performed if your baby's heart beat is heard." B. "This test will determine if your baby's lungs are mature." C. "This test requires the presence of amniotic fluid." D. "After the test, you will be given Rh0 immune globulin since you are Rh positive."

C. "This test requires the presence of amniotic fluid." i: Amniocentesis requires adequate amniotic fluid for testing, which is not available until after 14 weeks of gestation.

A nurse provides instructions to a breast-feeding mother who is experiencing breast engorgement about measures that will provide comfort. Which statement by the mother indicates an understanding of these measures?

"I will massage the breasts before feeding to stimulate let-down." (Comfort measures for breast engorgement include massaging the breasts before feeding to stimulate let-down, wearing a supportive well-fitting bra at all times, taking a warm shower just before feeding or applying warm compresses, and alternating the breasts during feeding. Options 1, 2, and 4 are incorrect measures.)

A nurse is caring for a client who is postpartum. The nurse should recognize which of the following statements by the client as an indication of inhibition of parental attachment?

"I wish he had more hair. I will keep a hat on his head until he grows some."

A home health nurse is teaching a client who is breastfeeding about managing breast engorgement. Which of the following client statement indicates understanding of the teaching?

"I'll feed by baby q2h"

The nurse demonstrates bathing of a neonate to a primiparous client. Which statement by the client indicates understanding?

"I'm going to bathe the baby in the kitchen because it's nice and warm there."

A nurse provides home care instructions to a postpartum client who had a cesarean delivery. Which statement by the client indicates an understanding of the instructions?

"If I develop a fever, I will call my doctor." (The client should not lift anything heavier than the baby for 2 weeks. When getting out of bed, the client should turn on the side and push up with the arms. The client should call the doctor if a fever develops. Abdominal exercises should not be started following abdominal surgery until 3 to 4 weeks postoperatively to allow for healing of the incision.)

The nurse is teaching a breast-feeding client how to care for her engorged breasts. Which statement by the client indicates the need for further teaching?

"If my breasts are uncomfortable, I'll limit the time I spend breast-feeding."

A nurse is caring for a client who is 6 hr postpartum. The client is Rh-negative and her newborn and Rh-positive. The client asks why an indirect Coombs test was ordered by the provider. Which of the following is an appropriate response by the nurse?

"It detects Rh-positive antibodies in the mother's blood"

The nurse explains the purpose of effleurage to a client in early labor. Which statement should the nurse include in the explanation?

"It is light stroking of the abdomen to facilitate relaxation during labor and provide tactile stimulation to the fetus."

On the second postpartum day, a client tells the nurse she feels anxious and tearful. Which response by the nurse would be appropriate?

"It isn't unusual to have those feelings after delivery."

A nurse provides explanation to a client prescribed methylergonovine maleate (Methergine) in the immediate postpartum period regarding this medication. Which of the following statements made by the client demonstrates understanding of rationale for administration?

"It will help to prevent and control bleeding if it occurs." (Methylergonovine maleate is an ergot alkaloid that stimulates smooth muscles. Because the smooth muscle of the uterus is especially sensitive to the medication, it is used in the postpartum period to stimulate the uterus to contract and prevent or control postpartum hemorrhage.)

A new mother who's breast-feeding asks how quickly she can expect to lose the 40 lb she gained during pregnancy. Which response by the nurse is best?

"It's important to avoid dieting while your milk supply is being established; a well-balanced diet with gradual weight loss is recommended."

After delivering her second baby, the client tells the nurse that she wants to breast-feed this baby. She indicates that she was unsuccessful at breast-feeding her first child and that she bottle-fed after 3 days of trying to nurse. Which of the following responses would best support this client's breast-feeding efforts?

"It's important to room-in with your newborn so that you can respond to her nursing cues."

The night nurse reports that a postpartum client is homeless, has poor hygiene, and has tested positive for the human immunodeficiency virus (HIV). The nurse assigned to care for the client requests that the assignment be changed because she's pregnant and doesn't want to risk exposure. Which response by the charge nurse indicates an understanding of the ethical responsibilities of a professional nurse?

"It's inappropriate to refuse this assignment; all clients should be treated equally."

The nurse is teaching a client how to use a diaphragm. Which instruction should the nurse provide?

"Leave the diaphragm in place for at least 6 hours after intercourse."

A nurse is caring for a client who is beginning to breastfeed her newborn after delivery. The new mother states, "I don't want to take anything for pain because I am breastfeeding." Which of the following statements should the nurse make?

"We can time your pain medication so that you have an hour or two before the next feeding."

Deep Vein Thrombosis S&S

Leg pain Chills Unilateral swelling, warmth, redness Warm extremity Calf tenderness Elevated temp Cough Tachycardia

A client has had a midline episiotomy. In relation to a mediolateral episiotomy, the nurse anticipates that this client will generally experience:

Less pain (Midline episiotomies are effective, easily repaired, and generally result in less pain. The blood loss is greater and the repair is more difficult and painful with the mediolateral episiotomy than the midline episiotomy.)

As a postpartum client adapts to her maternal role, she progresses through several phases. During which phase does she begin to accept the neonate as a separate individual?

Letting-go phase

After admission to the labor and delivery area, a client undergoes routine tests, including a complete blood count, urinalysis, Venereal Disease Research Laboratory test, and gonorrhea culture. The gonorrhea culture is positive, although the client doesn't have signs and symptoms of this disease. What is the significance of this finding?

Maternal gonorrhea may cause an eye infection in the neonate.

During labor, a primigravid client receives epidural anesthesia, and the nurse assists in monitoring maternal and fetal status. Which finding suggests an adverse reaction to the anesthesia?

Maternal hypotension

Fourth stage:

Maternal stabilization of vital signs

A nurse is admitting a client who experienced a vaginal birth 2 hr ago. The client is receiving an IV of LR with 25 units of oxytocin infusing and has large rubra lochia. Vital signs include BP 146/95, pulse 80, RR 18. The nurse reviews the prescriptions from the provider. Which of the following prescriptions from the provider. Which of the following prescriptions requires clarification?

Methylergonovine 0.2 mg IM now

A nurse is caring for a newborn with respiratory distress syndrome (RDS). Which of the following data obtained by the nurse indicate potential complications associated with this disorder?

No audible breath sounds in left lung; heart sounds louder in right side of chest (Pneumothorax is a complication associated with RDS. Clinical signs of pneumothorax include a sudden rapid deterioration in condition, tachypnea, grunting, pallor, cyanosis, decreased breath sounds in the affected lung, shifting of the cardiac apex away from the affected lung, bradycardia, and hypertension. Options 1, 2, and 3 are normal findings.)

Caput Succedaneum is:

Normal Should resolve within 24 hrs

A new mother is attempting to breast-feed for the first time. The nurse notices that the client has inverted nipples. What nursing action can the nurse take to assist the client in breast-feeding the newborn?

Provide breast shells and assist the mother with using a breast pump before each feeding to make the nipples easier for the newborn to grasp. (Wearing breast shells and using a breast pump before each feeding will make it easier for the newborn to grasp the nipple. True inverted nipples will retract if the areola is pressed between the thumb and forefinger, making option 2 incorrect. Option 3 is appropriate advice for mothers experiencing inverted nipples. Option 4 will only make the mother cold, and has no effect on inverted nipples.)

The nurse in a labor room is preparing to care for a client with hypertonic uterine contractions. The nurse is told that the client is experiencing uncoordinated contractions that are erratic in their frequency, duration, and intensity. What is the priority nursing action?

Provide pain relief measures.

The nurse is preparing to care for a client with hypertonic labor. The nurse is told that the client is experiencing uncoordinated contractions that are erratic in their frequency, duration, and intensity. Which is the priority nursing intervention?

Provide pain relief measures.

A pregnant client experienced a uterine rupture with subsequent fetal death. After ensuring that the client is physiologically stable, the nurse takes which approach as the best first step to support the client psychologically?

Collect data regarding how the client perceived the event. (As a result of anesthesia, anxiety, and the experience of a sudden catastrophic event, the client may well have experienced a decreased ability to take in and process information. The nurse should first identify the client's perception of the event before deciding how to intervene. Having time to interact with the infant may be helpful but not as a first step. The remaining options are not helpful because they are not therapeutic or deal with subjects the client may not be ready to face.)

The nurse in a delivery room is assessing a client immediately after delivery of the placenta. Which maternal observation could indicate uterine inversion and require immediate intervention?

Complaints of severe abdominal pain

The nurse-midwife determines that a client is in the second stage of labor and may start pushing. What marks the beginning of the second stage, and what marks the end?

Complete cervical dilation; delivery of the neonate

A labor room nurse is performing an assessment on a client in labor and notes that the fetal heart rate (FHR) is 158 beats/minute and regular. The client's contractions are every 5 minutes, with a duration of 40 seconds and of moderate intensity. On the basis of these assessment findings, what is the appropriate nursing action?

Continue to monitor the client.

The nurse has collected the following data on a client in labor. The fetal heart rate (FHR) is 154 beats/min and is regular, and contractions have moderate intensity, occur every 5 minutes, and have a duration of 35 seconds. Using this information, what is the appropriate action for the nurse to take?

Continue to monitor the client.

The nurse is preparing to care for a client in labor. The health care provider has prescribed an intravenous (IV) infusion of oxytocin. The nurse ensures that which intervention is implemented before initiating the infusion?

Continuous electronic fetal monitoring

During labor, a client's cervix fails to dilate progressively, despite her uncomfortable uterine contractions. To augment labor, the physician orders oxytocin (Pitocin). When preparing the client for oxytocin administration, the nurse describes the contractions the client is likely to feel when she starts to receive the drug. Which description is accurate?

Contractions will be stronger and more uncomfortable and will peak more abruptly.

A nurse is providing education to a client during the first prenatal visit. Which of the following statements by the client should indicate to the nurse a need for clarification? A. "I should drink about 2 liters of fluid each day." B. "I should not drink alcoholic beverages during my pregnancy." C. "I can have a moderate amount of caffeine daily." D. "I should increase my calcium intake to 1,500mg per day."

D. "I should increase my calcium intake to 1,500mg per day." i: A woman's dietary reference intake (DRI) of calcium for pregnancy and lactation is the same for a woman who is not pregnant. The DRI for a woman older than 19 years of age is 1,000 mg/day, which should supply enough calcium for fetal bone and tooth development and to maintain maternal bone mass.

A nurse is providing preconception counseling for a client who is planning a pregnancy. Which of the following supplements should the nurse recommend to help prevent neural tube defects in the fetus? A. Calcium B. Iron C. Vitamin C D. Folic acid

D. Folic acid i: Adequate amounts of folic acid before conception and during the first trimester of pregnancy are necessary for fetal neural tube development. This vitamin helps prevent spina bifida and other neurological disorders.

A nurse is assessing a client in labor who has had epidural anesthesia for pain relief. Which of the following findings should the nurse identify as a complication from the epidural block? A. Vomiting B. Tachycardia C. Respiratory depression D. Hypotension

D. Hypotension i: Maternal hypotension is an adverse effect of epidural anesthesia. The nurse should administer an IV fluid bolus prior to the placement of the epidural catheter to decrease the likelihood of this complication.

A nurse is caring for a client who is scheduled for a amternal serum-alpha-fetoprotein test at 15 weeks of gestation. The nurse provides which of the following explanations about this test to the client? A. This test assesses fetal lung maturity. B. It assesses various markers of fetal well-being. C. This test identifies an Rh incompatibility between the mother and fetus. D. It is a screening test for spinal defects in the fetus.

D. It is a screening test for spinal defects in the fetus. i: The maternal serum alpha-fetoprotein (MSAFP) screening test is used to identify suspected neural tube defects (NTDs) and abdominal wall defects. These include spina bifida, microcephaly, and anencephaly. This tool is the basis for further testing, such as amniocentesis and specialized ultrasounds.

A charge nurse observes a nurse checking fetal heart tones (FHT) for a client who is at 12 weeks of gestation. Which of the following actions by the nurse indicates a need for intervention by the charge nurse? A. Places a pillow under the client's head B. Counts the fetal heart rate for a full minute C. Auscultates about the symphysis pubis D. Listens with a fetoscope

D. Listens with a fetoscope i: A fetoscope is not able to detect FHT this early in the pregnancy. The nurse should use a Doppler or ultrasound stethoscope. Typically at 12 weeks, the heart tones will be heard midline just above the symphysis pubis with a Doppler or ultrasound device. A fetoscope can be used to assess FHT later in the pregnancy, around 16 to 20 weeks.

A nurse is preparing a client who is in active labor for an epidural analgesia. Which of the following actions should the nurse take? A. Have the client stand at the bedside with her arms at her side. B. Administer a 500mL bolus of 5% dextrose in water prior to induction. C. Inform the client the anesthetic effect will last for approximately 6hr. D. Obtain a 30min electronic fetal monitoring (EFM) strip prior to induction.

D. Obtain a 30min electronic fetal monitoring (EFM) strip prior to induction. i: The nurse should obtain a 20 to 30 min EFM strip before induction of the spinal anesthesia. The strip should be evaluated as baseline information. After induction, fetal heart rate and pattern is assessed and documented every 5 to 10 min and emergency care is provided for fetal distress, such as bradycardia or late decelerations.

A nurse is caring for an antepartum client whose laboratory findings indicate a negative rubella titer. Which of the following is the correct interpretation of this data? A. The client is not experiencing a rubella infection at this time. B. The client is immune to the rubella virus. C. The client requires a rubella vaccination at this time. D. The client requires a rubella immunization following delivery.

D. The client requires a rubella immunization following delivery. i: A negative rubella titer indicates that the client is susceptible to the rubella virus and needs vaccination following delivery. Immunization during pregnancy is contraindicated because of possible injury to the developing fetus. Following rubella immunization, the client should be cautioned not to conceive for 1 month.

A nurse is assessing a client who is in active labor and notes that the presenting part is at 0 station. Which of the following is the correct interpretation of this clinical finding? A. The fetal head is in the left occiput posterior position. B. The largest fetal diameter has passed through the pelvic outlet. C. The posterior fontanel is palpable. D. The lowermost portion of the fetus is at the level of the ischial spines.

D. The lowermost portion of the fetus is at the level of the ischial spines. i: The presenting part is at 0 station when its lowermost portion is at the level of an imaginary line drawn between the client's ischial spines. Levels above the ischial spines are negative values: -1, -2, -3. Levels below the ischial spines are positive values: +1, +2, +3.

A nurse enters a new mother's room and finds that the mother is crying and that the infant is undressed on the bed in front of the mother. The mother looks at the nurse and says, "I can't even dress this baby!" After reassuring the client, the nurse determines that the appropriate nursing action would be to:

Have the mother place the infant in the bassinet and assist the mother in dressing the baby. (The infant needs to be placed in the bassinet for safety. The mother needs to be reassured that she can safely care for her infant, and the nurse should assist the mother in dressing the baby. Option 1 is incorrect because the infant needs to be placed in the bassinet for safety. Options 2 and 3 are incorrect because these actions do not address the mother's needs. Option 4 is the only option that focuses on the mother's feelings and needs and the safety of the infant.)

Initial client assessment information includes blood pressure 160/110 mm Hg, pulse 88 beats/minute, respiratory rate 22 breaths/minute, and reflexes +3/+4 with 2 beat clonus. Urine specimen reveals +3 protein, negative sugar and ketones. Based on these findings, the nurse would expect the client to have which complaints?

Headache, blurred vision, and facial and extremity swelling

The nurse is monitoring a client who is in the active phase of labor. The client has been experiencing contractions that are short, irregular, and weak. Which type of labor dystocia should the nurse document that the client is experiencing?

Hypotonic

nurse caring for client, considering several methods of contraception, ID as being most reliable?

IUD -intrauterine device

What is the most likely reason for a temperature of 99.8° F (37.7° C) during the first 24 hours postpartum?

Dehydration

Third stage of labor:

Delivery of placenta

An ultrasound is performed on a client with suspected abruptio placentae, and the results indicate that a placental abruption is present. Which intervention should the nurse prepare the client for?

Delivery of the fetus

A client delivers a neonate prematurely at 28 weeks' gestation. The neonate is placed in the neonatal intensive care unit (NICU). Three days later, the client's husband seems withdrawn and barely speaks to the staff when visiting his child in the NICU. His behavior indicates that he's in which stage of grief?

Depression

The nurse is monitoring a client in labor whose membranes ruptured spontaneously. What is the initial nursing action?

Determine the fetal heart rate.

A nurse is caring for a client who is postpartum who asks the nurse when her breast milk will "come in." Which of the following responses should the nurse make?

In 3 to 5 days

A client with diabetes mellitus is in labor. She tells the nurse she has had trouble controlling her blood glucose level recently. She says she didn't take her insulin when the contractions began because she felt nauseous; about 1 hour later, when she felt better, she ate some soup and crackers but didn't take insulin. Now, she reports increased nausea and a flushed feeling. The nurse notes a fruity odor to her breath. What do these findings suggest?

Diabetic ketoacidosis

The nurse is assisting a client undergoing induction of labor at 41 weeks of gestation. The client's contractions are moderate and occurring every 2 to 3 minutes, with a duration of 60 seconds. An internal fetal heart rate monitor is in place. The baseline fetal heart rate has been 120 to 122 beats/minute for the past hour. What is the priority nursing action?

Discontinue the infusion of oxytocin.

A prenatal client with severe abdominal pain is admitted to the maternity unit. The nurse is monitoring the client closely because concealed bleeding is suspected. Which assessment findings indicate the presence of concealed bleeding? Select all that apply.

Increase in fundal height Hard, boardlike abdomen Persistent abdominal pai

A nurse on the labor and delivery unit is caring for a patient who is having induction of labor with oxytocin administered through a secondary IV line. Uterine contractions occur every 2 min, last 90 sec, and are strong to palpation. The baseline fetal heart rate is 150/min, with uniform decelerations beginning at the peak of the contraction and a return to baseline after the contraction is over. Which of the following actions should the nurse take?

Discontinue the infusion of the IV oxytocin.

During a staff meeting, the nurse-manager shares the results of the most recent client satisfaction survey. More than half of all clients who responded feel the nurses are less than empathetic toward women in labor. Which action should the nurse-manager and her staff take to address this issue?

Discuss ways to improve their approach to clients in labor, then hold weekly progress meetings.

A nurse is caring for a client who is 1 hr postpartum and observes a large amount of lochia rubra and several small clots on the client's perineal pad. The fundus is midline and firm at the umbilicus. Which of the following actions should the nurse take?

Document the findings and continue to monitor the client.

The nurse is caring for a client in labor and is monitoring the fetal heart rate patterns. The nurse notes the presence of episodic accelerations on the electronic fetal monitor tracing. Which action is most appropriate?

Document the findings and tell the mother that the pattern on the monitor indicates fetal well-being.

A nurse is caring for a newborn immediately following birth. After assuring a patent airway, what is the priority nursing action?

Dry the skin.

A nurse suspects her client has a pulmonary embolism when the client exhibits which of these signs and symptoms?

Dyspnea, tachypnea, and tachycardia (Pulmonary embolism is the passage of a thrombus into the lungs. The usual signs and symptoms are dyspnea, tachypnea, tachycardia, a congested cough (not a dry cough), hemoptysis (not hematemesis), pleuritic chest pain, and a feeling of impending doom. Back pain, edema, skin tenderness, hematemesis and increased skin temperature are not associated with pulmonary embolism.)

During assessment for admission to the labor and delivery area, a client and her husband ask the nurse whether their sons, ages 8 and 10, can witness the childbirth. Before answering this question, the nurse should consider which guideline?

Each child attending the childbirth should have a separate support person.

A client in labor tells the nurse-midwife that she feels a strong urge to push. Physical examination reveals that her cervix is not completely dilated. The nurse-midwife tells her not to push yet. What is the rationale for this instruction?

Early pushing may cause edema and impede fetal descent.

During a childbirth education class, a nurse-educator discusses pain control techniques used during labor and delivery. Which technique most effectively helps a client cope with the pain of uterine contractions?

Effleurage and other cutaneous stimulation

A nurse in the postpartum unit is instructing a mother regarding lochia and the amount of expected lochia drainage. The nurse instructs the mother that the normal amount of lochia may vary but should never exceed:

Eight pads a day (The normal amount of lochia may vary with the individual but should never exceed eight pads a day. The average number of pads used daily is six.)

A postpartum client has lost 700 mL of blood. The vital signs indicate hypovolemia and the uterus remains atonic in spite of treatment. The nurse assisting in caring for the client understands the treatment that is necessary in this situation and prepares the client for:

Emergency surgery (Options 1, 2, and 4 identify interventions to reverse uterine atony. When uterine atony cannot be reversed, surgery is required.)

Pain S&S:

Increased BP Tachycardia Hyperventilation

During the active phase of the first stage of labor, a client undergoes an amniotomy. After this procedure, which nursing diagnosis takes the highest priority?

Ineffective tissue perfusion (cerebral) related to cord compression

A nurse is adding to a plan of care for a postpartum client. Which intervention will promote parent-infant bonding?

Encourage her to hold the infant even when the infant is crying. (Holding the infant close and allowing the infant to feel the warmth initiates a positive experience for the mother and consoles the infant. The use of a high-pitched voice and participating in infant care are additional methods of promoting parent-infant attachment. Infants should not be allowed to sleep in the parental bed. The parents require time alone as a couple. Additionally, the danger of suffocation of the infant exists if the infant is allowed to sleep between the parents.)

A nurse is assigned to care for a client admitted to the postpartum unit following delivery of a full-term healthy infant. The nurse checks the mother's temperature and notes that it is 100.4° F (38° C). Which nursing action would be appropriate?

Encourage oral fluids. (Temperatures up to 100.4° F (38° C) in a mother during the first 24 hours after birth are often related to the dehydrating effects of labor. Increasing hydration by encouraging oral fluids will help bring the temperature to a normal reading. Options 2, 3, and 4 are unnecessary actions at this time.)

A client gives birth to a stillborn neonate at 36 weeks' gestation. When caring for this client, which strategy by the nurse would be most helpful?

Encourage the client to see, touch, and hold the dead neonate.

Which action should the nurse take to promote the descent of the fetus's presenting part?

Encourage the client to void every 2 hours.

When assessing a client who gave birth 12 hours ago, the nurse measures an oral temperature of 99.6° F (37.5° C), a heart rate of 82 beats/minute, a respiratory rate of 18 breaths/minute, and a blood pressure of 116/70 mm Hg. Which nursing action is appropriate?

Encouraging increased fluid intake

A nurse who is caring for a postpartum mother being tested for endometritis notes that the client has little interest in caring for her infant. What intervention would best facilitate the client's participation in infant care?

Encouraging the client to take pain medication as prescribed (Keeping the client comfortable by appropriately using prescribed analgesics will facilitate her interest in caring for the infant. Nursing responsibilities for the care of a client with endometritis include maintaining adequate hydration (3000 to 4000 mL/day), promoting bedrest in Fowler's position to facilitate drainage and lessen congestion. The correct option is the only nursing intervention that demonstrates the nurse's understanding of both the physiological and psychosocial needs of the postpartum client experiencing endometritis.)

The nurse is caring for a woman receiving a lumbar epidural anesthetic block to control labor pain. What should the nurse do to prevent hypotension?

Ensure adequate I.V. hydration according to the physician's order before the anesthetic is administered.

A client in labor is dilated 10 cm. At this point in the labor process, at least how often should the nurse assess and document the fetal heart rate?

Every 15 minutes

A client is in the second stage of labor. During this stage, how frequently should the nurse assess her uterine contractions?

Every 15 minutes

A nurse is preparing to care for a woman in the immediate postpartum period who has just delivered a healthy newborn. The nurse plans to take the woman's vital signs:

Every 15 minutes for the first 2 hours (During the immediate postpartum period, vital signs are taken every 15 minutes in the first hour after birth, every 30 minutes for the next 2 hours, and every hour for the next 2 to 6 hours. Vital signs are monitored thereafter every 4 hours for 24 hours and every 8 to 12 hours for the remainder of the hospital stay. )

While a client is holding and talking to her newborn immediately following delivery, she begins to cry. The nurse interprets this behavior as indicating the client is:

Experiencing a normal response to birth (The birth of a baby is an emotionally charged moment for new parents. Crying can be a normal expression of emotions surrounding birth. Holding, eye contact, and touch are signs of healthy maternal-newborn attachment.)

A pregnant 39-week-gestation client arrives at the labor and delivery unit in active labor. On confirmation of labor, the client reports a history of herpes simplex virus (HSV) to the nurse, who notes the presence of lesions on inspection of the client's perineum. Which should be the nurse's initial action?

Explain to the client why a cesarean delivery is necessary.

The nurse is preparing to provide contraceptive counseling for a young client. What should the nurse plan to do first?

Explore her own personal beliefs and feelings about contraception.

A nurse is caring for a client who experienced a vaginal birth 12 hr ago. The nurse recognizes the client is in the dependent, taking in phase of maternal postpartum adjustment. Which of the following findings should the nurse expect during this phase?

Expressions of excitement

nurse in prenatal clinic is caring for a client who asks what is her EDD, LMP May 4, 2015, appropriate response?

Feb 11 2016 count 9months ahead plus 7 days

The nurse is performing an assessment of a client who is scheduled for a cesarean delivery at 39 weeks of gestation. Which assessment finding indicates the need to contact the health care provider (HCP)?

Fetal heart rate of 180 beats/minute

The nurse is caring for a client with mild active bleeding from placenta previa. Which assessment factor indicates that an emergency cesarean section may be necessary?

Fetal heart rate of 80 beats/minute

A nurse on the labor and delivery unit is caring for a client who is having a difficult, prolonged labor with severe backache. Which of the following contributing causes should the nurse identify?

Fetal position is persistent occiput posterior.

BPP assess for

Fetal well being

A nurse is caring for a client with placenta previa who is at high risk for infection and hemorrhage, as a result of this condition. The nurse plans care based on what information related to the condition?

Fewer muscle fibers in the lower segment of the uterus will result in poor contractions. (In placenta previa, the placenta is in the lower segment of the uterus near or over the internal cervical os. After delivery, the muscle tissue in that segment has fewer muscle fibers and the weak contractions cannot compress the open vessels at the site. Infection is a high risk because the placenta site is located near the vagina, and any vaginal organisms can easily travel to the uterus, causing infection. Options 1, 2, and 4 are incorrect.)

The clients bladder needs to be _____ before an ultrasound.

Full

Second stage of labor:

Full dilation Intense contractions BIRTH!!

A nurse is assessing a client who is 8 hr postpartum and multiparous. Which of the following findings should alert the nurse to the client's need to urinate?

Fundus three fingerbreadths above the umbilicus

nurse in a prenatal clinic is reviewing client record who is 28 weeks of gest. the hx includes 1 preg, terminated at 9 weeks, birth of twins at 36 weeks, abortion at 15 weeks. What is GTPAL?

G 4 T 0 P 1 A 2 L 2

A nurse is assisting in preparing a plan of care for a client who just delivered a dead fetus. The appropriate initial intervention in meeting the emotional needs of the client and her spouse is which of the following?

Gather data from the client and spouse about the perception of the event. (The most appropriate initial intervention in planning to meet the emotional needs of the client and her spouse is to gather data about the perception of the event. Although options 1, 2, and 3 are likely to be a component of the plan of care, the initial intervention is to assess the perception of the event.)

The nurse is preparing to perform a physical examination on a postpartum client. Which statement best explains why the nurse must wear gloves during this examination?

Gloves are required for standard precautions.

A nurse is caring for a client who experienced a vaginal birth 3 hr ago. Upon palpation, the fundus is displaced to the right of midline, is firm, and is two finger breadths above the umbilicus. Which of the following actions should the nurse complete at this time?

Have the client urinate

When assessing a client who has just delivered a neonate, the nurse finds that the fundus is boggy and deviated to the right. What should the nurse do?

Have the client void.

A client in the first stage of labor enters the labor and delivery area. She seems anxious and tells the nurse that she hasn't attended childbirth education classes. Her husband, who accompanies her, is also unprepared for childbirth. Which nursing intervention would be effective for the couple at this time?

Instruct the husband on touch, massage, and breathing patterns.

A nurse is collecting data on a client who is 6 hours postpartum following delivery of a full-term healthy newborn. The client tells the nurse that she feels faint and dizzy. Which nursing action would be appropriate?

Instruct the mother to request help when getting out of bed. (Orthostatic hypotension may occur during the first 8 hours after birth. Feelings of faintness and dizziness are signs that caution the nurse to be aware of the client's safety. The nurse should advise the mother to get help the first few times getting out of bed. Option 1 requires a health care provider's prescription. Option 3 is not a helpful action. Option 4 is unnecessary.)

The nurse explains to new parents the importance of maintaining their infant's safety during hospitalization. Which action best ensures the infant's safety?

Instructing the mother to notify staff when she showers to avoid leaving the infant unattended

A nurse is assisting in administering beractant (Survanta) to a premature infant who has respiratory distress syndrome. The nurse understands that the medication will be administered by which of the following routes?

Intratracheal (Respiratory distress is common in premature neonates and may be due to lung immaturity as a result of surfactant deficiency. The mainstay of treatment is the administration of exogenous surfactant, which is administered by the intratracheal route)

The nurse is caring for a primigravid client in the labor and delivery area. Which condition would place the client at risk for disseminated intravascular coagulation (DIC)?

Intrauterine fetal death

A nurse is checking lochia discharge in a woman in the immediate postpartum period and notes that the lochia is bright red and contains some small clots. The nurse determines that this finding:

Is normal (Lochia, the uterine discharge present after birth, initially is bright red and may contain small clots. During the 2 hours after birth, the amount of uterine discharge should be approximately that of a heavy menstrual period. After that time the lochial flow should steadily decrease and the color of the discharge should change to a pinkish red or reddish brown.)

A nurse is teaching a client who is postpartum and has a new prescription for an injection of Rho (D) immunoglobulin. Which of the following should be included in the teaching?

It prevents the formation of Rh antibodies in mothers who are Rh negative.

Between 18 and 30 weeks the fundal height should measure what?

It should equal the week of gestation

The nurse is assisting with caring for a postpartum client who is experiencing uterine hemorrhage. When planning to meet the psychosocial needs of the client, the nurse would:

Keep the client and her family members informed of her progress. (Keeping the client and her family informed about her condition will help minimize fear and apprehension. )

The nurse is creating a plan of care for a pregnant client with a diagnosis of severe preeclampsia. Which nursing actions should be included in the care plan for this client? Select all that apply.

Keep the room semi-dark. Initiate seizure precautions. Pad the side rails of the bed. Avoid environmental stimulation

Which of the following options best describes the anticipated actions in the taking-hold phase of the maternal attachment process?

Kissing, embracing, and caring for the infant

In formulating the plan of care, which problem is most important to address for a postpartum client who has expressed concerns about not knowing how to care for her newborn?

Lack of knowledge regarding ability to care for the newborn (Lack of knowledge regarding ability to care for the newborn implies a lack of information or psychomotor skills concerning a condition or treatment. This problem best describes the situation presented in the question. Lack of ability to cope implies that the person is unable to manage stressors adequately. Lack of self-esteem with regard to caring for the newborn represents temporary negative feelings about self in response to an event. Grieving in a dysfunctional way implies prolonged unresolved grief leading to detrimental activities.)

A pregnant client arrives at the health care facility, stating that her bed linens were wet when she woke up this morning. She says no fluid is leaking but complains of mild abdominal cramps and lower back discomfort. Vaginal examination reveals cervical dilation of 3 cm, 100% effacement, and positive ferning. Based on these findings, the nurse concludes that the client is in which phase of the first stage of labor?

Latent phase

A client in the first stage of labor is being monitored with an external fetal monitor. The nurse notes variable decelerations on the monitoring strip. Into what position should the nurse assist the client?

Left lateral

When performing a postpartum assessment on a client, the licensed practical nurse (LPN) notes clots in the lochia. The LPN examines the clots and notes that they are larger than 1 cm. Which of the following nursing actions is appropriate?

Notify the registered nurse (RN). (Normally a few small clots may occur in the first 1 to 2 days after birth from pooling of the blood in the vagina. Clots larger than 1 cm are considered abnormal. The cause of such clots, such as uterine atony or retained placental fragments, must be determined and treated to prevent further blood loss. Although the findings would be documented, the most appropriate action is to notify the RN. Reassessing the client in 2 hours would delay necessary treatment. Increasing oral intake of fluids would not be an appropriate action in this situation.)

The nurse is caring for a postpartum client. At 4 hours postpartum, the client's temperature is 102° F (38.9° C). The appropriate nursing action would be to:

Notify the registered nurse, who will then contact the health care provider (HCP). (During the first 24 hours postpartum, the mother's temperature may be elevated as a result of dehydration. However, if the temperature is more than 2° F above normal, this may indicate infection, and the HCP will need to be notified. Applying cool packs to the abdomen is an inappropriate action and additionally, this action requires a prescription. The remaining options may be a component of care but are not the most appropriate based on the data in the question.)

A nurse is caring for a newborn whose mother had an elevated temperature during a prolonged labor. Which intervention(s) would be important to include in the newborn's plan of care?

Observe vital signs and central nervous system status frequently during the first 2 days. (Clinical signs of sepsis in the newborn include temperature instability, tachycardia, respiratory changes, and central nervous symptoms such as lethargy or irritability. If sepsis is a risk, the nurse would monitor vital signs and central nervous system status frequently. Promoting early maternal-newborn interaction is always important but is unrelated to this question. Delaying a feeding is not appropriate.)

A licensed practical nurse (LPN) who typically works in the neonatal intensive care unit is being cross-trained to work with postpartum clients. The charge nurse is busy with a delivery and assigns her to stock rooms, which is typically the responsibility of a nurse's aid. As she enters a client's room, the LPN notices that a client looks pale and shaky. Which action should she take?

Obtain a set of vital signs, check the client's fundus and flow, and compare the findings to baseline data.

The maternity nurse is preparing for the admission of a client in the third trimester of pregnancy who is experiencing vaginal bleeding and has a suspected diagnosis of placenta previa. The nurse reviews the health care provider's prescriptions and should question which prescription?

Obtain equipment for a manual pelvic examination.

Three hours after birth, a client becomes weak and dizzy as she attempts to ambulate for the first time. The client's hemoglobin level at the end of pregnancy was 10.4 g/dl. Two hours later she asks to use the bathroom. Which nursing intervention is the top priority?

Obtaining the assistance of a second nurse before attempting to assist the client with ambulation

A client in active labor is sweating profusely and has minimal urine output. How should the nurse intervene?

Offer the client ice chips and ask the charge nurse to notify the physician of the low urine output.

A client is at the end of her 1st postpartum day. When assessing her uterus, the nurse expects to find the top of the fundus at the midline and at which position?

One fingerbreadth below the umbilicus

The nurse is assigned to care for a client with hypotonic uterine dysfunction and signs of a slowing labor. The nurse is reviewing the health care provider's prescriptions and should expect to note which prescribed treatment for this condition?

Oxytocin infusion

A postpartum nurse is about to reinforce discharge instructions to a postpartum client who delivered a healthy newborn infant. The occurrence of which event should be reported to the health care provider?

Pain, redness, or swelling in the breasts (Signs of infection include pain, redness, heat, and swelling of a localized area of the breast. If these symptoms occur, the client needs to contact the health care provider. Options 1, 3, and 4 are normal changes that occur in the postpartum period.)

An amniotomy is performed on a client in labor. On the amniotic fluid examination, the delivery room nurse should identify which findings as normal?

Pale straw in color, with flecks of vernix

A woman in active labor has requested a regional anesthetic. She is currently 5 cm dilated. The health care provider has prescribed an epidural block. Which nursing intervention should be implemented after the epidural block has been placed?

Palpate the bladder at frequent intervals.

A nurse is caring for a client 2 hr after a spontaneous vaginal birth and the client has saturated two perineal pads with blood in a 30 min period. Which of the following is the priority nursing intervention at this time?

Palpate the client's uterine fundus

A client has just experienced a precipitate delivery. The nurse observes that the mother is lying quietly in bed and touches the infant only briefly and occasionally. How will the nurse be most therapeutic in this situation?

Provide support to the mother regardless of her reaction to the newborn. (After a precipitate delivery, the woman may need help to process what has happened and time to assimilate what has happened. The mother may be exhausted, in pain, stunned by the rapid nature of the delivery, or simply following cultural norms. Providing support to the mother is the most therapeutic action by the nurse. Culturally, competent care is correct but is not the best option because it does not take into account the other factors that could contribute to the mother's reactions after birth. Documentation does not enhance the therapeutic relationship. Breast-feeding is an appropriate nursing intervention, but the question does not indicate whether the mother is going to be breast-feeding.)

A nurse is caring for a client who is 1 day postpartum and is taking a sitz bath. To determine the client's tolerance of the procedure, which of the following assessments should the nurse perform?

Pulse rate

A nurse is caring for a woman who has delivered a baby after a pregnancy with a placenta previa. The nurse monitors the client frequently, knowing that the client is at risk for:

Postpartum hemorrhage (Because the placenta is implanted in the lower uterine segment that does not contain the same intertwining musculature as the fundus of the uterus, this site is more prone to bleeding. The nurse monitors the client frequently for signs of postpartum hemorrhage.)

A client who is breast-feeding her infant is experiencing breast engorgement. The nurse suggests breast pumping to relieve the breast engorgement. Which instruction should the nurse provide?

Pump each breast for at least 10 minutes every 3 to 4 hours; pump at night only if she's awake.

When coaching a client to push, the nurse should encourage her to use which technique?

Pushing when she feels like pushing

A nurse is caring for the postpartum client who is diagnosed with a low-lying placenta. The nurse monitors the client carefully for which complication?

Postpartum hemorrhage (The lower uterine segment does not contain the same intertwining musculature as the fundus of the uterus, making this site more prone to bleeding. The client is not at greater risk for postpartum infection, coagulopathy, or chronic hypertension with this disorder.)

The nurse is assisting with planning care for a postpartum woman who has small vulvar hematomas. To assist with reducing the swelling, the nurse should:

Prepare an ice pack for application to the area. (The application of ice will reduce the swelling caused by hematoma formation in the vulvar area. Options 1, 2, and 3 will not reduce swelling.)

Six clients are in active labor in the labor and delivery unit. Four additional clients in the early stages of labor were just admitted to the unit. There are three registered nurses (RNs) and two licensed practical nurses (LPNs) assigned to the unit this shift. Which client care assignment provides the best care for these clients?

RNs should be assigned to those in active labor and LPNs should be assigned to those in the early stages of labor.

a nurse is preparing to administer MgSO4 IV to client, who is experiencing preterm labor. priority nursing assessment for client?

RR

During labor, a client greatly relies on her husband for support. They previously attended childbirth education classes, and now he's working with her on comfort measures. Which nursing diagnosis would be appropriate for this couple?

Readiness for enhanced family coping related to participation in pregnancy and delivery

The client received epidural anesthesia during labor and had a forceps delivery after pushing for 2 hours. At 6 hours postpartum, the client's systolic blood pressure (BP) dropped 20 points, the diastolic BP dropped 10 points, and her pulse is 120 beats per minute. The client is very anxious and restless. The nurse is told that the client has a vulvar hematoma. On the basis of this diagnosis, the nurse would plan to:

Prepare the client for surgery. (The information provided in the question indicates that the client is experiencing blood loss. Surgery would be indicated for this complication to stop the bleeding)

nurse caring for client, scheduled for a maternal serum alpha fetoprotein test at 15 wks. explanation about test to client?

SCREENING test for spinal defects in fetus neural tube defects abdominal wall defects microcephaly anencepaly basis for possible further testing - Aminiocentesis and specialized ultrasounds

The nurse assesses a client who gave birth 24 hours earlier. Which of the following findings reveals the need for further evaluation?

Scant lochia rubra

A nurse has reinforced instructions to a new mother about how to perform postpartum exercises. The nurse determines that the client understands the instructions when she states that:

She should alternately contract and relax the muscles of the perineal area. (Kegel exercises are extremely important to strengthen the muscle tone of the perineal area. Postpartum exercises can begin soon after birth. The initial exercises should be simple, with progression to increasingly strenuous exercises. Women who maintain the perineal muscle tone may benefit in later life by the development of less stress urinary incontinence.)

A client in the postpartum unit complains of sudden, sharp chest pain. The nurse notes that the client is tachycardic and the respiratory rate is elevated. The nurse suspects a pulmonary embolism. The initial nursing action would be which of the following?

Prepare to administer oxygen at 8 to 10 L by tight face mask. (If pulmonary embolism is suspected, oxygen should be administered at 8 to 10 L by tight face mask. Oxygen is used to decrease hypoxia. The woman also is kept on bedrest with the head of the bed slightly elevated to reduce dyspnea. Morphine sulfate may be prescribed for the client, but this action would not be the initial nursing action. An IV line also will be required, but this action would follow the administration of the oxygen.)

A client who's being admitted to labor and delivery has the following data collection findings: gravida 2 para 1, estimated 40 weeks' gestation, contractions 2 minutes apart, lasting 45 seconds, vertex +4 station. Which of the following would be the priority at this time?

Preparing for immediate delivery

During the intrapartum period, the nurse is caring for a client with sickle cell disease. The nurse ensures that the client receives adequate intravenous fluid intake and oxygen consumption to achieve which outcome?

Prevent dehydration and hypoxemia.

While preparing a client for a postpartum tubal ligation, the nurse overhears the client tell her husband that they can always have reversal surgery if they decide they want more children in the future. Which intervention by the nurse is best?

Privately discussing with the client her understanding of the procedure

The purpose of a vaginal examination for a client in labor is to specifically assess the status of which findings? Select all that apply.

Station Dilation Effacement

Oxytocin (Pitocin) is prescribed to be administered intravenously to a client after a cesarean delivery. The nurse understands that the action of the medication is to:

Stimulate the uterus to contract, thus reducing possible blood loss. (The action of oxytocin is to stimulate the uterus to contract, to control uterine atony, and therefore reduce hemorrhage. Options 2, 3, and 4 are not actions of this medication.)

A multiparous client with pelvic thrombophlebitis is being treated with bed rest and anticoagulant therapy. The nurse should call for assistance immediately if the client experiences which symptom?

Sudden onset of shortness of breath

To promote comfort during labor, the nurse advises a client to assume certain positions and avoid others. Which position may cause maternal hypotension and fetal hypoxia?

Supine position

A client in labor is transported to the delivery room and prepared for a cesarean delivery. After the client is transferred to the delivery room table, the nurse should place the client in which position?

Supine position with a wedge under the right hip

After a precipitate delivery, a nurse notes that a new mother is passive and only touches her newborn briefly with her fingertips. The nurse would do which of the following first to help the woman process what has happened?

Support the mother no matter what her reaction to the newborn is. (There may be many reactions to the birth of a baby. The mother may be exhausted, in pain, stunned by the rapid nature of the delivery, or may be following her cultural norms. The mother may want to process what has happened and will need time to assimilate all that occurred. The new mother requires support, and the nurse needs to provide a nurturing and accepting attitude.)

Which of the following options is the most important aspect of nursing care in the postpartum period?

Supporting the mother's ability to successfully feed and care for her infant

The nurse is assessing the psychosocial status of a postpartum client. Which finding is most likely to promote parent-neonate attachment?

Sustained parent-neonate contact immediately after delivery

What helps prevent thrombosis in women with varicose veins?

TED hose

A nurse is conducting a prenatal session with a group of expectant parents. The nurse tells the parents that the primary hormone that stimulates the secretion of milk is:

Prolactin (Prolactin stimulates the secretion of milk. Testosterone is produced by the adrenal glands in the female and induces the growth of pubic and axillary hair at puberty. Oxytocin stimulates contractions during birth and stimulates postpartum contractions to compress uterine vessels and control bleeding. Oxytocin is also responsible for the "let-down" reflex associated with lactation, but it is not responsible for secretion of the milk. Progesterone stimulates the secretions of the endometrial glands, causing endometrial vessels to become highly dilated and tortuous in preparation for possible embryo implantation.)

A postpartum nurse reinforces information provided to a new mother following a vaginal delivery regarding a sitz bath. The nurse determines that the client understands the purpose of the sitz bath when the client states that it will:

Promote healing of the perineum. (Warm, moist heat provided by a sitz bath is used 24 hours after tissue trauma from a vaginal birth to provide comfort and promote healing and reduce the incidence of infection. Ice is used in the first 24 hours to reduce edema and numb the tissue in the perineal area. Promoting a bowel movement is best achieved by ambulation. Thrombophlebitis prevention is not related to a sitz bath.)

A nurse on the postpartum unit is caring for a group of clients with an assistive personnel (AP). Which of the following tasks should the nurse plan to delegate to the AP?

Provide a sitz bath to a client who has a fourth-degree laceration and is 2 days postpartum

Which of the following if noted in the new mother indicates the need for further data collection by the nurse for signs of postpartum depression?

The mother constantly complains of tiredness and fatigue. (Postpartum depression is not the normal depression that many new mothers experience from time to time. The woman experiencing depression shows less interest in her surroundings and a loss of her usual emotional response toward the family. The woman is also unable to show pleasure or love and may have intense feelings of unworthiness, guilt, and shame. The woman often expresses a sense of loss of self. Generalized fatigue, complaints of ill health, and difficulty in concentrating are also present. The mother would have little interest in food and experience sleep disturbances.)

A postpartum nurse is caring for a mother following delivery of a newborn infant. The nurse performs a perineal assessment on the mother and notes a trickle of bright red blood coming from the perineum. The nurse checks the mother's fundus and notes that it is firm. On review of the mother's record, the nurse also notes that an episiotomy was performed. Based on this information, the nurse determines that:

The bright red bleeding is abnormal and should be reported. (Lochial flow should be distinguished from bleeding originating from a laceration or episiotomy, which is usually brighter red than lochia, and presents as a continuous trickle of bleeding even though the fundus of the uterus is firm. This bright red bleeding is abnormal and needs to be reported. Options 1, 2, and 4 are incorrect interpretations of the assessment data.)

After episiotomy and the delivery of a newborn, the nurse performs a perineal check on the mother. The nurse notes a trickle of bright red blood coming from the perineum. The nurse checks the fundus and notes that it is firm. The nurse determines that:

The bright red bleeding is abnormal and should be reported. (Lochial flow should be distinguished from bleeding that originates from a laceration or an episiotomy, which is usually brighter red than lochia and presents as a continuous trickle of bleeding, even though the fundus of the uterus is firm. This bright red bleeding is abnormal and needs to be reported. Therefore, the other options are incorrect interpretations.)

The nurse is caring for a client in labor. Which assessment findings indicate to the nurse that the client is beginning the second stage of labor? Select all that apply.

The cervix is dilated completely. The spontaneous urge to push is initiated from perineal pressure.

The nurse is reviewing the history of a postpartum client. Which history factor strongly suggests that this client will experience afterpains?

The client is a gravida 6, para 5.

On March 10, the nurse performed an initial assessment on a client admitted to the labor and delivery unit for "rule out labor." The client has not received prenatal care but is certain that the first day of her last menstrual period (LMP) was July 7 the previous year. The nurse plans care based on which interpretation?

The client is possibly in preterm labor.

A client, age 22, is a gravida 1, para 0. During the first 24 hours after delivery, she doesn't show consistent interest in her neonate. How should the nurse interpret her behavior?

The client is showing expected behaviors for the taking-in period.

A clinical pathway is guiding care for an Rh-negative postpartum client who vaginally delivered a 9-lb, 1-oz (4,121-g) baby 5 hours ago. During the delivery, a second-degree median episiotomy was necessary. Which client outcome should be achieved during the first 12 hours postpartum?

The client will verbalize and demonstrate appropriate self-perineal care.

The nurse is discharging a 34-year-old multipara client who, after 16 hours of labor, delivered an 8-lb, 14-oz (4,032-g) baby vaginally. The nurse notes that the mother is rubella-immune with Rh-positive blood. Which client outcome takes priority for this client?

The client will verbalize the importance of reporting changes in lochia flow.

The nurse is providing emergency measures to a client in labor who has been diagnosed with a prolapsed cord. The mother becomes anxious and frightened and says to the nurse, "Why are all of these people in here? Is my baby going to be all right?" Which client problem is most appropriate to address at this time?

The client's fear

The nurse applies an external electronic fetal monitor (EFM) to assess a client's uterine contractions and evaluate the fetal heart rate (FHR). However, the client is uncomfortable and changes positions frequently, making FHR hard to assess. Consequently, the physician decides to switch to an internal EFM. Before internal monitoring can begin, which of the following actions must occur?

The membranes must rupture.

a nurse is caring for a client who is having a nonstress test performed. the fetal heart rate (FHR) is 130 to 150/min, but there has been no fetal movement for 15 min. which of the following actions should the nurse perform? a. immediately report the situation to the client's provider and prepare the client for induction of labor b. encourage the client to walk around without the monitoring unit for 10 min, then resume monitoring c. offer the client a snack of orange juice and crackers d. turn the client onto her left side

c (A nonstress test depends upon fetal movement, and this fetus is most likely asleep. Most fetuses are more active after meals due to the increase in the mother's blood sugar. Giving the mother a snack will promote fetal movement.)

A nurse is reviewing the health care record of a newborn admitted to the nursery; the newborn is suspected of having an imperforate anus. The nurse understands that which documented finding is unassociated with this disorder?

The passage of bloody mucus stool (Clinical manifestations of an imperforate anus include failure to pass meconium stool within 24 hours following birth, absence or stenosis of the anorectal canal, an anal membrane, and an external fistula to the perineum. During neonatal assessment, the defect should be identified easily on sight. However, a rectal thermometer may be necessary to determine patency if meconium stool is not passed. The presence of stool in the urine, the vagina, or a skin dimple should be reported immediately as an indication of abnormal anorectal development. Option 4 is a clinical manifestation of intussusception.)

A nurse is checking the lochia discharge on a 1-day postpartum woman. The nurse notes that the lochia is red and has a foul odor. The nurse determines that this finding indicates:

The presence of infection (Lochia, the discharge present after birth, is red the first 1 to 3 days and gradually decreases in amount. Normal lochia has a fleshy odor similar to the odor of menstrual flow. Foul-smelling or purulent lochia usually indicates infection, and these findings are not normal. Encouraging the woman to drink fluids and ambulate are not accurate interpretations related to the assessment finding.)

The nurse is assigned to care for the client during the postpartum period. The client asks the nurse what the term involution means. The nurse responds to the client, knowing that involution is:

The progressive descent of the uterus into the pelvic cavity, which occurs at a rate of approximately 1 cm/day (Involution is the progressive descent of the uterus into the pelvic cavity. After birth, descent occurs at a rate of approximately one fingerbreadth or 1 cm per day.)

It has been 12 hours since a client's delivery of a newborn. The nurse assesses the mother for the process of involution and documents that it is progressing normally when palpation of the client's fundus is noted at which level? Refer to figure.

The term, "involution," is used to describe the rapid reduction in size and the return of the uterus to a normal condition similar to its pre-pregnant state. Immediately following the delivery of the placenta, the uterus contracts to the size of a large grapefruit. The fundus is situated in the midline between the symphysis pubis and the umbilicus. Within 6 to 12 hours after birth, the fundus of the uterus rises to the level of the umbilicus. The top of the fundus remains at the level of the umbilicus for about a day and then descends into the pelvis approximately one fingerbreadth on each succeeding day.

A client who comes to the labor and delivery area tells the nurse she believes her membranes have ruptured. When obtaining her history, what should the nurse ask about first?

The time of membrane rupture

During the postpartum period, the nurse should assess for signs of normal involution. Which of the following would indicate that the client is progressing normally?

The uterus is descending at the rate of one fingerbreadth per day.

A nurse in the postpartum unit is assigned to care for a client who delivered a full-term, healthy baby. The nurse receives the report and is told that the mother had lost 500 mL of blood during the delivery. When checking the vital signs, the nurse notes that the woman's pulse is 90 beats per minute and is weak and thready. This finding would indicate which of the following to the nurse?

This may be a sign of hemorrhage or shock. (A pulse range of 50 to 70 beats per minute is normal in a mother following delivery and may occur for the first 1 to 2 days after delivery. A weak and thready or rapid pulse is abnormal and may be a sign of hemorrhage or shock. Particular attention should be paid to the pulse rate when there has been a blood loss of 500 mL or greater during or after delivery. Options 1, 3, and 4 are incorrect interpretations.)

A postpartum client with mastitis in the right breast complains that the breast is too sore for her to breast-feed her infant. The nurse tells the client:

To breast-feed from the left breast and gently pump the right breast

The nurse is teaching a postpartum client how to perform Kegel exercises. What is the purpose of these exercises?

To strengthen the perineal muscles

A 32-year-old multipara is admitted to the birthing room after her initial examination reveals her cervix to be at 8 cm, completely effaced (100%), and at 0 station. What phase of labor is she in?

Transitional phase

When caring for a client in the first stage of labor, the nurse documents cervical dilation of 9 cm and intense contractions lasting 45 to 60 seconds and occurring about every 2 minutes. Based on these findings, the nurse should recognize that the client is in which phase of labor?

Transitional phase

A nurse is assisting in developing a plan of care for a client in the fourth stage of labor who received an epidural. Which of the following problems is most likely to occur during this stage?

Urinary retention caused by the loss of sensation to void and rapid bladder filling (The fourth stage of labor is the period of time from 1 to 4 hours after delivery, when the woman's body begins to readjust and relax)

Which finding would lead the nurse to suspect that a client has developed hypovolemic shock caused by postpartum hemorrhage?

Urine output less than 25 ml/hour

A client who is breast-feeding her baby experiences pain, redness, and swelling of her left breast 9 days postpartum. She is diagnosed with mastitis. The nurse teaching the client how to care for her infected breast should include which information?

Use a warm, moist compress over the painful area.

A client's neonate was delivered by cesarean. Which management strategy should be implemented regarding breast-feeding after this type of delivery?

Use the football hold to avoid incisional discomfort.

The nurse is teaching a client how to perform perineal care to reduce the risk of puerperal infection. Which activity indicates that the client understands proper perineal care?

Using a peri bottle to clean the perineum after each voiding or bowel movement

A nurse is caring for a client who is 5 hr postpartum following a vaginal birth of a newborn weighing 9 lb 6 oz. (4252 g). The nurse should recognize that this client is at risk for which of the following postpartum complications?

Uterine atony

The nurse is assessing a pregnant client in the second trimester of pregnancy who was admitted to the maternity unit with a suspected diagnosis of abruptio placentae. Which assessment finding should the nurse expect to note if this condition is present?

Uterine tenderness

The nurse is collecting data from a pregnant client in the second trimester of pregnancy who was admitted to the maternity unit with a suspected diagnosis of abruptio placentae. Which findings are associated with abruptio placentae? Select all that apply.

Uterine tenderness Acute abdominal pain A hard, "boardlike" abdomen Increased uterine resting tone on fetal monitoring

Fetal bradycardia means (less than 60)

Uteroplacental insufficiency Umbilical cord prolapse Maternal hypotension Prolonged umbilical cord compression Fetal congenital heart block Anesthetic meds

A nurse is performing an assessment on a 2-day postpartum mother. The mother complains of severe pain and an intense feeling of swelling and pressure in the vulvar area. After hearing these complaints, the nurse specifically checks the client's:

Vulva for a hematoma (Hematoma is suspected when the client reports pain or pressure in the vulvar area. Massive hemorrhage can occur into the tissues, resulting in hypovolemia and shock; therefore the client's complaints must be checked so that interventions may begin immediately. The client's complaints are not related to options 1, 2, or 4.)

Candida Albicans S&S

Vulvar itching Thick, creamy white vag discharge Vulvar redness White patches on vag walls Gray white patches on tongue and gums

A nurse has instructed a postpartum client who is hepatitis B positive how to safely bottle-feed her newborn to prevent the transmission of the infection. Which action by the client indicates an understanding of this procedure?

Washes and dries her hands before feeding (Hepatitis B virus (HBV) is highly contagious by direct contact with blood and body fluids of infected persons. Strict handwashing before contact with the newborn will assist in prevention of the transmission of infection. Options 1 and 2 are appropriate feeding techniques for bottle-feeding but do not minimize disease transmission for hepatitis B. Option 4 will not affect disease transmission.)

A postpartum nurse is monitoring the amount of lochial flow in a client following delivery. Which activity is a part of the method to accurately determine the amount of flow for documentation purposes?

Weighing the perineal pad before and after use (The most accurate method for determining the amount of lochial flow is to weigh the perineal pads before and after use. Once these two weights are noted, the amount of lochial flow can be accurately determined. Each gram increase in the weight is roughly equivalent to 1 mL of blood loss. To obtain an accurate estimate of lochial flow, the time between pad changes is a factor that must also be incorporated into the analysis. The remaining options are incorrect.)

a nurse is caring for a client who experienced a cesarean birth due to dysfunctional labor. the client states that she is disappointed that she did not have natural childbirth. which of the following responses should the nurse make? a. "it sounds like you are feeling sad that things didn't go as planned" b. "at least you know you have a healthy baby" c. "maybe next time you can have a vaginal delivery" d. "you can resume sexual relations sooner than if you had delivered vaginally"

a

a nurse is caring for a client who is in the first stage of labor, undergoing external fetal monitoring, and receiving IV fluid. the nurse observes variable decelerations in the fetal heart rate on the monitor strip. which of the following is a correct interpretation of this finding? a. variable decelerations are due to umbilical cord compression b. variable decelerations are caused by uteroplacental insufficiency c. variable decelerations are a result of the administration of IV narcotic analgesics d. variable decelerations are related to fetal head compression

a

a nurse is assessing a client who received magnesium sulfate to treat preterm labor. which of the following clinical findings should the nurse identify as an indication of toxicity of magnesium sulfate therapy and report to the provider? a. respiratory depression b. facial flushing c. nausea d. drowsiness

a (Magnesium sulfate toxicity can cause life-threatening adverse effects, including respiratory and CNS depression. The nurse should report a respiratory rate slower than 12/min immediately to the provider and stop the infusion.)

A client, age 19, has an episiotomy to widen her birth canal. Delivery extends the incision into the anal sphincter. This complication is called:

a third-degree laceration.

Positive CST is normal or abnormal

abnormal The contraction stress test — also called a stress test or an oxytocin challenge test — may be done during pregnancy to measure the baby's heart rate during uterine contractions. Its purpose is to make sure the baby can get the oxygen he needs from the placenta during labor.

When caring for a client who's having her second baby, the nurse can anticipate the client's labor will be:

about half as long as her first labor.

DIC risk factors

abruptio placenta AFE missed abortion fetal death in utero Severe preeclampsia or eclampsia (GHTN) Septicemia cardiopulmonary arrest hemorrhage hydatiform mole

assessing client at 3 days PP and is breastfeeding. then nurse notes that the fundus is three fingerbreadths below the umbilicus, lochia rubra is moderate, and the breasts are hard and warm on palpation. interpretations of these findings?

additional intervention is not indicated at this time. the fundal location and lochia are characteristics w/in normal range. breast engorgement is typical, breast begin to produce milk. frequent BF and routine care can help relieve engorgement.

The third stage of labor ends:

after the birth of the placenta.

Intermittent uterine contratctions that may cause pain similar to menstrual cramps.

afterpain

nurse in prenatal clinic is caring for client, first trimester, client tells nurse she is upset, planned pregnancy, but having doubts and second thoughts. Appropriate response?

ambivalent feelings are quite common for women early in preg

The nurse assesses a client for evidence of postpartum hemorrhage during the third stage of labor. Early signs of this postpartum complication include:

an increased pulse rate, increased respiratory rate, and decreased blood pressure.

How is SVT treated?

analgesics and local application of heat. Elevate the legs

how is DVT treated?

analgesics, local application of heat, elevate legs, anticoagulant therapy (for up to 6 weeks after birth)

nurse is reviewing the hx of client who has new prescription of combined OC. nurse recognizes which medications can interfere with the effectiveness of OC?

anticonvulsants

nurse is caring for a client 2 days PP, is BF, and reports nipple soreness. Which measures would reduce discomfort during BF?

apply BM to nipples before feeding start on less sore nipple change infant position on nipple

For breast engorgement:

apply cool compresses b/t feedings apply warm compresses take warm shower b4 breast feeding

A client is to have an epidural block to relieve labor pain. The nurse anticipates that the anesthesiologist will inject the anesthetic agent into the:

area between the dura mater and the ligamentum flavum.

When do the breasts become lumpy and firm as blood flow increases and milk production begins?

around the 3rd day

A primigravid client is admitted to the labor and delivery area. Assessment reveals that she's in the early part of the first stage of labor. Her pain is likely to be most intense:

around the pelvic girdle.

When should the parents view and hold the infant after birth?

as soon as possible

The physician decides to artificially rupture the membranes. Following this procedure, the nurse checks the fetal heart tones to:

assess for prolapsed cord.

nurse is caring for client who is to undergo amniotomy, priority action??

assess the FHR pattern

nurse caring for client who is in 1st stage of labor, using pattern-paced breathing. client states she feels lightheaded, fingers are tingling. nurse should ?

assist client to breathe into a paper bag

nurse is caring for client at 40wks, in active labor, 6cm, 100%,, obtained pt BP 82/52. nursing intervention??

assist client to turn onto her side

A nurse is caring for a client who is postpartum and finds the fundus slightly boggy and displaced to the right. Based on these findings, which of the following actions should the nurse take?

assist the client to the bathroom to void

nurse is caring for client who experienced a vaginal delivery 12 hr ago. when palpating the clients abdomen, which position should the nurse expect to find the uterine fundus?

at the level of the umbilicus

an affectionate tie that occurs over time as infant and caregivers interact.

attachment

a nurse is caring for a client who is at 28 weeks of gestation and recieved terbutaline. which of the following findings should the nurse expect? a. FHR 100/min b. weakened uterine contractions c. enhanced production of fetal lung surfactant d. maternal blood glucose 63 mg/dL

b

a nurse is caring for a client who is in labor and assists the provider who performs an amniotomy. which of the following is the priority action by the nurse following the procedure? a. monitor the client's temperature b. assess the FHR c. assess the odor of the amniotic fluid d. provide clean, dry underpads

b

a nurse is admitting a client who is at 38 weeks of gestation and is in the first stage of labor. which of the following assessment findings should the nurse report to the provider first? a. expulsion of a blood-tinged mucous plug b. continuous contraction lasting 2 min c. pressure on the perineum causing the client to bear down d. expulsion of clear fluid from the vagina

b (A uterus contracting for more than 90 seconds is a sign of tetany and could lead to uterine rupture, which is the greatest risk to the client at this time. The nurse should report this finding immediately.)

nurse is leading discussion about contraception with group of 14 y.o clients. after presentation, client ask which method would be best for her to use. appropriate response?

before i can help you, i need to know more about your sexual activity

A nurse is reinforcing teaching about reducing perineal infection with a client following a vaginal delivery. Which of the following should the nurse include in the teaching?

blot the perineal area dry clean the perineal area from front to back perform hand hygiene before and after voiding wash the perineal area using a squeeze bottle of warm water after each voiding

a poorly contracted uterus is soft or?

boggy

a stron emotional tie that forms soon after birth between parent and the newborn.

bonding

What does BUBBLE-HE stand for?

breast uterus bladder bowel lochia episiotomy homans sign emotions (bonding)

A postpartum client requires teaching about breast-feeding. To prevent breast engorgement, the nurse should instruct her to:

breast-feed every 1½ to 3 hours.

lochia may increase when the mother?

breastfeeds

Normal lochial findings during the first 24 hours following delivery include:

bright red blood.

A nurse is monitoring a new mother for signs of postpartum depression. Which of the following, if noted in the new mother, would indicate the need for further data collection related to this form of depression?

The mother constantly complains of tiredness and fatigue. (Postpartum depression is not the normal depression that many new mothers experience from time to time. The woman experiencing depression shows less interest in her surroundings and a loss of her usual emotional response toward the family. The woman is also unable to show pleasure or love and may have intense feelings of unworthiness, guilt, and shame. The woman often expresses a sense of loss of self. Generalized fatigue, complaints of ill health, and difficulty in concentrating are also present. The mother would have little interest in food and would experience sleep disturbances.)

nurse is caring for a client who is primigravida at term, and having contractions, but stating that she is not really sure if she is in labor or not. nurse should recognize as a sign of true labor?

change in cervix

nurse caring for client , just delivered NB, following delivery, nurse action first to care for NB?

clear the Resp tract

A nurse is caring for a client who is postpartum and has a prescription for Rho (D) immunoglobulin. The nurse should verify which of the following prior to administration?

client is Rh negative and the newborn is Rh positive

cephalhematoma

collection of blood between periosteum and skull bone that it covers does not cross suture line results from trauma during birth

nurse is instructing woman, contemplating pregnancy about nutritional needs. to reduce risk of giving birth to NB w/ neural tube defect, include in teaching?

consume foods fortified with folic acid

Discontinue oxytocin if:

contraction frequency more often than every 2 mins contraction duration longer than 90 seconds no relaxation of uterus between contractions uterine resting tone greater than 20 mmHg between ctx

a nurse is preparing to assess a newborn who is postmature. findings to expect?

cracked peeling skin positive moro reflex

a nurse in the antepartum unit is caring for a client who is at 36 weeks of gestation and has pregnancy-induced HTN. suddenly, the client reports continuous abdominal pain and vaginal bleeding. the nurse should suspect which of the following complications? a. placenta previa b. prolapsed cord c. incompetent cervix d. abruptio placentae

d

a nurse is observing the EFHR monitor tracing for a client who is at 40 weeks of gestation and is in labor. the nurse should suspect a problem with the umbilical cord when she observes which of the following patterns? a. early decelerations b. accelerations c. late decelerations d. variable decelerations

d

a nurse is caring to a client who is in labor and has an epidural anesthesia block. the client's blood pressure is 80/40 mm Hg and the fetal heart rate is 140/min. which of the following is the priority nursing action? a. elevate the client's legs b. monitor vital signs every 5 min c. notify the provider d. place the client in a lateral position

d (Based on Maslow's hierarchy of needs, the client should be moved to a lateral position or a pillow placed under one of the client's hips to relieve pressure on the inferior vena cava and improve the blood pressure.) (a-The nurse should elevate the client's legs if there is no improvement in the blood pressure with the client in a lateral position, but this is not the priority nursing action.) (b-The client's vital signs should be monitored every 5 min, but this is not the priority nursing action.) (c-The provider should be notified, but this is not the priority nursing action.)

a nurse is caring for a client who is at 39 weeks of gestation and is in active labor. the nurse locates the fetal heart tones above the client's umbilicus at midline. the nurse should suspect that the fetus is in which of the following positions? a. cephalic b. transverse c. posterior d. frank breech

d (With a frank breech presentation, the fetal heart is generally above the level of the client's umbilicus.) (a - With a cephalic presentation, the fetal heart is generally below the level of the client's umbilicus.) (b-With a transverse presentation, the fetal heart is generally below the level of the client's umbilicus.) (c-With a posterior presentation, the fetal heart is generally below the level of the client's umbilicus.)

a nurse is caring for a client who is at 40 weeks gestation and is in active labor. the client has 6 cm of cervical dilation and 100% cervical effacement. the nurse obtains the client's BP reading as 82/52 mm Hg. which of the following nursing interventions should the nurse perform? a. prepare for a cesarean birth b. assist the client to an upright position c. prepare for an immediate vaginal delivery d. assist the client to turn onto her side

d () (a-Unless late decelerations are noted during fetal monitoring, there is no need to prepare for a cesarean birth.) (b-Placing the client in an upright position is unlikely to improve her blood pressure significantly.) (c-Unless the fetus is exhibiting changes during fetal monitoring, indicating distress, there is no need to hasten delivery.)

a nurse is caring for a client who is in labor at 40 weeks of gestation and reports that she has saturated two perineal pads in the past 30 min. the nurse caring for her suspects placenta previa. which of the following is an appropriate nursing action? a. examination to determine cervical status b. a magnesium sulfate infusion c. initiation of pushing d. preperation for cesarean birth

d (A cesarean birth is indicated for all clients who have a confirmed placenta previa.) (a-Vaginal exams are contraindicated in the presence of a placenta previa.) (b-Magnesium sulfate infusions are indicated for the treatment of preterm labor or the prevention of seizures in the preeclamptic client. The therapeutic action is smooth muscle relaxation.) (c-All clients with a confirmed placenta previa must deliver via cesarean section.)

nurse is caring for client, first trimester, ask nurse if she can continue to exercise during pregnancy. Appropriate response?

daily jogging for 30/min/day is fine throughout preg

clot dissolving factors _______ during pregnancy

decrease

involve veins in the feet and femoral area, characterized by pain, calf tenderness, leg edema, color changes, pain when walking and sometimes postive homans sign.

deep venous thrombosis (DVT)

A client asks the nurse about the rhythm (calendar-basal body temperature) method of family planning. The nurse explains that this method involves:

determination of the fertile period to identify safe times for sexual intercourse.

The nurse is teaching a client about oral contraceptive therapy. If a client misses three or more pills in a row, the nurse should instruct her to:

discard the pack, use an alternative contraceptive method until her menses begins, and start a new pack on the regular schedule.

What should you assess the bladder for?

distension

Which physiologic response should the nurse expect during the early postpartum period?

diuresis

A nurse is caring for a client who had a vaginal delivery 2 hr ago. Which of the following actions should the nurse anticipate in the care of this client?

document fundal height observe the lochia during palpation of fundus determine whether the fundus is midline administer methylergonovine maleate if uterus is boggy

nurse is completing discharge teaching to a client , 35th wek, has mild preeclampsia. information about nutrition should be in teaching?

drink 48 to 64 oz of water daily

a nurse is assessing a NB at 42.5 wks, expect findings?

dry cracked skin

nurse cairng for NB immediately after birth, after assuring patent airway, what is priority??

dry the skin

nurse is caring for client who is premature labor, recieving terbutaline, nurse should monitor client for what A/E to report to provider?

dyspnea - pulmonary edema

a nurse is instructing a female client about check BASAL TEMP to determine client is ovulating. nurse instruct client to check her temp at what time?

early in the morning before arising

A nurse is caring for a client who is 7 days postpartum and calls the clinic to report pain and redness of her left calf. Besides seeing her provider, which of the following interventions should the nurse suggest?

elevate her leg

Before administering pitocin, where should the fetus be?

engaged in the birth canal at a minimum of 0 station.

an incision made to enlarge the vaginal opening

episiotomy

Teach the clent to avoid sexual intercourse until:

episiotomy/laceration is healed & vaginal discharge has turned white

Newborns who are formula fed should be burped:

every 15-30 mLs

saturation of perineal pad in 15 minutes with lochia is?

excessive

nurse in prenatal clinic, caring for client suspected of having a hydatidform mole. findings to expect?

excessive uterine enlargement

nurse is caring for client who is positive pregnancy test, nurse is teaching client about common discomfort in 1st trimester of pregnancy and warning signs danger. instruct client to call clinic if she experiences manifestations?

facial edema - HTN or preeclampsia

nurse is caring for client who is to undergo, BPP, the client asks nurse what is being evaluated, nurse should include?

fetal breathing fetal motion aminiotic fluid

nurse on L&D unit, client having difficult, prolonged labor with severe backache, contributing causes should the nurse ID?

fetal position is persistent Occiput Posterior

telangectiatic nevi

flat pink or red marks that easily blanch and are found on the newborns back of the neck, nose, upper eye lids, and middle of the forehead

What can the nurse suggest to help with constipation?

fluids fiber ambulate stool softener

nurse providing preconception counseling for pt who is planning preg, which supplement should the nurse recommend to prevent neural tube defect in fetus?

folic acid

nurse is caring for client who is 39 weeks and is in active labor, nurse locates the fetal heart tones above the umbilicus at midline. should suspect fetus is in which position?

frank breech

What are the 3 signs of subinvolution?

fundal height greater than expected persistence of lochia rubra pelvic pain, heaviness, fatigue

The upper portion of the body of the uterus

fundus

Third stage: Signs of placental separation from the uterus are indicated by:

fundus firmly contracting swift gush from introitus of dark blood umbilical cord appears to lengthen as placenta descends vaginal fullness of exam

nurse is caring for a client who is PP, recieved methylergonovine. findings indicate the medication was effective??

fundus is firm to palpate

a nurse assessing a client who is 8hr PP and multiparous. Which of the following findings should alert the nurse to the client's need to urinate?

fundus is three fingerbreadths above the umbilicus

A nurse is assessing a client who is 8 hr postpartum and multiparous. Which of the following findings should alert the nurse to the client's need to urinate?

fundus three finger breadths above the umbilicus

teaching a newborn parent to care for the umbilical cord stump. include which instructions?

give sponge bath till cord falls off

About what size is the uterus after birth?

grapefruit

A nurse is planning care for a client who is 2 hrs postpartum following a cesarean birth. The client has a history of thromboembolic disease. Which of the following nursing interventions should be included in the plan of care?

have the client ambulate

caring for newborn, grandmother of the newborn asks if she can take the baby to the mothers room. Appropriate response?

have the mother call and I will take the baby to the room.

A nurse is caring for a client who is 12 hr postpartum. Which of the following findings should alert the nurse to the possibility of a postpartum complication?

heart rate 110/min

A client who delivered her first child 6 weeks ago seems overwhelmed by her new role as a mother. She tells the nurse, "I can't keep up with my housework any more because I spend so much time caring for the baby." The nurse should:

help the client break down large tasks into smaller ones.

after birth, fluid shifts into the blood stream and lowers the _________.

hematocrit

nurse is a college health clinic is speaking to group of adolescents about TSS, include in teaching about increasing risk of contracting TSS?

high absorbing tampons

A nurse in a clinic is assessing a client who is 8 wks gest and has HG. findings to expect?

hx of migraines - first 20 wks nulliparous - first trimester twin gestation - increasing levels of estrogen, progesterone, HCG

A client with moderate pregnancy-induced hypertension (PIH) is a poor candidate for regional anesthesia during labor and delivery. If she were to receive this form of anesthesia, she might experience:

hypotension.

Two complications that can occur following PP hemorrhage include:

hypovolemic shock anemia

a nurse is caring for a client who is PP. nurse should recognize the following as indication of inhibition of parenteral attachment?

i wish he had more hair, i will keep a hat on his head till he grow some

home heatlh nurse is teaching client, BF managing breast engorement. indicates understanding of teaching?

i'll feed the baby every 2 hrs

What can be applied to the perineal area in the first 12-24 hours after birth?

ice packs

When can the uterine fundus be felt?

immediately after the placenta is expelled

A nurse is caring for a client who is postpartum who asks the nurse when her breast milk will "come in". Which of the following responses should the nurse make?

in 3-5 days

nurse caring for client, PP, asks when her BM will come on, appropriate response?

in 3-5 days

clotting factors ______ during pregnancy

increase

nurse is prenatal clinic caring for client, 12 wks, asks about cause of her heartburn, nurse response?

increased progesterone production causes decreased motility of smooth muscle

Subinvolution s&s

increased vag bleeding uterus enlarged and higher than normal in the abd relative to umbilicus boggy uterus prolonged lochia d/c with irregular or excessive bleeding

What does a high pulse rate often indicate postpartum?

infection or hypovolemia

patient at 38 weeks, large amount of painless bright red vaginal bleeding. placed on fetal monitor; indicating FHR of 138/min, no uterine contractions. client vital signs are BP: 98/52, HR 118/min, RR 24/min, Temp 36.4 - 97.6,. priority action?

initiate IV access - large-bore IV catheter client is losing blood rapidly, HYPOTENSION - TACHYCARDIA - NEED fluids and blood to be administered IF HYPOVOLEMIA develops

First stage pain:

internal visceral may be felt as back/leg pain

The changes that the reproductive organs, particularly the uterus undergo after birth to return to their prepregnancy size and condition.

involution

Indomethacin

is an NSAID that suppresses preterm labor & uterine contraction.

nurse in prenatal clinic, caring for client at 7 weeks. client reports urinary frequency, asks if this will continue after delivery. appropriate response?

it occurs during the first trimester and near the end of preg

nurse is teaching client PP and has new RX for injection of RHOGAM, what should be included in teaching?

it prevents formation of RH antibodies in mothers who are RH negative given at 28week and following future preg

why is it important to massage a boggy uterus until firm?

it prevents hemorrhage

larger than 15cm stain or one pad saturated within 2 hours is?

large, heavy

What are 2 CAM therapy's for postpartum depression?

light therapy exercise

nurse in prenatal clinic, completing skin assessment for pt in 2nd trimester. expected findings?

linea nigra chloasma striae gravidarum

caput succadaneum

localized swelling of the soft tissues of the scalp caused by pressure on the head during labor.........normal finding......crosses suture line

nurse in prenatal clinic, caring for client at 38wks, reports heavy red vaginal bleeding. the bleeding started spontaneously in the morning, not accompanied by contractions. client is not in distressed, she states she can feel the baby moving. an ultrasound is scheduled STAT. explain to client the purpose of ultrasound to determine?

location of placenta - placenta previa

vaginal discharge after delivery, composed of endometrial tissue, blood and lymph

lochia

mostly mucous lochia that is clear and colorless or white

lochia alba

discharge that is composed mostly of blood, lasts 3 days after birth

lochia rubra

One day after a client gives birth, the nurse performs a postpartum assessment. At this time, the nurse expects to find:

lochia rubra.

discharge that is pinkish from blood and mucous, 3-10th day

lochia serosa

After 2 days of breast-feeding, a postpartum client reports nipple soreness. To relieve her discomfort, the nurse should suggest that she:

lubricate her nipples with expressed milk before feedings.

What do you do if the uterus is soft after birth?

massage it to make it firum

nurse caring for pt in labor, which of the following reflects application of the gate control theory of pain?

massage the clients back theory is based on concept of blocking or preventing the transmission of pain signals to the brain by using distraction techniques

If the uterus is soft (boggy) and the bladder is full, what do you do first?

massage the uterus to firmness, then empty the bladder.

During labor, a client tells the nurse that her last baby "came out really fast." The nurse can help control a precipitous delivery by:

massaging and supporting the perineum.

Cold cabbage leaves

may also be applied to breasts to decrease swelling and relive discomfort

What causes constipation after birth?

medications stretched abdominal muscles can't bear down soreness and swelling dehydration and little food intake

less than 15cm stain of lochia is?

moderate

APGAR or 4-6 indicates

moderate distress

Cord infection s&s:

moist and red foul odor purulent drainage

nurse is planning care for a NB who is SGA, which is priority intervention to include in care?

monitor blood glucose

a pervasive and sustained emotion that can color one's view of life.

mood

Post partum hemorrhage is considered to occur if the client loses how much blood?

more than 500 mL after vaginal birth more than 1000 mL after c/s

non-stress test ( NST)

most widely used technique for antepartum evaluation of fetal well being performed during the third trimester.

afterpains occur more often in ________ or in women whose uterus was overly distended.

multiparas

nurse caring for NB, who has myleomeningocele, nurse goal priority in care for NB?

myelomeningocele is a congenital d/o that causes the spine and spinal canal to not close prior to birth, which results in the spinal cord, meninges, and nerve roots, protruding out of the childs back, in the fluid filled sac. before surgery NB must be held carefully to reduce damange to the exposed spinal cord and....maintain integrity of the sac

nurse is completing admission assessment of a newborn, following anatomical landmarks, the nurse can use which to measure NB chest circumference??

nipple line

risk factors for post partum depression:

no social support poor relationship with partner stress low self esteem unplanned pregnancy

nurse is caring for a client, 40 wks and in labor. ultrasound examination indicates that fetus is SGA. intervention to include for NB care?

observe for meconium in Respiratory secretions SGA= increase risk of INTRAUTERINE HYPOXIA due to the presence of meconium in aminiotic fluid at risk for perinatal asphyxia due to stress of labor and depressed

What should the nurse do to help parents bond?

observe parenting behaviors model appropriate behavior for the parent

nurse caring for pt, 6 wks, first preg, asks nurse when she can expect to exp quickening, appropriate response?

occur between the fourth and fifth month

nurse is caring for client who is having a NST performed. the FHR is 130-150/min, but there is no fetal movement for 15 min. nurse action?

offer a snack of OJ and crackers, baby is probably sleeping. turning to lateral side, DOES NOT indicate FHR is an issue

Inversion of uterus s&s

pain in lower abd vag bleeding dizziness low BP pallor

a nurse is caring for client, 36 wk, suspected of placenta previa, findings support DX??

painless red vaginal bleeding

A nurse is assessing a client for postpartum infection. Which of the following findings should indicate to the nurse that the client requires further evaluation for endometritis?

pelvic pain

erythema toxicum

pink rash that appears suddenly anywhere on the body of a term newborn during the first 3 weeks.

a nurse in L&D is caring for client undergoing EFM, nurse observes the FHR begins to slow after the start of contraction. the lowest rate occurs after the peak of the contraction. first action by nurse?

place the client in the lateral position, LATE DECEL, fetal hypoxemia due to insufficient placental perfusion.

nurse in L&D, admitting a client who reports painful contractions, nurse determine contraction duration of 1min and frequency of 3 min. the nurse obtains VS, FHR - 130/min, MHR 128/min, MBP: 92/54. PRIORITY action

position pt with one hip elevated - client needs adequate BP to perfuse herself and her fetus BP 92/54

feelings experienced by new mothers. Conflicting feelings of joy and emotional letdown during the first few weeks after birth.

postpartum blues

a persisten mood of unhappiness after birth. It is not normal.

postpartum depression

blood clotting factors are higher during ______ and _________.

pregnancy, puerperium

nurse is prenatal clinic is caring for client, believes she might be pregnant, she feels the baby moving, appropriate statement?

presumptive sign

nurse is caring for a preterm newborn who is in an incubator to maintain neural thermal environment. the father of the NB asks the nurse why is this necessary. response?

preterm NB lack adequate temperature control mechanisms

When administering magnesium sulfate to a client with preeclampsia, the nurse understands that this drug is given to:

prevent seizures.

Dystocia

prolonged, difficult labor

Methods of induction

prostaglandins applied cervically admin of IV oxytocin amniotomy stripping of membranes nipple stimulation

involves serious impairment of one's perception of reality

psychosis

The postpartum period

puerperium

what is the fourth trimester of pregnancy?

puerperium (postpartum)

A nurse is caring for a client who is 1 days postpartum and is taking a sitz bath. To determine the client's tolerance of the procedure, which of the following assessments should the nurse perform?

pulse rate

Why does a full bladder interfere with uterine contractions?

pushes the fundus us and causes it to deviate to one side, usually the right side

If a uterus is soft, what must you avoid doing?

pushing down on it (this could invert it)

How often do you take vitals for the first 24 hours after birth?

q4h

nurse is admitting pt who has severe preeclampsia, 35 wks, review providers orders, which needs clarification?

question order for ambulate 2x daily

nurse is caring for pt, BF, asks the nurse about changes to make in her diet. dietary changes nurse suggest?

reduce intake of IRON

A client is experiencing true labor when her contraction pattern shows:

regular contractions that increase in frequency and duration.

nurse is caring for an antepartum client, lab findings indicate rubella titer. correct interpretation of this data?

requires a rubella immunization following delivery

Third stage pain:

similar to first stage pain

nurse is preparing to measure fundal height of client who is 22 wks, at which location should the nurse palpate the fundus??

slightly above umbilicus

Milia

small raised white spots on nose, chin, and forehead

What are the breasts like the first 2 or 3 days after birth?

soft and full

nurse in prenatal clinic is teaching client, new prescription for dinoprostone gel. nurse include in the teaching?

softens cervix stimulates uterine muscles

Second stage pain:

somatic occurs with fetal descent and expulsion

How do you clean the perineum after voiding?

squirt water on it from peribottle and pat dry

During the first formula feeding, a client has difficulty getting her neonate to take the artificial nipple into his mouth. To resolve this problem, the nurse should suggest that the mother:

stroke the neonate's lips gently with the nipple.

The failure of the uterus to return to the prepregnant state after 6 weeks?

subinvolution

A client says she wants to practice natural family planning. The nurse teaches her how to use the calendar method to determine when she's fertile and advises her to avoid unprotected intercourse. When teaching her how to determine her fertile period, the nurse should instruct her to:

subtract 18 days from her shortest menstrual cycle and 11 days from her longest cycle.

While talking to the nurse, the parents of a neonate in the neonatal intensive care unit (NICU) express concern that they're neglecting their 3-year-old son. The nurse suggests a sibling visit. To best promote the sibling's attachment to the neonate, the nurse should:

suggest that the sibling bring in a drawing to display near the neonate's crib in the NICU.

What kind of bra should a woman wear postpartum?

support bra

What are the hallmark signs of PE?

tachypnea dyspnea

What are Rubin's 3 stages of psychological changes during the puerperium?

taking in taking hold letting go

nurse is caring for client who has rubella at the time of delivery and ask why her NB is being placed in isolation. response?

the NB might be actively shedding the virus

The nurse should tell new mothers who are breast-feeding that breast milk is produced when:

the placenta is delivered, causing the secretion of prolactin.

nurse on L& D unit is caring for client following vaginal exam, by provider which documents, -1, interpret?

the present part is 1 cm above ischial spine

a nurse caring for a client who delivered a healthy NB via C-section birth. client asks the nurse, is there a chance that i could deliver my next baby w/out have a c-section. best response?

the primary consideration is what type of incision was performed this time.

nurse is caring for client in prenatal visit, BMI is 26.5, client ask how much weight she should gain during preg. appropriate response?

the recommendation for you is 15-25lbs BMI <18.5 - UNDERWT. 18 - 24.9 - NORMAL 25 - 29.9 OVERWT. >30 - OBESE

Why do women who have had a c-section have less lochia?

the uterine cavity was sponged at delivery

a nurse is providing teaching about KEGELs to group of clients who are in third trimester. indicates understanding of teaching?

these exercises help pelvic muscles to stretch during birth

After what stage of labor is there a fall in the blood levels of placental hormones, human placental lactogen, HCG, estrogen and progesterone?

third stage

nurse in community clinic, counseling client who received positive test result for chlamydia. appropriate statement?

this infection is treated with one dose of azithromycin

nurse is completing home visit to mother, 3 days PP, BF, mother expresses concern the amount of weight the newborn lost since birth. response by nurse?

this might be related to your baby having 3 stools a day. the NB is being BF typically has 3 or more stools per day during the first few weeks.

nurse is caring for a new mother, concerned her NBs eyes cross. therapeutic response?

this occurs because NBs lack muscle control to regulate eye movement

A client is taking a progestin-only oral contraceptive, or minipill. Progestin use may increase the client's risk of:

tubal or ectopic pregnancy.

nurse in labor and delivery, pt following delivery of placenta, the nurse examines the umbilical cord, expected findings to observe on cord?

two ARTERIES one VEIN

Why is it important to check to see if the fundus if firm?

uncontracted uterus allows blood to flow freely from vessels at the placenta insertion site (a contracted uterus helps with hemostasis)

Postpartum disorders are:

unexpected events or occurrences that may happen during the PP period

A nurse is caring for a client who has a suspected ectopic pregnancy at 8 weeks gest. Which of the following manifestations should the RN ID as consistent with the diagnosis

unilateral, cramp-like abdominal pain

A primigravid client, age 20, has just completed a difficult, forceps-assisted delivery of twins. Her labor was unusually long and required oxytocin (Pitocin) augmentation. The nurse who's caring for her should stay alert for:

uterine atony.

During the fourth stage of labor, the client should be assessed carefully for:

uterine atony.

nurse in the ambulatory surgery center is providing discharge teaching to client who had a D&C, following a spontenous miscarriage. include in the teaching?

vaginal intercourse can be resumed after 2 weeks

nurse is caring for client, 1st stage of labor, undergoing external FHR and recieving IV fluids. nurse observes variable decel in the FHR on the monitor strip. correct interpretation ?

variable decel is due to umbilical cord compression

What 3 situations increase a woman's risk for blood clot formation?

varicose veins c-section delayed ambulation

nurse caring for client beginning BF after delivery, new mother states, i don't want to take anything for pain because i am BF, appropriate response?

we can time your medication so that you have an hour or two before next feeding

nurse is caring for a client PP, client tells nurse that the NB maternal grandma was born deaf, and asks how to tell if the NB hears well. correct statement to make?

we do routine hearing screenings on NBS you'll know the results before you leave the hospital

When is REEDA used?

when checking the perineal area

When is the uterine lining shed?q

when the placenta detaches

How should nipples be washed after birth?

with plain water only to avoid them drying out and cracking

Why are uti's a problem after birth?

women's bladders fill up quickly to rid excess body fluid, but the tone of the bladder and ureters is weak and leaves urine in the bladder after voiding.

Is redness normal around the perinium?

yes, but pain is not. Pain could indicate infection

nurse is caring for client who is in preterm labor at 32 weeks . The client asks will my baby be okay? appropriate response?

you must be feeling scared and powerless

nurse caring for adolescent PT who has PID as consequence of sexually transmitted infection, will need IV antibiotic therapy. the client tells the nurse, my parents think i am a virgin, I don't think i can tell them i have this kind of infection. appropriate response?

you seemed scared to talk to your parents

a nurse in the ED is admitting a pt who is 40 wks gest, has ruptured membranes, nurse observes NB head crowning. Client tells nurse she wants to push. Correct reply?

you should try to pant as the delivery proceeds panting allows uterine forces to expel the fetus and permits controlled muscle expansion to avoid rapid expulsion of fetal head

nurse providing teaching to client about nutrition, at first prenatal visit. include in teaching?

you will need to double your intake of iron during pregnancy

nurse is completing discharge instructions for new mother and 2 day old NB. mother asks, how will i know if my NB gets enough BF? appropriate response?

your baby should have 6-9 wet diapers a day

When does the fundus start to descend after birth?

24 hours later

The recommended weight gain during pregnancy is usually:

25-35lbs 3-4 lb in first trimester 1 lb per week in the last two trimesters

Normal newborn weight range:

2500-4000 grams

A client is using the rhythm (calendar-basal body temperature) method of family planning. In this method, the unsafe period for sexual intercourse is indicated by:

3 days of elevated basal body temperature and clear, thin cervical mucus.

Active phase of labor:

4-7 cm moderate to strong contractions regular q 3-5 min lasts 40-70 sec

A client in labor is prescribed 1,000 ml of normal saline solution to infuse over 6 hours. The drip factor of the I.V. administration set is 15 drops/ml. The nurse on the previous shift hung the solution at the end of her shift. The oncoming nurse should recheck the drip rate to make sure that the I.V. is infusing at the prescribed drip rate. What is the prescribed drip rate?

42 drops/minute

How much blood is lost during natural birth?

500 mL

When do fluctuating hormone have their peak affect on a woman's mood?

5th day

How long does the postpartum phase last?

6 weeks

Moderate variability:

6-25/min

How long does it take the placental site to fully heal?

6-7 weeks

Newborns should have how many diapers a day?

6-8 wet diapers 3-4 stools per day

Newborn BP should be

60-80 systolic and 40-50 diastolic

It is recommended that _____ mcg of folic acid be taken during pregnancy.

600

The uterus returns to the pelvic cavity in which of the following time frames?

7 to 9 days postpartum

Transition phase of labor

8-10 cm strong to very strong contractions q 2-3 min lasts 45-90 sec

BPP abnormal score is

<4

Minimal variability:

> undetectable but <5/min

Marked variability:

>25/min

A postpartum client asks a nurse when she can resume sexual activity. The appropriate nursing response is which of the following?

"Sexual activity can be resumed in about 3 weeks once the episiotomy has healed and the lochia has stopped." (It is recommended that the woman refrain from sexual intercourse until the episiotomy has healed and the lochia has stopped. This process usually takes about 3 weeks. Options 1, 2, and 4 are inaccurate.)

A nurse attempts to encourage a new mother to understand and to accept the cesarean section that was necessary to deliver her baby, rather than to focus on the surgical aspect of the procedure. Which nursing statement would provide the best encouragement?

"Tell me about the delivery of your baby." (It is important for the mother to think of the procedure as the birth of the baby. The mother may become disappointed because she was unable to deliver vaginally, complicating the postpartum phase. Option 2 brings the surgery to focus and can inhibit the mother from bonding with the neonate. Options 3 and 4 place the focus on the future, and the mother needs to focus on the birth of the baby.)

The pregnant client who is anemic tells the nurse that she is concerned about her baby's condition following delivery. The nurse makes which statement that will best address the client's concern?

"The effects of anemia on your baby are difficult to predict, but let's review your plan of care to ensure you are providing the best nutrition and growth potential." (The effects of maternal iron deficiency anemia on the developing fetus and neonate are unclear. In general, it is believed that the fetus will receive adequate maternal stores of iron, even if a deficiency is present. Neonates of severely anemic mothers have been reported to experience reduced red cell volume, hemoglobin, and iron stores. Options 1 and 3 provide false reassurance to the client. Option 2 will cause further concern in the client. Option 4 provides the most realistic support for the client and allows the nurse an opportunity to review the client's plan of care to clarify information and reassure the mother.)

A client in the early stages of labor asks the nurse whether it's really necessary for her to purchase a car seat, noting that they're very expensive. Which response by the nurse is best?

"The only way to safely transport your baby in a car is to have him restrained securely in a car seat."

A nurse in a prenatal clinic is teaching a group of clients about nutrition requirements during lactation. Which of the following statements should the nurse make?

"Zinc intake should be at least 12 mg per day"

(Select all that apply) A nurse is assigned to assist with the admission of a client who is in labor. Which of the following actions are appropriate?

(1) Asking about the estimated date of delivery (EDD), (3) Taking maternal and fetal vital signs, (6) Asking about the amount of time between contractions

(SELECT ALL THAT APPLY) The nurse is assisting in the delivery room. The physician performs an episiotomy, an incision in the client's perineum to enlarge the vaginal opening and facilitate delivery. Which interventions should the nurse perform when caring for the client after this procedure?

(1) Check the episiotomy repair site., (2) Apply ice to the perineum, (4) Administer pain medication, as prescribed., (5) Explain perineal care to the client when she is able to focus on the instructions.

(SELECT ALL THAT APPLY) The nurse is assisting in monitoring a client who is receiving oxytocin (Pitocin) to induce labor. The nurse should be alert to which of the following maternal adverse reactions?

(1) Hypertension, (4) Fluid Overload, (5) Uterine Tetany

(SELECT ALL THAT APPLY) A client receives an epidural block for pain relief during labor. Which interventions by the nurse are important when caring for a client with an epidural block?

(1) Make sure oxygen is available., (3) Monitor vital signs frequently., (5) Monitor fetal heart rate and contractions closely.

(SELECT ALL THAT APPLY) The nurse observes several interactions between a mother and her new son. Which of the following behaviors by the mother would the nurse identify as evidence of mother-neonate attachment?

(1) Talks and coos to her son, (2) Cuddles her son close to her

When can heat be applied to the perineal area?

24 hours after birth

Choose the safety measures that should be implemented when working in the newborn nursery. Select all that apply.

1. Adhere to standard precautions. 5. The parents should be instructed to not release their infant to anyone wearing improper identification. 6. The mother should be fingerprinted and the infant should be footprinted on the identification card before removing the infant from the delivery room. (Newborn safety, infection prevention, and abduction prevention are a major responsibility for nurses working in the newborn nursery. Standard precaution guidelines need to be followed to prevent transmission of bacteria and other illnesses to newborns. Following safety precautions to prevent newborn abduction includes footprinting the newborn along with fingerprinting of the mother on the identification card. This also includes placing bracelet identification on the mother and infant prior to removing the newborn from the delivery room. Educating parents to release their newborn only to those wearing proper identification is key in preventing newborn abductions in the inpatient situation. Bassinets are to be 3 feet apart. Nurses who are ill should not be working in the nursery.)

A nurse is assisting in developing a plan of care for a postpartum client who was diagnosed with superficial venous thrombosis. The nurse anticipates that which of the following interventions should be included in the plan of care? Select all that apply.

1. Maintaining bed rest 2. Elevating the affected extremity 5. Applying warm compresses to the affected area as prescribed (Thrombosis that is limited to the superficial veins of the saphenous system is treated with analgesics, rest, and elastic support stockings. Elevation of the lower extremity improves venous return and may be recommended. Warm packs may be applied to the affected area to promote healing. Anticoagulants or anti-inflammatory agents are not needed unless the condition persists. After 5 to 7 days of bed rest, and when symptoms disappear, the woman may gradually begin to ambulate.)

A nurse in the newborn nursery is collecting data on a neonate who was born of a mother addicted to cocaine. Which of the following would the nurse expect to note in the neonate? Select all that apply.

1. Tremors 3. Irritability 4. Hypertension 6. Exaggerated startle reflex (Clinical symptoms at birth in neonates exposed to cocaine in utero include tremors, tachycardia, marked irritability, muscular rigidity, hypertension, and exaggerated startle reflex. These infants are difficult to console and exhibit an inability to respond to voices or environmental stimuli. They are often poor feeders and have episodes of diarrhea.)

The nurse is preparing a list of self-care instructions for a postpartum client who has been diagnosed with mastitis. Choose the instructions that would be included on the list. Select all that apply.

1. Wear a supportive non-underwire bra. 2. Rest during the acute phase. 3. Maintain a fluid intake of at least 3000 mL. 4. Continue to breast-feed if the breasts are not too sore. (Mastitis is an infection of the lactating breast. Client instructions include resting during the acute phase, maintaining a fluid intake of at least 3000 mL per day, and taking analgesics to relieve discomfort. Antibiotics may be prescribed and are taken until the complete prescribed course is finished. They are not stopped when the soreness subsides. Additional supportive measures include the use of moist heat or ice packs and the wearing of a supportive bra. Continued decompression of the breast by breast-feeding or breast pump is important to empty the breast and prevent the formation of an abscess.)

FHR can be heard by Doppler at:

10-12 weeks GA

The nurse is checking the fetal heart rate (FHR) of a client admitted to the labor and delivery area at term. Which of the following should the nurse identify as the normal range of the baseline FHR?

120 to 160 beats/minute

MSAFP screening done when?

15-22 weeks

MSAFP screening is done:

15-22 weeks of gestation maternal serum alpha-fetoprotein (MSAFP) screen. It's usually as part of a set of tests, which screen for genetic problems, called the quad screen. AFP is a substance made in the liver of an unborn baby (fetus).

AFP can be measured from the amniotic fluid between:

16 and 18 weeks alpha-fetoprotein

After receiving methylergonovine (Methergine) I.M. for postpartum hemorrhage, a client is prescribed methylergonovine 0.4 mg by mouth every 6 hours. The pharmacy sends 0.2 mg tablets. How many tablets must the nurse administer with each dose?

2

Which safety measures should be implemented at delivery and when working in the newborn nursery? Select all that apply.

2. Adhere to standard precautions during delivery and in the nursery. 4. Instruct the parents to not release their newborn infant to anyone wearing improper identification. 5. Fingerprint the mother and footprint the infant on the identification card prior to removing the infant from the delivery room.

A nulliparous client has been in the latent phase of the first stage of labor for several hours. Despite continued uterine contractions, her cervix hasn't dilated further since the initial examination. Her latent phase may be considered prolonged after:

20 hours.

A nurse at a prenatal clinic is caring for a client who is in her first trimester of pregnancy. The client tells the nurse that she is upset because, although she and her husband planned this pregnancy, she has been having many doubts and second thoughts about the upcoming changes in her life. Which of the following is an appropriate response by the nurse? A. "Ambivalent feelings are quite common for women early in pregnancy." B. "Perhaps you should see a counselor to discuss these feelings further." C. "Have you spoken to your mother about these feelings?" D. "Don't worry. You'll be fine once the baby is born."

A. "Ambivalent feelings are quite common for women early in pregnancy." i: This response uses the therapeutic communication technique of providing information while addressing the client's concerns and feelings. This statement is true and gives the client the information she needs; many antepartum women experience similar feelings in early pregnancy.

The goal for the postpartum client with thrombophlebitis is to prevent the complication of pulmonary embolism. In planning care to assist in meeting this goal, the nurse should:

Administer anticoagulants as prescribed. (The purpose of anticoagulant therapy is to prevent the clot from moving to another area.)

A nurse is monitoring a client at risk for postpartum endometritis. Which observation noted during the first 24 hours after delivery may support this diagnosis?

Abdominal tenderness and chills (Symptoms in the postpartum period heralding endometritis include delayed uterine involution, foul-smelling lochia, tachycardia, abdominal tenderness, and temperature elevations up to 104° F. This intrauterine infection may lead to further maternal complications such as infections of the fallopian tubes, ovaries, and blood (sepsis). Options 1, 2, and 4 represent normal maternal physiological responses in the immediate postpartum period. These changes represent the normal adaptation of reproductive organs (involution) and maternal physiological responses because of the decreased hormonal levels and fluid losses that occur during labor.)

A postpartum client is at high risk for infection. A goal has been developed that states, "The client will not develop an infection during her hospital stay." Which of the following data would support that the goal has been met?

Absence of fever (Fever is the first indication of an infection. An absence of fever would indicate that the goal stated in the question has been met. Chills, abdominal tenderness, and loss of appetite indicate the presence of an infection.)

A nurse is assessing a client who is 3 days postpartum and is breastfeeding. The nurse notes that the fundus is three finger breadths below the umbilicus, lochia rubra is moderate, and the breasts are hard and warm to palpation. Which of the following interpretations of these findings should the nurse make?

Additional interventions are not indicated at this time

The nurse is administering magnesium sulfate to a client for preeclampsia at 34 weeks' gestation. What is the priority nursing action for this client?

Assess for signs and symptoms of labor.

A nurse in a clinic is caring for a client who is at 39 weeks of gestation and who asks about the signs that precede the onset of labor. Which of the following should the nurse identify as a sign that precedes labor? A. Decreased vaginal discharge B. A surge of energy C. Urinary retention D. Weight gain of 0.5 to 1.5 kg

B. A surge of energy i: Prior to the onset of labor, the pregnant client experiences a surge of energy.

A nurse is caring for a client who is in labor and assists the provider who performs an amniotomy. Which of the following is the priority action by the nurse following the procedure? A. Monitor the client's temperature. B. Assess the fetal heart rate. C. Assess the odor of the amniotic fluid. D. Provide clean, dry underpads.

B. Assess the fetal heart rate. i: The fetal heart rate should be assessed before and immediately after the amniotomy to detect any changes.

A nurse is caring for a client who is in the first stage of labor and is using pattern-paced breathing. The client says she feels lightheaded and her fingers are tingling. Which of the following actions should the nurse take? A. Administer oxygen via nasal cannula. B. Assist the client to breathe into a paper bag. C. Have the client tuck her chin to her chest. D. Instruct the client to increase her respiratory rate to more than 42 breaths per min.

B. Assist the client to breathe into a paper bag. i: This client is experiencing respiratory alkalosis due to hyperventilation. The client should be assisted to breathe into a paper bag or to cup her hands over her mouth to increase the carbon dioxide level, which replaces the bicarbonate ion.

A nurse is completing a health history for a client who is at 6 weeks of gestation. The client informs the nurse that she smokes one pack of cigarettes per day. The nurse should advise the client that smoking places the client's newborn at risk for which of the following complications? A. Hearing loss B. Intrauterine growth restriction C. Type 1 diabetes mellitus D. Congenital heart defects

B. Intrauterine growth restriction i: Clients who smoke place their newborns and themselves at risk for diverse complications, including fetal intrauterine growth restriction, placental abruption, placenta previa, preterm delivery, and fetal death.

A nurse on a labor unit is admitting a client who reports painful contractions. The nurse determines that the contractions have a durtiong of 1min and a frequency of 3min. The nurse obtains the following vitals: fetal heart rate 130/min, maternal heart rate 128/min, and maternal blood pressure 92/54mmHg. Which of the following is the priority action for the nurse to take? A. Notify the provider of the findings. B. Position the client with one hip elevated. C. Ask the client if she needs pain medication. D. Have the client void.

B. Position the client with one hip elevated. i: Based on Maslow's hierarchy of needs, the client's need for an adequate blood pressure to perfuse herself and her fetus is a physiological need that requires immediate intervention. Supine hypotension is a frequent cause of low blood pressure in clients who are pregnant. By turning the client on her side and retaking her blood pressure, the nurse is attempting to correct the low blood pressure and reassess.

Rho(D) immune globulin (RhoGAM) is prescribed for a woman after the delivery of a newborn infant, and the nurse provides information to the woman about the purpose of the medication. The nurse determines that the woman understands the purpose of the medication if the woman states that it will protect her next baby from which of the following?

Being affected by Rh incompatibility (Rh incompatibility can occur when an Rh-negative mother becomes sensitized to the Rh antigen. Sensitization may develop when an Rh-negative woman becomes pregnant with a fetus who is Rh positive. During pregnancy and at delivery, some of the baby's Rh-positive blood can enter the maternal circulation, thus causing the woman's immune system to form antibodies against the Rh-positive blood. The administration of Rho(D) immune globulin prevents the woman from developing antibodies against Rh-positive blood by providing passive antibody protection against the Rh antigen.)

A nurse is assigned to care for a client 1 hour after delivery. The nurse palpates a firm, uterine fundus 2 cm above the umbilicus and displaced to the right. The nurse recognizes that this finding may indicate:

Bladder distention (Immediately following expulsion of the placenta, the fundus is firmly contracted, midline, and located one half to two thirds of the way between the symphysis pubis and umbilicus. Because the uterine ligaments are still stretched, a full bladder can move the uterus upward and to the side. Options 2, 3, and 4 are complications not usually indicated by a firm and displaced uterus.)

Methylergonovine (Methergine) is prescribed for a woman to treat postpartum hemorrhage. Before the administration of methylergonovine, the priority nursing action is to check the:

Blood pressure (Methylergonovine, which is an ergot alkaloid, is an agent that is used to prevent or control postpartum hemorrhage by contracting the uterus. Methylergonovine causes continuous uterine contractions and may elevate the blood pressure. A priority before the administration of the medication is to check the blood pressure. The health care provider should be notified if hypertension is present. Although options 1, 3, and 4 may be components of the postpartum data collection procedures, option 2 is related specifically to the administration of this medication. )

A client who has received a new prescription for oral contraceptives asks the nurse how to take them. Which of the following would the nurse instruct the client to report to her primary health care provider?

Blurred vision and headache

A nurse in a prenatal clinic is caring for a client who is at 38 weeks gestation and undergoing a contraction stress test. The test results are negative. Which of the following interpretations of this finding should the nurse make? A. There is no evidence of cervical incompetence. B. There is no evidence of two or more accelerations in fetal heart rate in 20 min. C. There is no evidence of uteroplacental insufficiency. D. There are less than 3 uterine contractions in a 10 min period.

C. There is no evidence of uteroplacental insufficiency. i: A contraction stress test determines how well the fetus tolerates the stress of uterine contractions. A test is negative when there are at least 3 uterine contractions in a 10-min period with no late or significant variable decelerations during electronic fetal monitoring. Uteroplacental insufficiency produces late decelerations.

A nurse in labor and delivery is caring for a client. Following delivery of the placenta, the nurse examines the umbilical cord. Which of the following vessels should the nurse expect to observe in the umbilical cord? A. Two veins and one artery B. One artery and one vein C. Two arteries and one vein D. Two arteries and two veins

C. Two arteries and one vein i: The vein carried the oxygenated, nutrient-rich blood from the placenta to the fetus, and the two arteries returned the blood to the placenta.

A pregnant 39-week-gestation gravida 1, para 0 client arrives on the labor and delivery unit with signs and symptoms of active labor. The nurse reviews the client's prenatal record and discovers that she has had a positive group B streptococcus (GBS) laboratory report during her prenatal course. After performing a cervical exam, the nurse confirms that the cervix is dilated 6 cm and 90% effaced. Which should be the nurse's first action?

Call the health care provider (HCP) to obtain a prescription for intravenous antibiotic prophylaxis (IAP).

During a busy shift on the labor and delivery unit, a nurse failed to document whether she administered a medication to aid uterine contractions. The nurse who's currently caring for the client is asked whether the medication has been successful. She explains that she hasn't been monitoring its effectiveness because she didn't know that the medication was administered. Which intervention by the nurse is best?

Call the nurse at home to inquire whether she gave the medication, then ask the client about the contractions while the physician is present.

A client is admitted to the labor and delivery area. The nurse-midwife checks for fetal descent, flexion, internal rotation, extension, external rotation, and expulsion. What do these terms describe?

Cardinal movements of labor

After receiving the shift report, the nurse realizes that she should monitor her postpartum client closely for puerperal infection. Which factor alerted the nurse to the client's risk for this complication?

Cesarean birth

The nurse is caring for a client in labor. The external fetal monitor shows a pattern of variable decelerations in fetal heart rate. What should the nurse do first?

Change the client's position.

A nurse is caring for a client who is breastfeeding and states that her nipples are sore. Which of the following interventions should the nurse suggest?

Change the newborn's position on the nipples with each feeding

A client arrives to the postpartum unit following the delivery of her newborn premature infant. On data collection, the nurse notes that the client is shaking uncontrollably. Which of the following nursing actions is appropriate?

Covering her with a warm blanket (In the postpartum period, a woman may commonly experience a shaking and uncontrollable chill immediately after birth. The exact cause of this occurrence is not known; however, it is thought to be associated with a nervous system reaction such as a vasovagal response. If the chill is not associated with an elevated temperature, it is of no clinical significance. The best nursing action is to provide a warm blanket to the client and a warm drink if this is not contraindicated. It is not necessary to contact the health care provider. Massaging the fundus and placing the client in the Trendelenburg's position have no effect on the client's condition.)

A nurse is providing teaching to a client who is planning on becoming pregnant about the changes she should expect. Identify the sequence of maternal changes. A. Quickening B. Lightening C. Goodell's sign D. Amenorrhea

D > C > A > B i: Amenorrhea, a presumptive sign of pregnancy, is one of the first physiological indications of pregnancy that occurs by 4 weeks of gestation. Goodell's sign, a probable sign of pregnancy, is the next of physiological indications to occur. Goodell's sign is the softening of the cervix that typically occurs at 5 to 6 weeks of gestation. Quickening, the mother's perception of the first fetal movement, is a presumptive sign of pregnancy that typically occurs between 16 and 20 weeks of gestation. Lightening is the last of these physiological signs of pregnancy to occur. As the fetus descends into the pelvic cavity the fundal height decreases, which typically occurs between 38 and 40 weeks of gestation.

A nurse is caring for a group of clients on an intrapartum unit. Which of the following findings should be reported to the provider immediately? A. A tearful client who is at 32 weeks of gestation and is experiencing irregular, frequent contractions. B. A client who is at 28 weeks of gestation and receiving terbutaline reports fine tremors. C. A client who has a diagnosis of preeclampsia has 2+ proteinuria and 2+ patellar reflexes. D. A client who has a diagnosis of preeclampsia reports epigastric pain and unresolved headache.

D. A client who has a diagnosis of preeclampsia reports epigastric pain and unresolved headache. i: These findings indicate that the client's condition is worsening and are signs of severe preeclampsia. They should be reported to the provider immediately. Other manifestations of severe preeclampsia include: blood pressure of 160/100 mm Hg or greater, proteinuria 3+ to 4+, oliguria, visual disturbances, such as blurred vision, hyperreflexia with clonus, nausea, vomiting, epigastric pain, and right upper-quadrant pain.

A postpartum nurse is collecting data from a client who delivered a viable newborn 2 hours ago. The nurse palpates the fundus and notes the character of the lochia. Which characteristic of the lochia would the nurse expect to note at this time?

Dark red lochia (In assessment of the perineum, the lochia is checked for amount, color, and the presence of clots. The color of the lochia during the fourth stage of labor (the first 1 to 4 hours after birth) is a dark red. Options 1, 2, and 3 are not the expected characteristics of lochia at this time.)

A nurse is caring for a postpartum client who is being treated for thrombophlebitis. The client is receiving an anticoagulant by intravenous infusion. The nurse monitors for adverse effects of the anticoagulant by checking the client for:

Hematuria, ecchymosis, and epistaxis (The treatment for thrombophlebitis is anticoagulant therapy. Adverse effects of anticoagulants include bleeding and would be recognized by the presence of hematuria, ecchymosis, and epistaxis. )

In the fourth stage of labor, a full bladder increases the risk of which postpartum complication?

Hemorrhage

When the client has been given instructions about postoperative complications following cesarean delivery, the nurse interprets that the client requires clarification of the information when the client identifies which situation as a reason to notify her health care provider?

Her temperature is 99° F. (By definition, a postpartum infection is present when the temperature is greater than 100.4° F or more on 2 successive days, not counting the first 24 hours after birth. Temperatures of this magnitude must be considered a sign of a postpartum infection unless proven otherwise. Therefore a temperature of 99° F does not warrant notification of the health care provider. The woman needs to contact the health care provider if the temperature rises above 100.4° F. The other signs listed should be reported as stated.)

A nurse is caring for a client who had a cesarean section to deliver a nonviable fetus as a result of abruptio placentae. The client develops signs of disseminated intravascular coagulopathy (DIC). The spouse asks the nurse what is happening, and the nurse explains the condition. The spouse becomes upset and says to the nurse, "I lost my baby and now my wife! What am I going to do?" Which of the following appropriately describes the situation?

Hopelessness related to loss of the baby and illness of the spouse (A person who lacks hope experiences hopelessness and sees no way out of the situation except for death. There are no data in the question that support the diagnosis of grieving, deficient knowledge, or anxiety.)

A nurse is reviewing the procedure for vitamin K injection in a newborn. Which information is included in the procedure?

Inject into skin that has been cleansed and allowed to have alcohol dry on the puncture site for 1 minute. (Vitamin K is given in the middle third of the vastus lateralis muscle using a 25-gauge, ⅝-inch needle. It is injected into skin that has been cleansed or allowed to alcohol dry for 1 minute to remove organisms and prevent infection. It is given at a 90-degree angle. The site is massaged after removing the needle to increase absorption.)

A nurse is reviewing the procedure for vitamin K injection in the newborn with a nursing student. Which information would the nurse provide to the student?

Inject into skin that has been cleansed with alcohol. (Vitamin K is given in the middle third of the vastus lateralis muscle using a 25-gauge, ⅝-inch needle. It is injected into skin that has been cleansed with alcohol and allowed to dry for 1 minute; this removes organisms and prevents infection. It is administered at a 90-degree angle. The site is massaged after removing the needle to increase absorption of the medication.)

A woman with type 1 diabetes mellitus is in labor. Based on the knowledge of insulin and diabetes and pregnancy, the nurse will be prepared to care for a newborn infant who is likely to have which complication?

Macrosomia (Typically, infants of diabetic mothers are large for gestational age. Maternal glucose crosses over the placenta to the fetus. The fetus is able to produce its own insulin; therefore excessive body growth (macrosomia) results from high maternal glucose. After birth, hypoglycemia may be a problem because the infant's pancreas continues to produce large amounts of insulin (hyperinsulinemia), which quickly deplete the infant's glucose supply. Infants of diabetic mothers usually are delivered just before or at term because of an increased risk of ketoacidosis and intrauterine fetal death after 36 weeks. Polycythemia, not anemia, is commonly associated with infants of diabetic women.)

A mother is breast-feeding her newborn baby and experiences breast engorgement. The nurse encourages the mother to do which of the following to provide relief of the engorgement?

Massage the breasts before feeding to stimulate let-down. (Comfort measures for breast engorgement include massaging the breasts before feeding to stimulate let-down, wearing a supportive and well-fitting bra at all times, taking a warm shower or applying warm compresses just before feeding, and alternating breasts during feeding.)

A nurse is caring for a client who is 4 hr postpartum following a vaginal birth. The client has saturated a perineal pad within 10 min. Which of the following actions should the nurse take first?

Massage the client's fundus

On a client's first postpartum day, nursing assessment reveals vital signs within normal limits, a boggy uterus, and saturation of the perineal pad with lochia rubra. Which nursing intervention takes highest priority?

Massaging the uterus gently

Breast-feeding instructions for the postpartum mother should include avoidance of soaps on the nipples, frequent changing of breast pads, intermittent exposure of nipples to air, and handwashing before handling the breast and before breast-feeding. The nurse understands that these measures are specific to the prevention of:

Mastitis (Mastitis is an infection frequently associated with a break in the skin surface of the nipple. The measures described in the question are personal hygiene measures to help prevent mastitis.)

On assessment of the fetal heart rate (FHR) of a laboring woman, the nurse discovers decelerations that have a gradual onset, last longer than 30 seconds, and return to the baseline rate with the completion of each contraction. The nurse plans care, knowing that this identifies which category of decelerations?

Periodic, early decelerations that indicate fetal head compression

Methylergonovine (Methergine) is prescribed for a client with postpartum hemorrhage. Before administering the medication, a nurse contacts the health care provider who prescribed the medication if which of the following conditions is documented in the client's medical history?

Peripheral vascular disease (Methylergonovine is an ergot alkaloid that is used to treat postpartum hemorrhage. Ergot alkaloids are avoided in clients with significant cardiovascular disease, peripheral vascular disease, hypertension, eclampsia, or preeclampsia, because these conditions are worsened by the vasoconstrictive effects of the ergot alkaloids)

The nurse in a birthing room is monitoring a client with dysfunctional labor for signs of fetal or maternal compromise. Which assessment finding should alert the nurse to a compromise?

Persistent nonreassuring fetal heart rate

The maternity nurse is caring for a client with abruptio placentae and is monitoring her for disseminated intravascular coagulation (DIC). Which assessment findings are most likely associated with disseminated intravascular coagulation? Select all that apply.

Petechiae Hematuria Prolonged clotting times Oozing from injection sites

The nurse performs a vaginal assessment on a pregnant client in labor. On assessment, the nurse notes the presence of the umbilical cord protruding from the vagina. Which is the initial nursing action?

Place the client in Trendelenburg's position.

Retained placenta:

Placenta or fragments of the placenta remain in the uterus preventing the uterus from contracting which leads to uterine atony or subinvolution Med given: oxytocin....if unsuccessful then tocolytic for d&c

A postpartum client is ready for discharge. During discharge preparation, the nurse should instruct her to report which of the following to her primary health care provider?

Redness, warmth, and pain in the breasts

A nurse is caring for a client who is breastfeeding her newborn and asks the nurse about the changes she should make in her diet. Which of the following dietary changes should the nurse suggest?

Reduce her intake of iron

A nurse is caring for a client who is breastfeeding her newborn and asks the nurse about the changes she should make in her diet. Which of the following dietary changes should the nurse suggest?

Reduce her intake of iron.

A client admitted with preeclampsia complains of a headache. When caring for a client with preeclampsia, which action is a priority?

Reducing visual and auditory stimulation

A nurse receives a report at the beginning of the shift regarding a client with an intrauterine fetal demise. Which of the following would the nurse expect to note when collecting data on the client?

Regression of pregnancy symptoms and absence of fetal heart tones (Symptoms of an intrauterine fetal demise include decrease in fetal movement, no change or a decrease in fundal height, and absent fetal heart tones. Many symptoms of pregnancy may diminish, such as uterine size, and breast size and tenderness. Option 2 identifies signs of preeclampsia. Option 3 can be a result of twins. Option 4 is associated with hyperemesis gravidarum.)

A client in active labor believes in a holistic approach to health care. Which intervention might be useful to the client during labor?

Reiki

Relationship of the fetus's presenting part to the mother's pelvis

Relationship of the fetus's presenting part to the mother's pelvis

A 41-year-old multipara client had a spontaneous vaginal delivery of an 8-lb (3,629-g) baby 3 hours ago. The nurse collects the following data: "Fundus firm, three fingerbreadths above the umbilicus and deviated to the right. Perineal pad saturated after 20 minutes." Which nursing intervention by the licensed practical nurse (LPN) is best?

Reminding the client to void and helping her to the bathroom

A client is admitted to the labor and delivery suite with an intrauterine fetal demise. A nurse determines that the discussion with the parents was effective in preparing them for the delivery when the parents:

Request to hold the infant following delivery (The nurse should explain to the parents the expected events following delivery of the fetus and should tell the parents that they can hold their infant following delivery. Viewing and holding the dead infant can alleviate any negative images the mother or her partner may have. Providing a picture or other mementos will help preserve the memory of the infant. If the parents refuse a picture, most hospitals will keep a picture and copy of the footprints on file for parents to access later. Parents should be encouraged to verbalize their feelings, ask questions about the process, and make their own decisions about care as much as possible.)

The nurse has been working with a laboring client and notes that she has been pushing effectively for 1 hour. What is the client's primary physiological need at this time?

Rest between contractions

Inversion of the uterus

Retained placenta Uterine atony Excessive fundal pressure Abnormally adherent placental tissue Multiparity Fundal implantation of the placenta Extremem traction applied to the umbilical cord Leiomyomas

A client experiences subinvolution during the puerperium. The nurse recalls that which of the following are the most common causes for this occurrence?

Retained placental fragments and infections (Retained placental fragments and infections are the primary causes of subinvolution. When either of these processes is present, the uterus has difficulty contracting.)

The nurse is caring for a postpartum client who had a vaginal delivery with a midline episiotomy. Which nursing diagnosis takes priority for this client?

Risk for deficient fluid volume related to hemorrhage

A postpartum client suspected of having an infection is informed that she will be unable to have the newborn present in the room with her. The nurse plans care, knowing that which of the following is the highest priority problem at this time?

Risk of ineffective bonding between the mother and newborn (There is a period shortly after birth that is uniquely important to attachment and mother-infant bonding. Option 2 identifies the problem that could exist if the client is unable to have the newborn infant present in the room. Options 1, 3, and 4 do not relate to the information provided in the question.)

A breast-feeding baby will turn his head toward the mother's breast in a natural instinct to find food. What is the name of this reflex?

Rooting reflex

Late decels mean:

UPI causing inadequate fetal oxygenation Maternal hypotension placental abruption Uterine hyperstimulation w/pitocin

Variable decels mean:

Umbilical cord compression Short cord Prolapsed cord Nuchal cord Oligohydraminos

A client is in the third stage of labor. Which finding indicates impending placental separation?

Umbilical cord lengthening

When can bathing by immersion be done?

When the newborn's umbilical cord has fallen off and the circumcision has healed on males

a nurse on the labor and delivery unit is caring for a client following a vaginal examination by the provider which is documented as: -1. which of the following interpretations of this finding should the nurse make? a. the presenting part is 1 cm above the ischial spines b. the presenting part is 1 cm below the ischial spines c. the cervix is 1 cm dilated d. the cervix is effaced 1 cm

a (Station is the relation of the presenting part to the ischial spines of the maternal pelvis and is measured in centimeters above, below, or at the level of the spines. If the station is minus (-) 1, then the presenting part is 1 cm above the ischial spines.) (b-Station is the relation of the presenting part to the ischial spines of the maternal pelvis and is measured in centimeters above, below, or at the level of the spines. If the presenting part is 1 cm below the ischia spines it would be documented as plus (+) 1.) (c-Rationale C. Dilation of the cervix is measured from closed to 10 cm. It is not documented in terms of minus 1.) (d-Effacement or thinning and shortening of the cervix is measured from 0 to 100%. It is not documented in terms of minus 1.)

a nurse is caring for a client who is in preterm labor at 32 weeks of gestation. the client asks the nurse, "will my baby be okay?" which of the following responses should the nurse offer? a. "you must be feeling scared and powerless" b. "everyone worries about her baby when she's in labor" c. "you pregnancy is advanced so your baby should be fine" d. "we have a neonatal unit here that's equipped to handle emergencies"

a (This response illustrates the therapeutic communication technique of restatement. The nurse shows empathy for the client by recognizing that the client is concerned about the safety of the fetus and is powerless to do anything about the situation. This open-ended statement encourages further communication by the client.)

a nurse is caring for a client who is in the active phase of the first stage of labor. when monitoring the uterine contractions, which of the following findings should the nurse report to the provider? a. contractions lasting longer than 90 seconds b. contractions occurring every 3 to 5 min c. contractions are strong in intensity d. client reports feeling contractions in lower back

a (A pattern of prolonged uterine contractions lasting more than 90 seconds is an indication that there is inadequate uterine relaxation and should be reported to the provider.) (b-In the active phase of the first stage of labor, contractions are more regular and occur at 3 to 5 min intervals. This is an expected finding.) (c-This is an expected finding in a client who is moving from the active to transition phase of the first stage of labor. It does not need to be reported to the provider.) (d-This is an expected finding in a client who is in true labor. As the labor progresses, the contractions radiate to the abdomen.)

a nurse is teaching a client about black cohosh. which of the following information sould the nurse include in the teaching? a. "black cohosh should not be taken during pregnancy" b. "black cohosh helps relieve headache pain" c. "black cohosh increases the risk for bleeding" d. "black cohosh is a stimulant"

a (Black cohosh has estrogenic properties and should not be taken during pregnancy.) (b-Black cohosh has no analgesic effect to relieve headache pain; however, feverfew is known to alleviate migraine headaches when taken prophylactically.) (c-Black cohosh does not affect bleeding time; however, garlic decreases platelet aggregation and can increase the risk for bleeding.) (d-Black cohosh does not act as a stimulant; however, ephedra acts as a stimulant and can increased heart rate and elevated blood pressure.)

a nurse receives report about assignned clients at the start of the shift. which of the following clients should the nurse plan to see first? a. a client who experienced a cesarean birth 4 hr ago and reports pain b. a client who has preeclampsia with a BP of 138/90 mm Hg c. a client who experienced a vaginal birth 24 hr ago and reports no bleeding d. a client who is scheduled for discharge following a laparoscopic tubal ligation

a (Using Maslow's hierarchy of needs, assessment of pain and meeting the physiological needs of a surgical client are the priority nursing actions.) (b-A BP of 138/90 mm Hg is an expected finding in a client who has mild preeclampsia and this client does not need to be seen first.) (c-A client who experienced a vaginal birth 24 hr ago and reports no bleeding is an expected finding and this client does not need to be seen first.) (d-A client who is scheduled for discharge following a laparoscopic tubal ligation does not need to be seen first.)

a nurse is caring for a client who experienced a vaginal delivery 12 hr ago. when palpating the client's abdomen, at which of the following positions should the nurse expect to find the uterine fundus? a. at the level of the umbilicus b. 2 cm above the umbilicus c. one fingerbreadth above the symphysis pubis d. to the right of the umbilicus

a (Within 12 hr, the fundus should be palpable at the level of the umbilicus and then recede 1 to 2 cm each day.) (b-The position of the fundus 2 cm above the umbilicus is an indication of subinvolution.) (c-The uterus would be palpated at a position between the umbilicus and symphysis pubis in a client who is approximately 1 week postpartum.) (d-A uterine fundus that is deviated to the right or left of the umbilicus indicates the client has a full bladder.)

APGAR scoring is:

a brief physical exam done immediately following birth to rule out abnormalities.

nevus flammues (port wine stains)

a capillary angioma below the surface of the skin that is purple or red, varies in size and shape, is commonly seen on face and does not blanch or disappear.

a nurse is caring for a client who is to undergo a biophysical profile. the client asks the nurse what is being evaluated during this test. which of the following should the nurse include? select all that apply a. fetal breathing b. fetal motion c. fetal neck translucency d. amniotic fluid volume e. fetal gender

a, b, d (a-A biophysical profile is an assessment of fetal well-being and includes ultrasound evaluation of fetal breathing movements, gross fetal movements, and amniotic fluid volume.) (b-A biophysical profile is an assessment of fetal well-being and includes ultrasound evaluation of fetal breathing movements, gross fetal movements, and amniotic fluid volume.) (d-A biophysical profile is an assessment of fetal well-being and includes ultrasound evaluation of fetal breathing movements, gross fetal movements, and amniotic fluid volume.) (c-Fetal neck, or nuchal translucency, also called NT screening, is a separate evaluation tool that can be performed using ultrasound.) (e-Fetal gender can be identified via ultrasound but is not included in a biophysical profile.)

a nurse on the obstetric unit is caring for a client who experienced abruptio placentae. the nurse observes petechia and bleeding around the IV access site. the nurse should recognize that this client is at risk for which of the following complications? a. anaphylactoid syndrome of pregnancy b. disseminated intravascular coagulation c. preeclamsia d. peurperal infection

b (Clinical manifestations of disseminated intravascular coagulation (DIC) include oozing from intravenous access and venipuncture sites; petechiae, especially under the site of the blood pressure cuff; spontaneous bleeding from the gums and nose; other signs of bruising; and hematuria.) (a-Anaphylactoid syndrome of pregnancy, due to an amniotic fluid embolism, typically occurs within 30 min after birth and is manifested by sudden, acute onset of hypoxia, hypotension, cardiac arrest, and coagulopathy.) (c-Preeclampsia is typically seen in the antepartum period and is manifested by elevated blood pressure, hyperactive reflexes, proteinuria, and edema.) (d-Puerperal or postpartum infection is identified by the presence of a fever of 380 C (100.40 F) or higher on 2 consecutive days of the first 10 postpartum days and can include endometritis, wound infections, urinary tract infections, and mastitis.)

a nurse in a prenatal clinic is caring for a client who is at 38 weeks of gestation and reports heavy, red vaginal bleeding. the bleeding started spontaneously in the morning and is not accompanied by contractions. the client is not in distress and she states that she can "feel the baby moving." an ultrasound is scheduled stat. the nurse should explain to the client that the purpose of the ultrasound is to determine which of the following? a. fetal lung maturity b. location of the placenta c. viability of the fetus d. the biparietal diameter

b (Painless, spontaneous vaginal bleeding might indicate that the client has placenta previa. Placenta previa is a condition in which the placenta is implanted low in the uterus, sometimes to the point of covering the cervical os. As the cervix effaces, the client begins to bleed. The ultrasound will show the location of the placenta and help to determine what sort of delivery the client requires and how emergent it is.)

a nurse is caring for a client who presents to a labor and delivery unit experiencing rapidly progressing labor. which of the following is the priority action for the nurse to take? a. cut the umbilical cord b. apply perineal pressure to the emerging fetal head c. prevent the perineum from tearing d. promote delivery of the placenta

b (Using Maslow's hierarchy of needs, the priority intervention is to prevent injury to the fetus during the delivery by applying gentle perineal pressure to the emerging head. This avoids rapid expulsion of the fetal head. A change in pressure within the fetal skull due to a rapid delivery can cause neurologic damage (increased intracranial pressure and dural/subdural tearing). Rapid birth can also cause maternal injury, such as vaginal or perineal lacerations.)

a nurse is caring for a client during a nonstress test (NST). at the end of a 30-min period of observation, the nurse notes the following findings: the FHR baseline is 120/min with minimal variability and no accelerations. there are two decelerations of 15/min in the FHR during a period of fetal movement, each lasting 20 seconds. which of the following interpretations of these findings should the nurse make? a. a negative test b. a nonreactive test c. a positive test d. a reactive test

b (An NST that does not produce two or more qualifying accelerations within a 20-min period is interpreted as nonreactive. Qualifying accelerations peak at least 15 /min above the FHR baseline and last at least 15 seconds.) (a-A negative test is one of the findings for a client having a contraction stress test (CST). This result indicates that at least three uterine contractions occurred in a 10-min period with no late or significant variable decelerations.) (c-A positive test is one of the findings for a client having a contraction stress test (CST). This result indicates that late decelerations occurred with 50% or more of the contractions, even if fewer than three contractions occurred in a 10-min period.) (d-An NST is interpreted as reactive if the fetus has a minimum of two accelerations in a 20-min period, each lasting at least 15 seconds and peaking at least 15/min above the FHR baseline.)

a nurse on a labor unit is admitting a client who reports painful contractions. the nurse determines that the contractions have a duration of 1 min and a frequency of 3 min. the nurse obtains the following vital signs: FHR 130/min, maternal HR 128/min, and maternal BP 92/54 mm Hg. which of the following is the priority action for the nurse to take? a. notify the provider of the findings b. position the client with one hip elevated c. ask the client if she needs pain medication d. have the client void

b (Based on Maslow's hierarchy of needs, the client's need for an adequate blood pressure to perfuse herself and her fetus is a physiological need that requires immediate intervention. Supine hypotension is a frequent cause of low blood pressure in clients who are pregnant. By turning the client on her side and retaking her blood pressure, the nurse is attempting to correct the low blood pressure and reassess.) (a-Calling the provider may be appropriate; however, this is not the priority intervention.) (c-The client's comfort should be addressed; however, this is not the priority intervention.) (d-The client should be encouraged to empty her bladder every 2 hr during labor; however, this is not the priority intervention.)

a nurse on the labor and delivery unit is caring for a patient who is having induction of labor with oxytocin administered through a secondary IV line. uterine contractions occur every 2 min, last 90 sec, and are strong to palpation. the baseline FHR is 150/min, with uniform decelerations beginning at the peak of the contraction and a return to baseline after the contraction is over. which of the following actions should the nurse take? a. decrease the rate of infusion of the maitenance IV solution b. discontinue the infusion of the IV oxytocin c. increase the rate of infusion of the oxytocin d. slow the client's rate of breathing

b (Discontinue the oxytocin infusion immediately if a client is experiencing late decelerations due to uterine hyperstimulation.) (a-Increasing the rate of infusion of the maintenance IV solution is an appropriate action to take when late decelerations occur.) (c-Increasing the rate of the oxytocin infusion can result in fetal distress due to uterine hyperstimulation.) (d-Oxygen should be administered at a rate of 8 to 10 L/min when late decelerations occur due to uterine hyperstimulation.)

a nurse in a hospital is caring for a client who is at 38 weeks of gestation and has a large amount of painless, bright red vaginal bleeding. the client is placed on a fetal monitor indicating a regular FHR of 138/min and no uterine contractions. the client's vital signs are: BP 98/52 mm HG, HR 118 BPM, RR 24/min, and temp 36.4 C (97.6 F). which of the following is the priority nursing action? a. insert an indwelling urinary catheter b. initiate IV access c. witness the signature for informed consent for surgery d. prepare the abdominal and perineal areas

b (Insertion of a large-bore IV catheter is the priority nursing action. The client is losing blood rapidly, has hypotension, and tachycardia. IV access will allow IV fluids and blood to be administered quickly if hypovolemia develops.) (a-An indwelling urinary catheter can be inserted in the delivery room just prior to delivery. This is not the priority nursing action.) (c-Rationale C. This is not the nurse's priority action at this time. A family member can sign the consent form if needed.) (d-Skin preparation can be delayed until just prior to a cesarean delivery. This is no the priority nursing action.)

a nurse is caring for a client who is in active labor and notes late decelerations on the fetal monitor. which of the following is the priority nursing action? a. elevate the client's legs b. position the client on her side c. administer oxygen via face mask d. increase the infusion rate of the IV fluid

b (Late decelerations stem from decreased blood perfusion to the placenta or compression of the placenta. A position change should increase perfusion or decrease compression, and it is the first intervention the nurse should try. The greatest risk to the client is fetal hypoxia, so the priority action is the one that has the best chance of improving fetal perfusion.) (a-Elevating the client's legs might help relieve maternal hypertension, but there is a higher priority action.) (c-Administering oxygen can help increase the oxygen concentration of whatever blood does get to the placenta, but there is a higher priority action.) (d-Increasing the rate of fluid infusion is an appropriate intervention for late decelerations, but there is a higher priority action.)

a nurse is admitting a client who is at 36 weeks gestation and has painless, bright red vaginal bleeding. the nurse should recognize this finding as an indication of which of the following conditions? a. abruptio placentae b. placenta previa c. precipitous labor d. threatened abortion

b (Painless, bright red vaginal bleeding in the second or third trimester is a manifestation of placenta previa.) (a-Abruptio placentae classically presents with vaginal bleeding, abdominal pain, uterine tenderness, and contractions. In some cases the hemorrhage can remain concealed.) (c-Precipitous labor contractions are hypertonic and often tetanic in intensity.) (d-Symptoms of a threatened abortion include spotting of blood and possible mild uterine cramping prior to 20 weeks of gestation.)

a nurse is creating the plan of care for a client who is at 39 weeks of gestation and in active labor. which of the following actions should the nurse include in the plan of care? a. keep four side rails up while the client is in bed b. check the cervix prior to analgesic administration c. monitor the (FHR) every hour d. insert an indwelling urinary catheter

b (Prior to administering an analgesic during active labor, the nurse must know how many centimeters the cervix has dilated. Administration too close to the time of delivery could cause respiratory depression in the newborn.) (a-Raising side rails is not usually necessary during labor unless clients have received opioid or sedative medications. Many clients in labor prefer to walk to help their labor progress. Raising four rails restrains the client.) (c-Monitoring the FHR every hour is not frequent enough. Even for low-risk clients, most facilities' protocols require monitoring the FHR every 15 to 30 min while the client is in the first stage of labor and every 5 to 15 min in the second stage (as long as the FHR has reassuring characteristics). High-risk clients require more frequent monitoring.) (d-Inserting an indwelling urinary catheter is not generally necessary. The nurse should assess for bladder fullness, especially if the client has had epidural anesthesia. If the client is unable to urinate, a straight catheter will suffice in most instances.)

a nurse in a prenatal clinic is caring for a client who is at 39 weeks of gestation and who asks about the signs that precede the onset of labor. which of the following should the nurse identify as a sign that precedes labor? a. decreased vaginal discharge b. a surge of energy c. urinary retention d. weight gain of 0.5 to 1.5 kg

b (Prior to the onset of labor, the pregnant client experiences a surge of energy.) (a-Increased vaginal discharge is a result of congestion of vaginal mucous membranes.) (c-The return of urinary frequency precedes the onset of labor due to bladder pressure after lightening.) (d-A loss of 0.5 to 1.5 kg occurs before the onset of labor due to water loss.)

a nurse is caring for a client who is in the first stage of labor and is using pattern-paced breathing. the client says she feels lightheaded and her fingers are tingling. which of the following actions should the nurse take? a. administer oxygen via nasal cannula b. assist the client to breathe into a paper bag c. have the client tuck her chin to her chest d. instruct the client to increase her respiratory rate to more than 42 breaths per min

b (This client is experiencing respiratory alkalosis due to hyperventilation. The client should be assisted to breathe into a paper bag or to cup her hands over her mouth to increase the carbon dioxide level, which replaces the bicarbonate ion.) (a-This client is experiencing respiratory alkalosis due to hyperventilation. She needs to rebreathe carbon dioxide to replace the bicarbonate ion rather than receive additional oxygen.) (c-This client is experiencing respiratory alkalosis due to hyperventilation. Having the client tuck her chin does not relieve this condition.) (d-This client is experiencing respiratory alkalosis due to hyperventilation. She needs to rebreathe carbon dioxide to replace the bicarbonate ion and should maintain a breathing rate of 32 to 40 breaths per minute.)

a nurse is admitting a client who has a diagnosis of preterm labor. the nurse anticipates a prescription by the provider for which of the following medication? select all that apply a. prostaglandin E2 b. indomethacin c. magnesium sulfate d. methylergonovine e. oxytocin

b, c (b-Indomethacin is used to relax uterine smooth muscles and suppress uterine activity in clients who have a diagnosis of preterm labor.) (c-Magnesium sulfate is a tocolytic and stops contractions in clients experiencing preterm labor.) (a-Prostaglandin E2 is used to stimulate cervical ripening and hasten the onset of labor.) (d-Methylergonovine promotes uterine contractions to manage postpartum hemorrhage.) (e-Oxytocin is used to induce and augment labor.)

a nurse receives report about a client who is in labor and is having contractions 4 min apart. which of the following patterns should the nurse expect on the fetal monitoring tracing? a. contractions that last for 60 seconds each with a 4-min rest between contractions b. a contraction that lasts 4 min followed by a period of relaxation c. contractions that last for 60 seconds each with a 3-min rest between contractions d. contractions that last 45 seconds each with a 3-min rest between contractions

c (A contraction interval indicates how often a uterine contraction occurs. The nurse should measure the interval from the beginning of one contraction to the beginning of the next contraction. A contraction lasting 60 seconds with a relaxation period of 3 min is equivalent to contractions every 4 min.) (a-A contraction interval indicates how often a uterine contraction occurs. The nurse should measure the interval from the beginning of one contraction to the beginning of the next contraction. This contraction interval is 5 min.) (b-A contraction interval indicates how often a uterine contraction occurs. The nurse should measure the interval from the beginning of one contraction to the beginning of the next contraction. This pattern is longer than 4 min.) (d-A contraction interval indicates how often a uterine contraction occurs. The nurse should measure the interval from the beginning of one contraction to the beginning of the next contraction. This contraction pattern is less than 4 min.)

a nurse is performing leopold maneuvers on a client who is in labor and determines the fetus is in an RSA position. which of the following fetal presentations should the nurse document in the client's medical record? a. vertex b. shoulder c. breech d. mentum

c (An RSA position indicates that the body part of the fetus that is closest to the cervix is the sacrum. Therefore, the buttocks or feet are the presenting part, which is classified as a breech presentation.) Fetal position is indicated by a three letter abbreviation. The first letter indicates the side of the maternal pelvis that the presenting part of the fetus is located. The second letter indicates the part of the fetus that is closest to the cervix. The third letter indicates whether the fetal presenting part is located in the anterior, posterior, or transverse portion of the maternal pelvis. (a-A vertex presentation indicates that the fetal head is the closest fetal part to the cervix. A letter "O" as the second letter in the abbreviation would indicate the fetal occiput was the presenting part.) (b-A shoulder as the presenting part is denoted as an "A" for the acromion process.) (d-A letter "M" as the second letter in the abbreviation would indicate the fetal chin was the presenting part. Mentum indicates that the fetus has fully extended its head and is presenting with its chin.)

a nurse admits a woman who is at 38 weeks of gestation and in early labor with ruptured membranes. the nurse determines that the client's oral temperature is 38.9 C (102 F). besides notifying the provider, which of the following is an appropriate nursing action? a. recheck the client's temperature in 4 hr b. administer glucocorticoids intramuscularly c. assess the odor of the amniotic fluid d. prepare the client for emergency cesarean section

c (Chorioamnionitis is an infection of the amniotic cavity that presents with maternal fever, tachycardia, increased uterine tenderness, and foul-smelling amniotic fluid.) (a-The client's temperature should be checked at least every 2 hours after rupture of membranes.) (b-Antenatal glucocorticoids are indicated for all women between 24 and 34 weeks of gestation when preterm birth is threatened.) (d-While clients who have chorioamnionitis are more likely to have a dysfunctional labor, it is not an indication for an emergent cesarean section.)

a nurse in a provider's office is caring for a client who is at 36 weeks of gestation and scheduled for an amniocentesis. the client asks why she is having an ultrasound prior to the procedure. which of the following is an appropriate response by the nurse? a. "this will determine if there is more than one fetus" b. "it is useful for estimating fetal age" c. "it assists in identifying the location of the placenta and fetus" d. "this is a screening tool for spina bifida"

c (Identifying the positions of the fetus, placenta, and amniotic fluid pockets immediately prior to the amniocentesis increases the safety of this test by assisting with correct placement of the needle.) (a-Although ultrasound might be used to assess the number of fetuses, this is not the purpose of an ultrasound prior to amniocentesis.) (b-Although ultrasound might be used to assess the age of the fetus, this is not the purpose of an ultrasound prior to amniocentesis.) (d-Although ultrasound might be used to assess the fetus for spina bifida, this is not the purpose of an ultrasound prior to amniocentesis.)

a nurse is caring for a client who is in active labor with 7 cm of cervical dilation and 100% effacement. the fetus is at 1+ station, and the client's amniotic membranes are intact. the client suddenly states that she needs to push. which of the following actions should the nurse take? a. assist the client into a comfortable position b. observe the perineum for signs of crowning c. have the client pant during the next contractions d. help the client to the bathroom to void

c (Panting is rapid, continuous, shallow breathing. It helps a client in labor refrain from pushing before her cervix reaches full dilation. Observe for hyperventilation and have the client exhale slowly through pursed lips.) (a-A comfortable position will not affect the client's need to push.) (b-At 7 cm of cervical dilation, it is too soon to observe for crowning.) (d-Emptying the bladder does not alter the client's urge to push.)

a nurse is caring for a client who is in labor. which of the following nursing actions reflects application of the gate control theory of pain? a. administer prescribed analgesic medication b. encourage the client to rest between contractions c. massage the client's back d. turn the client onto her left side

c (The gate control theory of pain is based on the concept of blocking or preventing the transmission of pain signals to the brain by using distraction techniques such as massage. Massaging the client's back focuses on neuromuscular and cognitive changes.) (a, b, d-The gate control theory of pain is based on the concept of preventing the transmission of pain signals to the brain by using distraction techniques. Administering pain medication does not address this theory.)

a nurse is caring for a client who was admitted to the maternity unit at 38 weeks of gestation and who is experiencing polyhydramnios. the nurse should understand that this diagnosis means which of the following? a. the client is carrying more than one fetus b. there is an elevated level of AFP in the amniotic fluid c. an excessive amount of amniotic fluid is present d. the fetus is likely to have a congenital anomaly, be growth restricted, or demonstrate fetal distress during labor

c.

a nurse in a prenatal clinic is caring for a client. using leopold maneuvers, the nurse palpates a round, firm, moveable part in the fundus of the uterus and a long, smooth surface on the client's right side. in which abdominal quadrant should the nurse expect to auscultate fetal heart tones? a. left lower b. right lower c. left upper d. right upper

d (Fetal heart tones are best auscultated directly over the location of the fetal back, which, in this breech presentation, would be in the right upper quadrant.) (a-This is an incorrect response. Fetal heart tones are best auscultated directly over the location of the fetal back, which, in this breech presentation, would be above the maternal umbilicus.) (b-This is an incorrect response. Fetal heart tones are best auscultated directly over the location of the fetal back which, in this breech presentation, would be above the maternal umbilicus.) (c-This is an incorrect response. Fetal heart tones are best auscultated directly over the location of the fetal back.)

a nurse is preparing to administer magnesium sulfate IV to a client who is experiencing preterm labor. which of the following is the priority nursing assessment for this client? a. temperature b. FHR c. bowel sounds d. respiratory rate

d (Magnesium sulfate is typically administered to a client in preterm labor to achieve the tocolytic (uterine relaxation) effect. Magnesium sulfate depresses the function of the central nervous system, causing respiratory depression. Baseline assessment of respiratory status, checking the respiratory rate frequently, and reassessment of respiratory status with each change in dosage of magnesium sulfate is the primary focus when assessing the client. There is a narrow margin between what is considered a therapeutic dose and a toxic dose of magnesium sulfate.)

a nurse is assessing a client who is in active labor and notes that the presenting part is at 0 station. which of the following is the correct interpretation of this clinical finding? a. the fetal head is in the left occiput posterior position b. the largest fetal diameter has passed through the pelvic outlet c. the posterior fontanel is palpable d. the lowermost portion of the fetus is at the level of the ischial spines

d (The presenting part is at 0 station when its lowermost portion is at the level of an imaginary line drawn between the client's ischial spines. Levels above the ischial spines are negative values: -1, -2, -3. Levels below the ischial spines are positive values: +1, +2, +3.) (a-This describes a reference point of the fetal head in relation to the maternal pelvis, indicating a vertex presentation with the fetus in an attitude of general flexion.) (b-The pelvic outlet is the lower border of the true pelvis. When the largest fetal diameter has passed through the outlet, the station is greater than zero.) (c-This is a clinical finding indicating that the fetal lie is longitudinal with the fetus in an attitude of general flexion.)

a nurse is caring for a client who is in the first stage of labor. the nurse observes the umbilical cord protruding from the vagina. which of the following actions should the nurse perform first? a. cover the cord with a sterile, moist saline dressing b. prepare the client for an immediate birth c. place the client in knee-chest position d. insert a gloved hand into the vagina to relieve pressure on the cord

d (This is the first nursing action because it is essential to prevent any pressure on the umbilical cord to promote oxygenation of the fetus.) (a-While this is appropriate, it is not the first action the nurse should take.) (b-Although an emergency vaginal or cesarean birth might be necessary to deliver the fetus safely, this is not the first action the nurse should take.) (c-Although this is appropriate, it is not the first action the nurse should take.)

a nurse is caring for a client who is in active labor when the client's membranes rupture. the fetal monitor tracing shows late decelerations. which of the following actions should the nurse take first? a. palpate the client's uterus b. administer oxygen to the client c. increase the client's IV fluid infusion rate d. turn the client onto her side

d (When using the urgent vs non-urgent approach to client care, the nurse determines that the priority action is to turn the client onto her left side. Late decelerations indicate that the client might have uteroplacental insufficiency, maternal hypotension, uterine tachysystole form oxytocin administration, or several other complicating factors. The client might be exerting pressure on the inferior vena cava, which decreases the oxygen to the placenta and thus to the fetus. Turning the client onto her side will relieve the pressure and facilitate better blood flow to the placenta, thereby increasing the fetal oxygen supply.) (a-The nurse should palpate the client's uterus to check for tachysystole. However, another action is the priority.) (b-The nurse should administer oxygen at 8 to 10 L/min by nonrebreather facemask to enhance placental perfusion. However, another action is the priority.) (c-The nurse should increase the client's IV fluid infusion rate to increase circulating fluid volume. However, another action is the priority.)


Ensembles d'études connexes

Most Common Themes in Literature

View Set

NetAcad Chapter 5 Exam Questions - 'Getting Help'

View Set

Business chapter 14 (final exam)

View Set